1intranet.tdmu.edu.ua/data/kafedra/internal/xirtop/zbtest... · web viewmodule 1. operative surgery...

214
Module 1. Operative surgery and topographical anatomy of the head and neck, regions and organs of the thoracic and abdominal cavities 1. Which of the following nerves lies on spermatic cord within inguinal canal? A. intercostalis B. anterior scrotal C. pudendal D. lumbalis E. ilioinguinalis * 2. Which of the following anatomical structures forms the anterior wall of sheath of rectus muscle at level of ribs? A. aponeurosis of external oblique abdominis muscle B. aponeurosis of internal oblique abdominis muscle C. aponeurosis of transverse abdominis muscle D. pyramidal muscle E. cartilages of V-VII ribs * 3. Why a surgeon can not cut compressed ring before incision of hernial sac? A. possibility of damage of parietal peritoneum B. possibility of damage of visceral peritoneum C. possibility of damage of contents of hernial sac D. inability to perform grafting E. because of necessity to check viability of compressed organs * 4. Which of the following anatomical structures belongs to the round ligament of liver? A. obliterated umbilical artery B. urachus C. ductus choledochus D. obliterated umbilical vein E. obliterated umbilical artery and vein * 5. During surgery injury of small intestine wall was founded. Which of the surgery should be performed? A. intestinal resection B. enterotomy C. enterostomy D. enteropexy E. intestinal suture * 6. A patient with perforated stomach ulcer has complication – subdiaphragmal abscess. Which of the following abdominal spaces should be drainaged first? A. omental bursa B. pancreatic C. left lateral abdominal canal

Upload: others

Post on 06-Apr-2020

23 views

Category:

Documents


2 download

TRANSCRIPT

Page 1: 1intranet.tdmu.edu.ua/data/kafedra/internal/xirtop/zbtest... · Web viewModule 1. Operative surgery and topographical anatomy of the head and neck, regions and organs of the thoracic

Module 1. Operative surgery and topographical anatomy of the head and neck, regions and organs of the thoracic and abdominal cavities

1. Which of the following nerves lies on spermatic cord within inguinal canal?A. intercostalisB. anterior scrotalC. pudendalD. lumbalisE. ilioinguinalis *2. Which of the following anatomical structures forms the anterior wall of sheath of rectus

muscle at level of ribs? A. aponeurosis of external oblique abdominis muscleB. aponeurosis of internal oblique abdominis muscleC. aponeurosis of transverse abdominis muscleD. pyramidal muscleE. cartilages of V-VII ribs *3. Why a surgeon can not cut compressed ring before incision of hernial sac? A. possibility of damage of parietal peritoneum B. possibility of damage of visceral peritoneumC. possibility of damage of contents of hernial sacD. inability to perform graftingE. because of necessity to check viability of compressed organs *4. Which of the following anatomical structures belongs to the round ligament of liver?A. obliterated umbilical arteryB. urachusC. ductus choledochusD. obliterated umbilical veinE. obliterated umbilical artery and vein *5. During surgery injury of small intestine wall was founded. Which of the surgery should be

performed?A. intestinal resectionB. enterotomyC. enterostomyD. enteropexyE. intestinal suture *6. A patient with perforated stomach ulcer has complication – subdiaphragmal abscess. Which

of the following abdominal spaces should be drainaged first?A. omental bursaB. pancreaticC. left lateral abdominal canalD. mesenteric sinusesE. hepatic bursa *7. During cholecystectomy bleeding from hepatic bed was found. The bleeding was arrested by

pressing of hepato-duodenal ligament in region of omental opening. Which of the following arteries was pressed in this case?

A. right hepatic arteryB. left hepatic arteryC. common hepatic arteryD. gastroepiploic arteryE. proper hepatic artery *8. What is the name of part of small intestine wall that is not covered with peritoneum?A. margo mesentericaB. margo epiploicaC. pars mesenterica

Page 2: 1intranet.tdmu.edu.ua/data/kafedra/internal/xirtop/zbtest... · Web viewModule 1. Operative surgery and topographical anatomy of the head and neck, regions and organs of the thoracic

D. pars epiploicaE. pars nuda *9. A. et v. lienales are within which of the following ligaments?A. lig. phrenico- colicumB. lig. hepato-duodenalisC. lig. phrenico-renaleD. lig. lieno-renaleE. lig. phrenico-lienale *10. A. cystica is branch of which of the following arteries?A. a. hepaticaB. a. hepatica sinistraC. a. hepatica propriaD. a. gastro-epiploica sinistraE. a. hepatica dextra *11. Lower wall of the omental bursa is formed by:A. peritoneum, aortaB. peritoneum, inferior caval veinC. hepato-duodenal ligament, hepato-renal ligamentD. mesentery of small intestineE. mesentery of transverse colon *12. Upper wall of the omental bursa is formed by:A. hepato-renal ligament, small curvature of the stomachB. hepato-duodenal ligament, lobus caudatus of liverC. superior part of duodenum, large curvature of the stomachD. all are correctE. lobus caudatus of liver, posterior wall of stomach *13. Posterior wall of the omental bursa is formed by:A. superior part of duodenum, large curvature of the stomachB. hepato-renal ligament, horizontal part of duodenumC. lobus caudatus of liver, posterior wall of stomachD. hepato-duodenal ligament, lobus caudatus of liverE. pancreas, inferior caval vein, aorta *14. Which of the following approaches to pancreas is most comfortable?A. Through mesentery of transverse colonB. extraperitoneal approachC. in according to site of surgeryD. through omental bursaE. through gastro-epiploic ligament *15. Porto-caval anastomoses are good developed in which of the following regions?A. stomach, caecumB. large intestine, stomachC. duodenum,large intestineD. caecum, large intestineE. esophagus, rectum, ventral abdominal wall *16. Which of the following ligaments should be cutted during mobilization of liver lobes?A. lig. coronariumB. lig. falciformeC. lig.hepatorenalisD. lig. hepatogastricumE. lig. triangulare dexter et sinister *17. Inferior caval vein lies within which of the following hepatic ligaments? A. lig. coronariumB. lig. triangulare dexterC. lig. triangulare sinister

Page 3: 1intranet.tdmu.edu.ua/data/kafedra/internal/xirtop/zbtest... · Web viewModule 1. Operative surgery and topographical anatomy of the head and neck, regions and organs of the thoracic

D. lig. falciformeE. lig.hepatorenalis *18. Identification of Latarzhe’s nerve is necessary in case of:A. resection of stomachB. gastrectomyC. apendectomyD. cholecystectomyE. selective vagotomy *19. Which of the following anatomical structures goes within gastro-pancreatic ligament?A. a. et v. lienalis sinistraeB. r. esophagealis from a. gastrica sinistraC. a. et v. gastrici sinistraeD. lymph nodesE. r. pancreaticus from a. gastroduodenalis *20. Right subdiaphragmatic space is bounded from above and from front by:A. intraabdominal fasciaB. lig. coronariumC. lig. triangulare dextrumD. superior-posterior surface of liverE. diaphragm *21. What is a Richter-Littre’s hernia?A. indirect inguinal herniaB. sliding herniaC. congenital herniaD. irreducible herniaE. incarcerated parietal hernia *22. Fibers of which of the following muscles should be cutted together with vaginal sheath of

spermatic cord in case of herniotomy?A. internal oblique muscleB. external oblique muscleC. transverse muscleD. pyramidal muscleE. cremaster muscle *23. Hernial sac in boys in case of congenital inguinal herniae is formed with which of the

following structures?A. wall of caval organB. skin, subcutaneous tissueC. peritoneumD. external spermatic fasciaE. vaginal appendage of peritoneum *24. What is congenital inguinal hernia?A. directB. direct and indirectC. reducibleD. incarceratingE. indirect *25. What is hernia when hernial sac formed with wall of caval organ?A. indirectB. irreducibleC. congenitalD. incarceratingE. sliding *26. What is Riolan’s arch?A. anastomosis between right and middle colic arteries

Page 4: 1intranet.tdmu.edu.ua/data/kafedra/internal/xirtop/zbtest... · Web viewModule 1. Operative surgery and topographical anatomy of the head and neck, regions and organs of the thoracic

B. anastomosis between iliac and colic arteriesC. anastomosis between caeliac trunk and superior mesenteric arteryD. anastomosis between caeliac trunk and inferior mesenteric arteryE. anastomosis between inferior and superior mesenteric arteries *27. Branches of superior mesenteric artery are:A. ilio-colic, right colicB. right colic, middle colicC. ileum, colic, middle colicD. right and middle colicE. ilio-colic, right colic, middle colic *28. Branches of inferior mesenteric artery are:A. right colic, middle colic, superior rectalB. left colic, middle colicA. C.colic, middle colic, sygmoidC. right colic, middle colicD. left colic, sygmoid, superior rectal *29. Sources of blood supply of duodenum are:A. caeliac trunkB. superior mesenteric arteryC. superior and inferior mesenteric arteriesD. short gastric branchesE. caeliac trunk and superior mesenteric artery *30. What is Gartmann’s pocket?A. dilatation of choledoch ductB. bend in place of gallbladder bodyC. dilatation of gallbladder fundusD. dilatation of gallbladderE. dilatation of gallbladder neck *31. Common hepatic artery is divided into:A. left and right hepatic arteriesB. left and right hepatic arteries and gallbladder arteryC. left and right hepatic arteries, gastro-duodenal arteryD. proper hepatic, gastric, duodenal arteriesE. proper hepatic and gastro-duodenal arteries *32. What is Gubarev’s sphincter?A. first narrowing of esophagusB. junction between pharynx and esophagusC. muscular layer of esophagusD. abdominal part of esophagusE. muscular layer of esophagus with muscles of diaphragm *33. Czerny’s suture should be done through which of the following layers of intestinal wall?A. serous, muscularB. muscular, submucous, mucousC. submucous, mucousD. serousE. serous, muscular, submucous, mucous *34. What is Lambert’s suture?A. PenetratingB. interruptedC. internalD. infectedE. seroso-muscular *35. What is cholecystoduodenoanastomosis?A. performing of fistula of choledoch duct

Page 5: 1intranet.tdmu.edu.ua/data/kafedra/internal/xirtop/zbtest... · Web viewModule 1. Operative surgery and topographical anatomy of the head and neck, regions and organs of the thoracic

B. performing of fistula of gallbladderC. performing of fistula between choledoch duct and stomachD. performing of fistula between choledoch duct and duodenumE. performing of fistula between gallbladder and duodenum *36. A patient with perforated stomach ulcer. A surgeon performed gastoraphy. The hole of

stomach in case of perforated ulcer should be stitched by which of the following sutures?A. two layers of sero-muscular with stiching of omentumB. marginal, sero-serous, Z-shapedC. sero-muscular, sero-serousD. marginal, sero-muscular, purse-stringE. marginal, sero-muscular with stiching of omentum *37. A surgeon completes the appendectomy. The stump of appendix should be immersed by

which of the following sutures?A. Albert’s sutureB. sero-serous sutureC. marginal sutureD. Schmiden's sutureE. purse-string suture *38. During cholecystectomy surgeon found Kallo’s triangle. This triangle is landmark for which

of the folowing structures?A. choledoch ductB. left hepatic arteryC. right hepatic arteryD. cystic ductE. cystic artery *39. Which of the following arteries should be left in case of subtotal stomach resection?A. right gastro-epiploic arteryB. common hepatic arteryC. right gastric arteryD. gastro-duodenal arteryE. left gastric artery *40. A surgeon performed removing of left part of large intestine because of tumor of intestine.

Which of the following arteries should be ligated and cutted?A. superior mesenteric arteryB. gastro-duodenal arteryC. right colic arteryD. splenic arteryE. left colic artery *

41. A surgeon performed removing of right part of large intestine. Which of the following arteries should be ligated?

A. а. colica sinistraB. а. gastroduodenalisC. а. mesenterica superiorD. а. mesenterica inferiorE. а. colica dextra *42. Stomach resection (Bilrot-II method) in case of tumor of pyloric part should be completed

by performing which of the following anastomoses?A. gastro-enteroanastomosis antecolica anteriorB. gastro-enteroanastomosis antecolica роsteriorC. gastro-enteroanastomosis retrocolica роsteriorD. gastro-duodenoanastomosis “termino-terminalis”E. gastro-enteroanastomosis retrocolica anterior *

Page 6: 1intranet.tdmu.edu.ua/data/kafedra/internal/xirtop/zbtest... · Web viewModule 1. Operative surgery and topographical anatomy of the head and neck, regions and organs of the thoracic

43. There was bleeding from gallbladder bed during antegrade cholecystectomy. The surgeon press lig. hepatoduodenale for temporary arrest bleeding. Which of the following vessels are within this ligament?

A. а. hepatica communisB. а. hepatica communis, v. роrtaC. а. суstica and v. роrtaD. а. суstica, v. роrtaE. а. hepatica propria, v. роrta *44. Stomach resection (Bilrot-I method) in case of bleeding ulcer should be completed by

performing which of the following anastomoses?A. gastro-duodenoanastomosis “end-to-side”B. gastro-duodenoanastomosis “side-to-side”C. gastro-enteroanastomosis “end-to-side”D. gastro-enteroanastomosis “side-to-side”E. gastro-duodenoanastomosis “end-to-end” *45. Ligation and cutting of proper hepatic artery was mistakenly performed during resection of

the pancreas. Which of the folowing arteries is main anastomosis for blood supply of pancreas?

A. inferior mesenteric arteryB. right gastricC. left gastricD. right gastro-epiploicE. gastro-duodenal *46. A patient with tumor has obstructive jaundice. Where tumor is localized?A. body of pancreasB. tail of pancreasC. spleenD. small intestineE. head of pancreas *47. A surgeon cut gastro-colic ligament during operative access to contents of omental bursa.

Which of the following arteries was ligated and cutted in this case?A. left and right gastricB. proper hepatic and gastro-duodenalC. proper gastricD. short gastricE. left and right gastro-epiploic *48. A surgeon found combined injury of duodenum and pancreas during examination of patient

with penetrating wound of abdomen. Which of the following parts of pancreas could be damaged?

A. neckB. body and tailC. tailD. body and neckE. head *49. Which of the following nerves innervates parietal peritoneum?A. vagus nerveB. sciatic nerveC. sympathetic nerveD. obturator nerveE. iliohypogastic nerve *50. What does “Caput Medusae” mean?A. distending of superior mesenteric veinB. distending of inferior mesenteric veinC. varicose veins of esophagus

Page 7: 1intranet.tdmu.edu.ua/data/kafedra/internal/xirtop/zbtest... · Web viewModule 1. Operative surgery and topographical anatomy of the head and neck, regions and organs of the thoracic

D. distending of veins of lower limb E. varicose veins in umbilical region *51. Which of the following veins connect superficial veins of umbilical region with portal

system?A. hepatic veinsB. gastric veinsC. superior epigastric veinD. inferior epigastric veinE. paraumbilical veins *52. Which of the following anatomical structures passes through inguinal canal in female?A. femoral nerveB. obturator nerveC. pudendal nerveD. ilio-hypogastric nerveE. ilio-inguinal nerve *53. Which of the following anatomical structures passes through inguinal canal in male?A. obturator arteryB. internal pudendal arteryC. pudendal nerveD. ilio-hypogastric nerveE. testicular artery *54. Which of the following herniae could be often incarcerated in female?A. umbilicalB. inguinalC. midline of abdomenD. obturatorE. femoral *55. External border of umbilical region is:A. medial edge of rectus muscleB. alba line of abdomenC. Spiegel’s lineD. Douglas’ lineE. lateral edge of rectus muscle *56. The most rare type of hernia is:A. alba line herniaB. postoperative herniaC. Spigel’s line herniaD. lumbar herniaE. xyphoid process hernia *57. Superior border of inguinal region is:A. lin. costarumB. lin albaC. lin. spinoumbilialisD. lin. pararectalisE. lin. spinarum *58. At which of the following levels median laparotomy is accompanied by incision of sheath of

rectus muscle?A. near the xyphoid processB. 2-3 см below umbilicusC. 3-4 см above umbilicuD. near the pubisE. at level of umbilicus *59. Which of the following anatomical structures forms posterior wall of sheath of rectus

muscle at 2-5 cm below umbilicus?

Page 8: 1intranet.tdmu.edu.ua/data/kafedra/internal/xirtop/zbtest... · Web viewModule 1. Operative surgery and topographical anatomy of the head and neck, regions and organs of the thoracic

A. aponeurosis of external oblique muscleB. aponeurosis of internal oblique muscleC. cartilages of V-VII ribsD. internal lamina of aponeurosis of internal oblique muscleE. transverse fascia *60. Pus in case of phlegmon of suprasternal interaponeurotic space is located between which of

the following neck fasciae?A. between I and II neck fasciaeB. between superficial and proper neck fasciaeC. between III and IV neck fasciaeD. between superficial and deep laminae of proper neck fasciaE. between II and III neck fasciae *61. In case of which of the following phlegmons pus could spreads in the mammary gland?A. phlegmon of suprasternal interaponeurotic spaceB. phlegmon of sheath of sternocleidomastoid muscleC. phlegmon of sac of submandibular glandD. phlegmon of Gruber’s sacE. phlegmon of interfascial space (between laminae of superficial fascia) *62. In case of which of the following phlegmons pus could spreads behind the mammary gland?A. phlegmon of suprasternal interaponeurotic spaceB. phlegmon of sheath of sternocleidomastoid muscleC. phlegmon of sac of submandibular glandD. phlegmon of Gruber’s sacE. subfascial phlegmon (between I and II fasciae) *63. V. jugularis externa formes at which of the following levels?A. level of cricoid cartilageB. level of thyroid cartilageC. level of 7-8 tracheal ringsD. level of thymusE. level of angle of mandible *64. Pus could spreads from sac of submandibular gland to_____________?A. Pyrogoff’s triangle B. carotid triangleC. interaponeurotic spaceD. Gruber’s sacE. along its duct to floor of mouth *65. V. facialis passes along which of the following surfaces of submandibular gland?A. internal surfaceB. lateral surfaceC. medial surfaceD. superior surfaceE. external surface *66. A. facialis passes along which of the following surfaces of submandibular gland?A. lateral surfaceB. external surfaceC. medial surfaceD. superior surfaceE. internal surface *67. Thymus is closely adjacent to which of the following anatomical structures?A. arch of aortaB. common carotid arteryC. superior caval veinD. thoracic ductE. mediastinal pleura *

Page 9: 1intranet.tdmu.edu.ua/data/kafedra/internal/xirtop/zbtest... · Web viewModule 1. Operative surgery and topographical anatomy of the head and neck, regions and organs of the thoracic

68. Which of the following operative approaches is best for incision of superior mediastinitis?A. sternotomyB. thoracotomyC. intercostalD. paravertebralE. cervical *69. At which of the following levels the pleural cavities are most closely adjacent to each other

in interpleural space?A. I ribB. V ribC. VI ribD. VII ribE. III rib *70. Pus could spreads to anterior mediastinum from which of the following cellular spaces of

neck? A. retrovisceralB. suprasternalC. submandibularD. prevertebralE. previsceral *71. Which of the following anatomical structures form anterior wall of sheath for rectus muscle

in fouth level?A. transverse fasciaB. cartilages of V-VII ribsC. pyramidal muscleD. parietal peritoneumE. aponeuroses of muscles of lateral part *72. Which of the following arteries realizes blood supply of thymus?A. arch of aortaB. lateral thoracic arteryC. descending pharyngeal arteryD. inferior thyroid arteryE. internal thoracic artery *73. Veins from thoracic part of esophagus flow into which of the following veins?A. inferior caval veinB. intercostal veinsC. internal thoracic veinD. superior caval veinE. azygos vein *74. Which of the following nerves innervates the esophagus?A. accessory nerveB. phrenic nerveC. subclavian nerveD. intercostal nerveE. vagus nerve *75. Posterior border of mediastinum is:A. sternumB. right sheet of mediastinal pleuraC. left sheet of mediastinal pleuraD. superior edge of manubrium sterniE. thoracic part of vertebrum *76. Skeletotopy of thoracic part of aorta?A. ТhV - TXIIB. ТhV - ThX

Page 10: 1intranet.tdmu.edu.ua/data/kafedra/internal/xirtop/zbtest... · Web viewModule 1. Operative surgery and topographical anatomy of the head and neck, regions and organs of the thoracic

C. ТhV - ThIXD. ТhVI - ThIXE. ТhIV – ThXI *77. How we should cut pericardium during operative approach to heart?A. transverse incisionB. combined incisionC. oblique incisionD. V-shaped incisionE. longitudinal incision *78. Where is located junction between right and left brachiocephalic veins to form superior

caval vein?A. at level of right sterno-clavicular jointB. at level of II right costal cartilage on sternal lineC. at level of III right costal cartilage on sternal lineD. at level of IV right costal cartilage on sternal lineE. at level of I right costal cartilage on sternal line *79. Where is Marphan’s point for pericardiocentesis? A. III intercostal space at left parasternal lineB. III intercostal space at right parasternal lineC. IV left intercostal space between parasternal and midclavicular lineD. at angle between xyphoid process and costal archE. III intercostal space at right parasternal line *80. Universal approach to heart is:A. paravertebralB. posterior-lateralC. lateralD. middle-lateralE. sternotomy *81. Which of the following nerves innervates the pericardium?A. supraclavicular nervesB. Letarzhe’s nerveC. accessory nerveD. lateral thoracic nerveE. intercostal nerve *82. The biggest sinus of pericardium is:A. transverseB. longitudinalC. verticalD. obligueE. anterio-lower *83. Which of the following complications could develop after pulmonectomy?A. hypotension in pulmonary arteryB. lymphostasisC. atelectasisD. hemopericardiumE. hypertension in pulmonary artery *84. Which of the following surgical approaches often is used in case of tumor of lung?A. lobectomyB. combined resection of lungC. segmental resection of lungD. bilobal resection of lungE. pulmonectomy *85. Which of the following surgical approaches is most comfortable for excision of middle lobe

of lung?

Page 11: 1intranet.tdmu.edu.ua/data/kafedra/internal/xirtop/zbtest... · Web viewModule 1. Operative surgery and topographical anatomy of the head and neck, regions and organs of the thoracic

A. anteriorB. lateralC. medianD. posteriorE. antero-lateral *86. On a hot summer evening, two not-overly-virtuous young ladies got into a scuffle over

which one would go home with a medical student. They came too near a powerful exhaust fan which caught the hair of one of them and scalped her. The bloody, hairy mass whirling in the fan had separated between the:

A. Skin and subcutaneous tissueB. Subcutaneous tissue and the aponeurosisC. Aponeurosis and the periosteum *D. Periosteum and the boneE. Outer table of the skull and the diploic space87. Cranial nerves transmitting parasympathetic preganglionic neurons include the:A. Oculomotor nerve *B. Trochlear nerveC. Abducens nerveD. Trigeminal nerveE. Hypoglossal nerve88. Match the posterior meningeal artery with its origin from the list below:A. Occipital *B. FacialC. Ascending pharyngealD. Posterior auricularE. Lingual89. The abducens nerve:A. Enters the orbit outside the fibrous ringB. Enters the ocular surface of the lateral rectus *C. Has its motor nucleus located in the midbrainD. Carries parasympathetic fibers to the lacrimal glandE. All of the above are true90. A patient with a lesion of the XII cranial nerve will present one of the following signs:A. Loss of sensation over the left margin of the tongueB. Partial loss of tasteC. Inability to protrude the tongue to the leftD. Loss of sensation from the left half of the dorsum of the tongueE. None of the these *91. The hypoglossal (XII) nerve:A. Emerges from the cranial cavity through the posterior condylar canalB. Has its cell bodies located in the lower ponsC. Is attached by rootlets to the medulla between the inferior cerebellar peduncleD. Supllies only the intrinsic muscles of the tongueE. None of the these *92. Following a cervical injury, a patient has loss of sensation over ann area lateral to the

ligamentum nuchae on one side indicating injury to a series of posterior primary rami. Which of the following muscles would probably not be paralyzed?

A. Splenius capitisB. Splenius cervicisC. Erector spinae (cervical portion)D. Longus colli *E. All of the above would be paralyzed93. The nerve to the mylohyoid passes:A. Deep to the mylohyoid muscle

Page 12: 1intranet.tdmu.edu.ua/data/kafedra/internal/xirtop/zbtest... · Web viewModule 1. Operative surgery and topographical anatomy of the head and neck, regions and organs of the thoracic

B. Medial to the medial pterygoid muscleC. Along the upper border of the hyoid boneD. Inferior to the mylohyoid line *E. None of these94. In breaking up a knife fight, a policeman receives a slash under the chin. Prior to suturing,

you explore the wound and find it extends down to but not through the hyoglossus muscle. Which of the following structures would you expect to find damaged?

A. The submandibular duct and the lingual nerve *B. The submandibular duct and nerveC. The glossopharyngeal nerve and lingual arteryD. The submandibular duct and the glossopharyngeal nerveE. The lingual artery and nerve95. A patient with carcinoma of the stomach has a small nodal metastasis in the ipsilateral (same

side) inferior deep cervical chain near the thoracic duct termination. The most probable complication would be:

A. Loss of cough reflexB. Phrenic nerve paralysis *C. Weakness of the rotator cuffD. A winged scapula (serratus anterior paralysis)E. None of these96. The superior cervical cardiac nerve originates from:A. Cervical nerves 3 and 4B. Cervical nerve 5 C. The superior cervical sympathetic ganglion *D. The middle cervical sympathetic ganglionE. The inferior cervical sympathetic ganglion97. An infiltrating tumor which involves the stellate ganglion (fused inferior cervical and first

thoracic sympathetic ganglion) would lead to which of the following symptoms on the involved side of the face?

A. Constriction of the pupilB. Constriction of the pupil and flushing of the skin *C. Excessive sweating and flushing of the skinD. Excessive dryness of the mouth and sweatingE. Constriction of the pupil and excessive sweating98. Match the artery to the parotid gland with its origin from the list below:A. OccipitalB. FacialC. Ascending pharyngealD. Posterior auricular *E. Lingual99. Match the artery to the sternocleidomastoid with its origin from the list below:A. Occipital *B. FacialC. Ascending pharyngealD. Posterior auricularE. Lingual100. Match the submental artery with its origin from the list below:A. OccipitalB. Facial *C. Ascending pharyngealD. Posterior auricularE. Lingual101. Match the artery to the submandibular gland with its origin from the list below:A. Occipital

Page 13: 1intranet.tdmu.edu.ua/data/kafedra/internal/xirtop/zbtest... · Web viewModule 1. Operative surgery and topographical anatomy of the head and neck, regions and organs of the thoracic

B. Facial *C. Ascending pharyngealD. Posterior auricularE. Lingual102. Which of the following surgical approaches is most comfortable for inferior

lobectomy? A. anteriorB. antero-lateral C. medianD. posteriorE. posterio-lateral *103. Which of the following surgical approaches is most comfortable for superior

lobectomy? A. anteriorB. antero-lateralC. postero-latearlD. posteriorE. lateral *104. Place for draining of pleural cavity in case of trauma of thorax:A. phrenico-mediastinal sinusB. VI intercostal space at scapular lineC. III intercostal space at midclavicular lineD. two drainage at posterior axillary and midclavicular linesE. VII intercostal space at posterior axillary line *105. Which of the following anatomical structures should be stitched in first layer in case

of small penetrating wound of chest wall?A. superficial muscle, superficial fasciaB. subcutaneous layer, superficial fasciaC. intercostal muscle, subcutaneous layerD. superficial fascia, subcutaneous layer, skinE. pleura, intrathoracic fascia, intercostal muscle *106. Which of the following anatomical structures should be stitched in second layer in

case of small penetrating wound of chest wall?A. pleura, intrathoracic fasciaB. intercostal musclesC. superficial fasciaD. subcutaneous tissue, skinE. superficial muscles *107. Which of the following anatomical structures should be stitched in third layer in case

of small penetrating wound of chest wall?A. pleura, intrathoracic fasciaB. intercostal musclesC. superficial muscles, superficial fasciaD. skin, intrathoracic fasciaE. skin, subcutaneous tissue, superficial fascia *108. Pleurocentesis in case of presence of air should be done at which of the following

lines? A. parasternalB. anterior axillaryC. posterior axillaryD. scapularE. midclavicular *109. Level of pleurocentesis (puncture of pleural cavity) in case f presence of air in

pleural cavity:

Page 14: 1intranet.tdmu.edu.ua/data/kafedra/internal/xirtop/zbtest... · Web viewModule 1. Operative surgery and topographical anatomy of the head and neck, regions and organs of the thoracic

A. I intercostal spaceB. IV-V intercostal spacesC. V intercostal spaceD. V-VI intercostal spacesE. II-III intercostal spaces *110. Level of puncture in case f presence of fluid in pleural cavity:A. IV intercostal spaceB. V intercostal spaceC. VI intercostal spaceD. IX-X intercostal spacesE. VII-VIII intercostal spaces *111. Puncture in case of presence of fluid in pleural cavity should be done at which of the

following lines?A. midclavicular lineB. between midclavicular and anterior axillary linesC. middle axillary lineD. paravertebralE. between middle axillary and scapular lines *112. Skin incision in case of lateral thoracotomy should be completed at which of the

following lines?A. middle axillary lineB. scapular lineC. paravertebral line D. parasternal lineE. posterior axillary line *113. Skin incision in case of lateral thoracotomy should be began at which of the

following levels?A. IV intercostal space at anterior axillary lineB. V intercostal space at middle axillary line, IV intercostal space at anterior axillary lineC. IV-V intercostal spaces at anterior axillary lineD. IV intercostal space at middle axillary line, V intercostal space at anterior axillary lineE. IV-V intercostal spaces at middle axillary line *114. Skin incision in case of posterior lateral thoracotomy should be completed at which

of the following levels?A. VI-VII ribs at posterior axillary lineB. V intercostal space at anterior axillary lineC. ІV intercostal space, middle axillary lineD. ІV intercostal space, posterior axillary lineE. VI-VII ribs at anterior axillary line or middle axillary line *115. Skin incision in case of posterior lateral thoracotomy should be began at which of the

following levels?A. posterior axillary line at level of spinal processes of V-IV thoracic vertebraeB. III-IV thoracic vertebrae at scapular lineC. V intercostal space at posterior axillary lineD. ІV intercostal space posterior axillary lineE. paravertebral line at level of I-II thoracic vetebrae *116. Skin incision in case of anterior lateral thoracotomy should be began at which of the

following levels?A. cartilage of III rib at middle axillary lineB. cartilage of IV rib at midclavicular lineC. cartilage of IV rib at anterior axillary lineD. V intercostal space at anterior axillary lineE. cartilage of III rib at parasternal line *

Page 15: 1intranet.tdmu.edu.ua/data/kafedra/internal/xirtop/zbtest... · Web viewModule 1. Operative surgery and topographical anatomy of the head and neck, regions and organs of the thoracic

117. Skin incision in case of anterior lateral thoracotomy should be completed at which of the following levels?

A. middle axillary lineB. scapular lineC. paravertebral lineD. midclavicular lineE. posterior axillary line *118. Foreign bodies of the upper airways in 70 % of cases fall in which of the following

bronchi?A. left main bronchusB. right lobar bronchusC. left lobar bronchusD. all are falseE. right main bronchus *119. Indicate relative position of components of root of right lung in frontal plane?A. artery, vein, bronchusB. artery, vein, nerveC. vein, bronchus, arteryD. vein, artery, bronchusE. bronchus, artery, vein *120. Indicate relative position of components of root of right lung in frontal plane A. artery, vein, bronchusB. vein, bronchus, arteryC. vein, artery, bronchusD. bronchus, vein, arteryE. artery, bronchus, vein *121. Lower border of lungs at scapular line is: A. upper edge of VI ribB. VIII ribC. IX ribD. X ribE. lower edge of VII rib *122. Lower border of lungs at paravertebral line is: A. upper edge of VI ribB. lower edge of VI ribC. IX ribD. X ribE. XI rib *123. Lower border of right lung at sternal line is:A. V ribB. VII ribC. VIII ribD. IX ribE. VI rib *124. Lower border of left lung at parasternal line is:A. V ribB. VII ribC. VIII ribD. IX ribE. VI rib *125. Lower border of lungs at midclavicular line is:A. upper edge of VI ribB. lower edge of VI ribC. lower edge of VII rib

Page 16: 1intranet.tdmu.edu.ua/data/kafedra/internal/xirtop/zbtest... · Web viewModule 1. Operative surgery and topographical anatomy of the head and neck, regions and organs of the thoracic

D. VIII ribE. upper edge of VII rib *126. Lower border of lungs at anterior axillary line is:A. upper edge of VII ribB. VIII ribC. IX ribD. X ribE. lower edge of VII rib *127. Lower border of lungs at middle axillary line is:A. lower edge of VI ribB. upper edge of VII ribC. lower edge of VII ribD. XI ribE. VIII rib *128. Projection of pleural cupula from the front is:A. spinal process of V cervical vertebraB. spinal process of VII cervical vertebraC. 1 сm below clavicleD. 1 сm above clavicleE. 2-3 сm above clavicle *129. Projection of pleural cupula from behind is:A. spinal process of V cervical vertebraB. spinal process of VI cervical vertebraC. 1 сm above clavicleD. 2-3 сm above clavicleE. spinal process of VII cervical vertebra *130. Which of the following borders of lungs is almost the same with borders of parietal

pleura?A. lateralB. lowerC. superiorD. all are not sameE. anterior, posterior *131. Which of the following organs usually is contents of diaphragmatic hernia?A. retroperitoneal space, left lobe of liverB. right lobe of liverC. anterior wall of stomachD. posterior wall of stomach, fundus of stomachE. cardial part of stomach *132. Which of the following weak places of diaphragm usually is place for entrance of

diaphragmatic hernia into posterior mediastinum?A. hiatus aorticusB. centrum tendineumC. hiatus inferior vena cava D. Bohdaleck’s triangle E. hiatus esophageus *133. Which of the following anatomical structures is weak place of diaphragm?A. hiatus aorticusB. centrum tendineumC. foramen venae cavae inferiorisD. pericardial bedE. lumbo-costal triangles *134. Which of the following anatomical structures goes between internal and middle

diaphragmatic peduncles?

Page 17: 1intranet.tdmu.edu.ua/data/kafedra/internal/xirtop/zbtest... · Web viewModule 1. Operative surgery and topographical anatomy of the head and neck, regions and organs of the thoracic

A. aorta, inferior vena cavaB. inferior vena cava, esophagusC. thoracic duct, vagus nerveD. sympathetic trunkE. azygos and hemiazygos veins *135. Which of the following anatomical structures goes between external and middle

diaphragmatic peduncles?A. aorta, inferior vena cavaB. esophagus, azygos veinC. hemiazygos veinD. thoracic duct, vagus nerveE. sympathetic trunk *136. Which of the following anatomical structures goes between internal diaphragmatic

peduncles?A. inferior vena cava, aortaB. azygos and hemiazygos veinsC. internal trunk, thoracic branches of axillary arteryD. sympathetic trunk, thoracic ductE. aorta, esophagus, thoracic duct, vagus nerve *137. Which of the following anatomical structures goes through centrum tendineum of the

diaphragm?A. aortaB. azygos veinC. hemiazygos veinD. thoracic ductE. inferior vena cava *138. The main way of collateral blood flow in case of stagnation in IVC system realizes

through: A. subclavian vein, lumbar veinB. common iliac veinC. bronchial veinD. brachiocephalic vein, pulmonary veinE. azygos and hemiazygos veins *139. Main way of lymph draining from mammary gland is:A. parasternal lymph nodesB. intercostal, mediastinal lymph nodesC. prevertebral lymph nodesD. visceral abdominal lymph nodesE. axillary lymph nodes *140. Sources of blood supply of mammary gland are:A. thyro-cervical trunk, subscapular arteryB. internal thoracic, subscapular arteriesC. thoraco-acromial, subclavian arteriesD. axillary, subscapular arteriesE. lateral thoracic artery, intercostal arteries, internal thoracic artery *141. Retromammary cellular space is located between which of the following anatomical

structures? A. skin, superficial fasciaB. subcutaneous tissue, proper fasciaC. clavi-pectoral fasciaD. internal thoracic fasciaE. superficial fascia, pectoral fascia *142. Which of the following anatomical structures provides mobility of mammary gland?A. skin, subcutaneous tissue

Page 18: 1intranet.tdmu.edu.ua/data/kafedra/internal/xirtop/zbtest... · Web viewModule 1. Operative surgery and topographical anatomy of the head and neck, regions and organs of the thoracic

B. skin, superficial fasciaC. ligamentum suspensorium mammaeD. pectoral fasciaE. retromammary cellular space *143. Place attachment of ligamentum suspensorium mammae is:A. sternal angleB. sternal bodyC. manubrium sterniD. acromial process of scapulaE. clavicle *144. Which of the following structures belong to fixative apparatus of mammary gland?A. muscles of thoraxB. vasculo-nervous bundlesC. retromammary cellular spaceD. fat tissue, vasculo-nervous bundlesE. ligamentum suspensorium mammae, fat tissue *145. Which of the following fasciae forms capsula of mammary gland?A. pectoralB. clavi-pectoralC. properD. internal thoracicE. superficial *146. Terminal branches of internal thoracic artery are:A. mediastinal, esophagealB. bronchialC. thoracicD. pericardo-phrenicE. musculo-diaphragmatic, superior epigastric *147. Pericardio-diaphragmatic artery arises from internal thoracic artery at which of the

following levels?A. II ribB. III ribC. V ribD. VI ribE. I rib *148. Internal thoracic artery locates at which of the following distance from lateral sternal

edge in retrosternal space at level of VI intercostal space? A. 2-3 mmB. 5-10 mmC. 10-13 mmD. 15-18 mmE. to 20 mm *149. Internal thoracic artery locates at which of the following distance from lateral sternal

edge in retrosternal space at level of I intercostal space?A. 2-3 mmB. 10-13 mmC. 15-18 mmD. tо 20 mmE. 5-10 mm *150. Internal thoracic artery locates at which of the following distance from lateral sternal

edge in retrosternal space at level of III intercostal space?A. 2-3 mmB. 5-10 mmC. 15-18 mm

Page 19: 1intranet.tdmu.edu.ua/data/kafedra/internal/xirtop/zbtest... · Web viewModule 1. Operative surgery and topographical anatomy of the head and neck, regions and organs of the thoracic

D. tо 20 mmE. 10-13 mm *151. Intercostal vasculo-nervous bundle is not covered by ribs in front of which f the

following lines?A. scapularB. posterior axillaryC. anterior axillaryD. midclavicularE. middle axillary *152. Indicate the reason of excessive bleeding from both ends of the damaged intercostal

vessels:A. high pressureB. slow venous outflowC. anastomosis with mediastinal arteryD. arterio-venous anastomosisE. anastomosis with internal thoracic artery *153. Muscular fibers of ______________muscles reach just until angle of ribs which

explains the possibility of involvement of intercostal nerves in the inflammatory process of the pleura?

A. external intercostalB. subcostal, external intercostalC. external and internal intercostalD. subcostalE. subcostal, internal intercostal *154. Muscular fibers which of the following muscles reach to sternum?A. external intercostalB. subcostalC. subcostal, external intercostalD. internal intercostal, subcostalE. internal intercostal *155. Distribution of purulent processes from the deep subpectoral cellular space is

possible in which of the following regions?A. subclavianB. scapular, subdeltoidC. subdeltoid, subclavianD. thoracic cavity, abdominal cavityE. supraclavicular, axillary, thoracic cavity *156. Incision in case of subpectoral phlegmon should be made along external edge of

_________ muscle for avoiding damage of clavi-pectoral fascia. A. lesser pectoral muscleB. subclavian muscleC. m. serratus anteriorD. lesser pectoral and subclavian musclesE. greater pectoral muscle *157. During incision of subpectoral phlegmon which of the following parts of clavi-

pectoral fascia should be saved for prevention distribution of inflammation to axillary fossa? A. coracoid processB. clavicularC. costalD. clavicular and costalE. suspending *158. Which of the following structures could be damaged in case of surgeries in

subclavian region within pectoral triangle? A. cephalic vein, superior thoracic artery

Page 20: 1intranet.tdmu.edu.ua/data/kafedra/internal/xirtop/zbtest... · Web viewModule 1. Operative surgery and topographical anatomy of the head and neck, regions and organs of the thoracic

B. thoraco-acromial artery, basilic veinC. subscapular artery, superior thoracic nerveD. suprascapular nerve, long thoracic nerveE. axillary artery, lateral thoracic artery, long thoracic nerve *159. Which of the following structures could be damaged in case of surgeries in

subclavian region within clavi-pectoral triangle? A. lateral thoracic arteryB. basilic vein, subscapular arteryC. superior thoracic nerve, long thoracic nerveD. long thoracic nerve, suprascapular nerveE. cephalic vein, superior thoracic artery, thoraco-acromial artery, superior thoracic nerve *160. Borders of phlegmon of deep subpectoral cellular space are:A. greater pectoral muscle (clavicular portion), greater pectoral muscle (sterno-costal portion)B. greater pectoral muscle (abdominal portion)C. m. serratus anterior, external intercostal musclesD. internal intercostal muscles, thoracic fascia, intrathoracic fasciaE. lesser pectoral muscle, clavi-pectoral fascia, thoracic fascia *161. Borders of phlegmon of superficial subpectoral cellular space are:A. greater pectoral muscle (clavicular portion), external intercostal musclesB. m. serratus anterior, internal intercostal musclesC. thoracic fascia, greater pectoral muscleD. intrathoracic fascia, thoracic fasciaE. greater pectoral muscle, lesser pectoral muscle, clavi-pectoral fascia *162. Which of the following vasculo-nervous bundles goes within subcutaneous tissue at

level of middle axillary line?A. internal thoracic artery, subclavian nerveB. costo-cervical artery, dorsal nerveC. superior thoracic artery, subclavian nerve D. dorsal nerve, internal thoracic arteryE. lateral thoracic artery, long thoracic nerve *163. Indicate the sources of superficial vessels of chest wall:A. transversal cervical arteryB. diaphragmatic arteryC. lateral thoracic artery, diaphragmatic arteryD. lateral thoracic artery, diaphragmatic artery, transversal cervical arteryA. E internal thoracic artery, intercostal arteries, lateral thoracic artery *164. Which of the following arteries goes in antescalenus space in transverse direction? A. subscapular arteryB. transverse cervical arteryC. superficial cervical arteryD. dorsal scapular arteryE. suprascapular artery *165. Which of the following nerves passes behind the right subclavian artery? A. phrenic nerveB. accessory nerveC. ansa cervicalis inferiorD. facial nerveE. reccurent laryngeal nerve *166. Which of the following complications could be developed in case of injure of neck

veins? A. veins thrombosis B. arterial thrombosisC. varicose veinsD. subcutaneous emphysema

Page 21: 1intranet.tdmu.edu.ua/data/kafedra/internal/xirtop/zbtest... · Web viewModule 1. Operative surgery and topographical anatomy of the head and neck, regions and organs of the thoracic

E. air embolism *167. Subclavian vein passes ________________ the anterior scalenus muscle. A. behindB. aboveC. in middle thirdD. benethE. in front of *168. Which of these fasciae are within occipital triangle?A. II, III, IVB. III, IV, VC. IV, VD. I, IV, VE. I, ІІ, V *169. Which of the following branches of subclavian artery passes between trunks of

brachial plexus? A. thyro-cervical trunkB. internal thoracic arteryC. costo-cervical trunkD. deep cervical arteryE. transverse cervical artery *170. Which of the following branches of subclavian artery makes anastomosis with

circumflex scapular artery in scapular region?A. transverse cervical aretryB. dorsal scapular arteryC. vertebral arteryD. inferior thyroid arteryE. suprascapular artery *171. Which f the following branches arises from subclavian artery before its entrance to

interscalenus space of neck?A. thyro-cervical trunkB. internal thoracic arteryC. costo-cervical trunkD. deep cervical arteryE. vertebral artery *172. Which of the following nerves descends on anterior surface of right subclavian artery

in its anterior part? A. vagus nerveB. ansa cervicalis inferiorC. recurrent laryngeal nerveD. all of theseE. phrenic nerve *173. Venous angle is formed by union which of the following veins?A. internal jugular vein, external jugula veinB. brachiocephalic vein, subclavian veinC. vertebral vein, external jugular veinD. deep cervical vein, brachiocephalic veinE. subclavian vein, internal jugular vein *174. Bottom of scaleno-vertebral triangle of neck is: A. subclavian veinB. subclavian arteryC. clavicleD. subclavian muscleE. pleural cupula *175. Which of the following nerves lies on anterior surface of anterior scalenus muscle?

Page 22: 1intranet.tdmu.edu.ua/data/kafedra/internal/xirtop/zbtest... · Web viewModule 1. Operative surgery and topographical anatomy of the head and neck, regions and organs of the thoracic

A. hypoglossal nerveB. accessory nerveC. superior laryngeal nerveD. greater occipital nerveE. phrenic nerve *176. Which of the following nerves forms thyroid nervous plexus?A. superior laryngeal nerveB. accessory nerveC. hypoglossal nerveD. phrenic nerveE. recurrent laryngeal nerve *177. Cellular space of medial vasculo-nervous bundle is bounded by which of the

following fasciae?A. ІB. ІІC. visceral lamina of IV fasciaD. VE. parietal lamina of IV fascia *178. Branches of which of these thyroid arteries is crossed by recurrent laryngeal nerve on

posterior surface of lateral lobe of thyroid gland? A. superior thyroid arteryB. proper thyroid arteryC. unpair thyroid arteryD. middle thyroid arteryE. inferior thyroid artery *179. Which fascia separates esophagus from long muscles of neck? A. IB. IIC. IIID. ІІІ; IVE. ІV; V *180. Which of the following incisions should be done for draining of cellular space of

medial vasculo-nervous bundle of neck?A. along anterior edge of sternocleidomastoid muscleB. along posterior edge of sternocleidomastoid muscleC. Kocher’s approachD. combinedE. along projection line of vasculo-nervous bundle *181. Which of the following vessels passes on anterior surface of trachea at level of

sterno-clavicular junction? A. arch of aortaB. isthmus of aortaC. brachio-cephalic trunkD. subclavian arteryE. comon carotid artery *182. Point of injection during vago-sympathetic block should be located at level of: A. middle of anterior edge of sternocleidomastoid muscleB. jugular notchC. midpoint of superior edge of esophagusD. along posterio-lateral surface of esophagusE. middle of posterior edge of sternocleidomastoid muscle *183. Middle fistula of neck is located at level of which of the following anatomical

structures: A. thyroid cartilage

Page 23: 1intranet.tdmu.edu.ua/data/kafedra/internal/xirtop/zbtest... · Web viewModule 1. Operative surgery and topographical anatomy of the head and neck, regions and organs of the thoracic

B. cricoid cartilageC. arytenoid cartilageD. corniculate cartilageE. hyoid bone *184. In which direction thyro-cricoid cartilage should be cutted in case of conycotomy? A. longitudinalB. obliqueC. combinedD. radialE. transversal *185. Superior thyroid artery is branch of which of the following arteries?A. internal carotid arteryB. superior laryngeal arteryC. inferior laryngeal arteryD. subclavian arteryE. external carotid artery *186. Inferior thyroid artery is branch of which of the following arteries?A. common carotid arteryB. internal carotid arteryC. subclavian arteryD. arch of aortaE. thyro-cervical trunk *187. Superior cervical node of sympathetic trunk is crossed from front by vagus nerve at

which of the following cervical vertebrae? A. IB. IIC. IVD. VE. III *188. Which of the following arteries arises from external carotid artery at level of greater

cornu of hyoid boneA. superior laryngeal arteryB. superior thyroid arteryC. ascending laryngeal arteryD. lingual arteryE. facial artery *189. Which nerves form synocarotid zone in region of bifurcation of common carotid

artery?A. facial, phrenic nerves, sympathetic trunkB. lingual, facial, phrenic nervesC. laryngeal recurrent, lingual, facial nervesD. glossopharyngeal, lingual, facial nervesE. vagus, hypoglossal nerves, sympathetic trunk *190. Which of these nerves innervates platysma?A. lingual nerveB. mandibular nerveC. transverse cervical nerveD. sympathetic trunkE. facial nerve *191. Which of the following nerves associates with nerves of cervical plexus to form

superior root of ansa cervicalis? A. lingual nerveB. facial nerveC. thyro-hyoid nerve

Page 24: 1intranet.tdmu.edu.ua/data/kafedra/internal/xirtop/zbtest... · Web viewModule 1. Operative surgery and topographical anatomy of the head and neck, regions and organs of the thoracic

D. transversal cervical nerveE. hypoglossal nerve *192. Pulsation of superficial temporal artery could be palpated:A. 2 cm in front of tragus B. midpoint of zygomatic archC. at the external corner of eyeD. at external edge of temporal boneE. 1,2 cm in front of tragus *193. Which of the following directions of incisions of soft tissue of skull to avoid injury

of large vessels and nerves?A. longitudinalB. transversalC. semilunarD. cmbinedE. radial *194. Interpterygoid interfascial cellular space is located in which of the following regions

of head? A. temporal regionB. buccal regionC. occipital regionD. oral cavityE. deep region of face *195. Specify the characteristic of facial vein:A. undulating courseB. presence of valvesC. presence of veno-arterial anastomosesD. thick wallE. valveless *196. Specify the characteristic of facial vein:A. undulating courseB. thick wallC. situated near the corner of the mouthD. situated at midpoint of anterior edge of masseter muscleE. straight course *197. Specify the characteristic of facial artery:A. straight courseB. thick wallC. presence of valvelesD. valvelessE. undulating course *198. Which of the following approaches is used for cervical part of esophagus?A. along anterior edge of right sternocleidomastoid muscleB. Kocher’s approachC. combinedD. radialE. along anterior edge of left sternocleidomastoid muscle *199. Which of the following structures lies most closely to esophagus within carotid

triangle?A. internal jugular veinB. vagus nerveC. external jugular veinD. hypoglossal nerveE. common carotid artery *

Page 25: 1intranet.tdmu.edu.ua/data/kafedra/internal/xirtop/zbtest... · Web viewModule 1. Operative surgery and topographical anatomy of the head and neck, regions and organs of the thoracic

200. Weak place of bed of submandibular gland is located in slit between which of the following muscles?

A. digastric, sternocleidomastoid musclesB. bellies of digastric muscleC. digastric, stylohyoid musclesD. geniohyoid, digastric musclesE. mylohyoid, hyoglossus muscles *201. Which of these structures could not be damaged by a deep laceration superior to the

superior nuchal line?A. Occipital arteryB. Greater occipital nerveC. Nuchal ligament *D. Lesser occipital nerveE. Occipitofrontalis muscle.202. Which of the following statements about the scalp is not true?A. Loose areolar tissue occupies the subaponeurotic spaceB. The prolific arterial blood supply of the scalp is in the dense subcutaneous tissueC. The scalp is innervated by the infraorbital nerve *D. The subaponeurotic layer is sometimes referred to as the danger space of the scalpE. All of the above statements are true.203. The sensory nerves of the scalp always include branches from the following nerve

except:A. Greater occipital nerve (C2)B. Lesser occipital nerve (C2)C. Posterior primary ramus of C1 (least occipital nerve) *D. Supraorbital branch of the ophthalmic division of VE. Al of the above.204. A bump on the head produced a “goose egg” while a punch in the face produces a

black eye because:A. The subcutaneous tissue around the eye is very loose while that in the scalp is dense *B. There is subcutaneous muscle in the face but not the scalpC. The skin over the face is thick while that over the scalp is thinD. There are no lymphatics in the scalpE. The statement is simply a fact without anatomical explanation. 205. The foramen ovale in the skullA. Allows entrance of the spinal part of the accessory nerve into the cranial cavityB. Is located in the petrous part of the temporal boneC. Allows entrance of the middle meningeal artery into the cranial cavityD. Allows exit of the mandibular division of the trigeminal nerve *E. Allows exit of the glossopharyngeal nerve.206. The foramen spinosum in the skullA. Allows exit of the facial nerveB. Is located in the lesser wing of the sphenoidC. Allows entrance of the middle meningeal artery into the cranial cavity *D. Allows exit of the maxillary division of the trigeminal nerveE. Allows passage only of emissary veins.

207. The internal acoustic meatus in the skullA. Is located in the body of the sphenoid boneB. Is located in the mastoid boneC. Allows passage of the glossopharyngeal nerveD. Allows passage of the facial nerve onlyE. Allows passage of the vestibulocochlear nerve and the facial nerve. *208. The foramen magnum in the skullA. Allows entrance of the spinal part of the accessory nerve into the cranial cavity *

Page 26: 1intranet.tdmu.edu.ua/data/kafedra/internal/xirtop/zbtest... · Web viewModule 1. Operative surgery and topographical anatomy of the head and neck, regions and organs of the thoracic

B. Allows exit of the spinal part of the accessory nerve out of the cranial cavityC. Is located in the sphenoid boneD. Is located in the temporal boneE. Allows entrance of the cranial part of the accessory nerve.209. The jugular foramen in the skullA. Is located in the petrous part of the temporal boneB. Allows exit of the hypoglossal nerveC. Is located in the middle cranial fossaD. Allows exit of the vagus nerve *E. Allows entrance of the external jugular vein.210. The middle ethmoid sinuses drain into theA. Middle meatus of the nose *B. Superior meatus of the noseC. Sphenoethmoidal recessD. Inferior meatus of the noseE. Nasolacrimal duct.211. The sphenoid sinus drains into theA. Middle meatus of the noseB. Superior meatus of the noseC. Sphenoethmoidal recess *D. Inferior meatus of the noseE. Nasolacrimal duct.212. The frontal sinus drain into theA. Middle meatus of the nose *B. Lacrimal sacC. Superior meatus of the noseD. Sphenoethmoidal recessE. Inferior meatus of the nose.213. The anterior ethmoidal sinuses drain into the A. Lacrimal sacB. Superior meatus of the noseC. Middle meatus of the nose *D. Sphenoid sinusE. Inferior meatus of the nose.214. The nasolacrimal duct drains into theA. Lacrimal sacB. Inferior meatus of the nose *C. Superior meatus of the noseD. Middle meatus of the noseE. Sphenoethmoidal recess.215. The mylohyoid muscle:A. Is separated from the anterior belly of the digastric by the nerve to the mylohyoid and

submental branch of the facial artery *B. Is supplied by its own nerve from the submandibular branch of the facial nerveC. Separates the submandibular lumph glands from the platysma muscleD. Is supplied by the submandibular branch of the facial nerve which also innervates the

anterior belly of the digastric muscleE. All of the above are true.216. Which muscle found on the face shares a common origin and fascial covering with a

muscle of the pharynx?A. Orbicularis orisB. Depressor anguli orisC. RisoriusD. Caninus

Page 27: 1intranet.tdmu.edu.ua/data/kafedra/internal/xirtop/zbtest... · Web viewModule 1. Operative surgery and topographical anatomy of the head and neck, regions and organs of the thoracic

E. Buccinator. *217. Which of the following relations of the hyoglossus muscle is incorrectly stated? It is

deep to the:A. Lingual artery *B. Hypoglossal nerveC. Lingual nerveD. Mylohyoid muscleE. Submandibular gland.218. Which of these statements are not true?A. The stylohyoid muscle is supplied by the facial nerveB. The styloglossus muscle is supplied by the hypoglossal nerveC. The posterior belly of digastric muscle is supplied by the trigeminal nerve *D. The stylopharyngeal muscle is supplied by the glossopharyngeal nerveE. All of these statements are true.219. Which of the following statements concerning the temporalis muscle are true?A. It is relatively small in the newbornB. It inserts into the medial surface of the coronoid processC. It is innervated by the mandibular division of VD. By contraction of the posterior fibers it retracts the mandibleE. All of the above are true.220. The lateral pterygoids may cause which of the following actions of the mandible?A. DepressionB. ProtractionC. Side to side movementD. All of the above *E. None of the above.221. The transverse facial artery is usually a branch of the:A. Superficial temporal artery *B. Maxillary arteryC. Facial arteryD. Angular arteryE. Infraorbital artery.222. Which of the following arteries does not accompany its correspondingly named

nerve throughout most of its course?A. Lingual *B. Posterior superior alveolarC. Inferior alveolarD. Greater palatineE. Infraorbital.223. The maxillary artery is distributed to about the same regions and structures as those

supplied by the:A. Ophthalmic nerveB. Maxillary nerveC. Mandibular nerveD. Ophthalmic and mandibular nervesE. Maxillary and mandibular nerves. *224. Which of the following structures is not supplied by the ophthalmic artery?A. RetinaB. Frontal sinusC. Ethmoid air cellsD. Lateral wall of the noseE. All of these are supplied. *225. The maxillary artery supplies all of the following areas except the:A. Upper eyelid *

Page 28: 1intranet.tdmu.edu.ua/data/kafedra/internal/xirtop/zbtest... · Web viewModule 1. Operative surgery and topographical anatomy of the head and neck, regions and organs of the thoracic

B. Lower eyelidC. MeningesD. Upper teethE. Lower teeth.226. The maxillary artery has sixteen or seventeen branches. From the selection given

here, indicate the most important branch for anyone in medical practice to know:A. Deep auricularB. MassetericC. Middle meningeal *D. Anterior deep temporalE. Infraorbital.227. Which of the following branches does not arise from the facial artery?A. LabialB. TonsillarC. SubmentalD. Stylomastoid *E. Submandibular.228. Which of the following is not a branch of the maxillary artery?A. Artery of the pterygoid canalB. Mylohyoid artery *C. Pharyngeal arteryD. Infraorbital arteryE. Posterior superior alveolar artery.229. Which of the following structures is supplied by the facial artery?A. TonsilB. Soft palateC. Sublingual glandD. Anterior belly of digastricE. All of these structures are supplied by the facial artery. *230. Following a boil on the upper lip, a rural lowan became comatose. The infection has

spread to the cavernous sinus from the facial vein by its communication with the:A. Occipital sinusB. Straight sinusC. Superior saggital sinusD. Maxillary sinusE. Ophthalmic veins. *231. Tributaries of the internal jugular vein include the:A. Common facial veinB. Inferior petrosal sinusC. Middle meningeal veinD. Inferior alveolar veinE. All of these. *232. The pterygoid plexus receives tributaries from the:A. Sphenopalatine veinB. Deep temporal veinC. Middle meningeal veinD. Inferior alveolar veinE. All of these. *233. In the root of the neck the left vagus nerve lies:A. Anterior to the sympathetic trunk *B. Medial to the common carotid arteryC. Lateral to the phrenic nerveD. Posterior to the subclavian arteryE. All of the above are true.

Page 29: 1intranet.tdmu.edu.ua/data/kafedra/internal/xirtop/zbtest... · Web viewModule 1. Operative surgery and topographical anatomy of the head and neck, regions and organs of the thoracic

234. Around which artery does the right recurrent laryngeal nerve hook as it passes to the larynx?

A. The aortic archB. The brachiocephalic trunkC. The right subclavian artery *D. The right common carotid arteryE. The right inferior thyroid artery.235. An invasive tumor of the thyroid gland is least likely to result in which of the

following?A. HoarsenessB. Tracheal obstructionC. Phrenic nerve dysfunction *D. Cervical sympathetic dysfunctionE. Lymph node enlargement in the neck.236. Which one of the following structures does not come into contact with the thyroid

gland or its capsule?A. The right recurrent laryngeal nerveB. The left vagus nerve *C. The left common carotid arteryD. The esophagusE. All of the above come into contact with the thyroid gland or its capsule.237. The arterial supply to the larynx includes branches of the:A. Lingual arteryB. Superior thyroid arteryC. Facial arteryD. Inferior thyroid arteryE. Superior and inferior thyroid arteries. *238. The following statements concerning an intercostal space are correct except which?

A. The anterior intercostals arteries of the lower five intercostals spaces are branches of the musculophrenic artery

B. The sensory fibers in the lower five intercostals nerves supply the skin of lateral thoracic and anterior abdominal walls

C. The posterior intercostals arteries of the lower nine spaces are branches of the thoracic aorta

D. Throughout an intercostals space, the intercostals nerves and blood vessels lie close to the upper border of the lower rib*

E. The intercostals nerves and blood vessels run between the internal and the innermost intercostals muscles.

239. The following statements concerning the structure of the heart are correct except which?A. The trabeculae carneae are internal surface structures of both the left and the right

ventriclesB. The pericardial cavity is the potential space between the fibrous and the serous

pericardia*C. The coronary arteries are functional end arteriesD. The sinuatrial node is supplied by the right and sometimes the left coronary arteryE. The four pulmonary veins open through the posterior wall of the left atrium and there are

no valves.240. The following statements regarding the innervation of thoracic structures are correct

except which?A. The lung and visceral pleura are innervated by the autonomic nerves and are not

sensitive to sensation of temperature, touch, and pressureB. The motor innervation of the diaphragm is provided by the third, fourth, and fifth

cervical spinal nerves and by the lower six intercostals nerves*

Page 30: 1intranet.tdmu.edu.ua/data/kafedra/internal/xirtop/zbtest... · Web viewModule 1. Operative surgery and topographical anatomy of the head and neck, regions and organs of the thoracic

C. The sensory nerve supply to the mucous membrane of the lower part of the trachea is from the vagus and the recurrent laryngeal nerves

D. The nerve supply of the pericardium is the phrenic nervesE. The sinuatrial node is supplied by sympathetic and parasympathetic nerves via the

cardiac plexuses.241. The following statements concerning thoracic structures are correct except which?

A. The carina is the name given to the site of bifurcation of the tracheaB. The ligamentum arteriosum is the remains of the ductus arteriosusC. The ductus arteriosus is formed from the sixth left pharyngeal archD. The thymus lies in the middle mediastinum*E. The thymus receives its arterial supply mainly from the internal thoracic artery.

242. The following events occur on inhalation except which?A. The diaphragm descendsB. The external intercostals muscles contractC. The abdominal muscles contract and push the abdominal viscera cranially *D. The ribs are raisedE. The vertical dimension of the thoracic cavity increases.

243. When passing a needle through the chest wall and into the pleural cavity in the midaxillary line, the following structures will be pierced except which?

A. The external intercostal muscleB. The skinC. The parietal pleuraD. The levator costarum *E. The internal intercostal muscle.

244. The following statements concerning the bronchopulmonary segments are correct except which?

A. It is a subdivision of a lung lobeB. It is pyramidal in shape, with its apex toward the lung surface *C. It is surrounded by connective tissueD. It has a segmental bronchus, a segmental artery, lymph vessels, and autonomic nervesE. When diseased, it can be removed surgically as a structural unit.245. The following statements concerning the main bronchi are correct except which?A. The right main bronchus is wider than the left main bronchusB. The right main bronchus is shorter than the left main bronchusC. The right main bronchus is m than the left main bronchusD. The left main bronchus passes to the left in front of the esophagusE. The left main bronchus gives off the superior lobar bronchus before entering the hilum of

the lung. *246. The following structures open into the right atrium except which?A. The superior vena cavaB. The coronary sinusC. The anterior cardiac veinD. The inferior vena cavaE. The right pulmonary veins. *247. The conducting system of the heart is composed of the following structures except

which?A. The Purkinje plexusB. The deep cardiac plexus *C. The sinuatrial nodeD. The atrioventricular bundleE. The atrioventricular node.248. Pain due to a gastric ulcer may be referred to the

A. Umbilical regionB. Right iliac region

Page 31: 1intranet.tdmu.edu.ua/data/kafedra/internal/xirtop/zbtest... · Web viewModule 1. Operative surgery and topographical anatomy of the head and neck, regions and organs of the thoracic

C. Epigastric region *D. Penis or clitorisE. None of the above.

249. Pain caused by appendicitis may first be referred to the A. Right iliac regionB. Umbilical region *C. Point of the shoulderD. Epigastric regionE. Below the right shoulder blade.250. Interminent pain (colic) in the small intestine may be referred toA. The epigastric regionB. The left iliac regionC. Just above the symphysis pubisD. The umbilical region *E. None of the above.251. Pain caused by the passage of a stone down the lower end of the left ureter may be

referred to theA. Umbilical regionB. Right iliac regionC. Epigastric regionD. Penis or clitoris *E. None of the above.252. The short gastric arteries originate from the

A. Superior mesenteric arteryB. Hepatic arteryC. Splenic artery *D. Inferior phrenic arteryE. Left renal artery.

253. The right gastric artery originates from theA. Inferior mesenteric arteryB. Superior mesenteric arteryC. Hepatic artery *D. Gastroduodenal arteryE. None of the above.254. The left gastric artery originates from theA. Superior mesenteric arteryB. Left renal arteryC. Splenic arteryD. Celiac artery *E. Hepatic artery.255. The gastroduodenal artery originates from theA. Splenic arteryB. Hepatic artery *C. Superior mesenteric arteryD. Right renal arteryE. Celiac artery.256. The left gastroepiploic artery originates from theA. Inferior mesenteric arteryB. Superior mesenteric arteryC. Splenic artery *D. Celiac arteryE. Left renal artery257. The descending colon receives parasympathetic nerves from theA. Pelvic splanchnic nerves *

Page 32: 1intranet.tdmu.edu.ua/data/kafedra/internal/xirtop/zbtest... · Web viewModule 1. Operative surgery and topographical anatomy of the head and neck, regions and organs of the thoracic

B. Spinal cord segments L1 and 2C. Greater splanchnic nerveD. Lesser splanchnic nerveE. Vagus nerve.258. The jejunum receives sympathetic nerves from theA. Vagus nerveB. Pelvic splanchnic nervesC. Spinal cord segments L1 and 2D. Greater splanchnic nerve and lesser splanchnic nerve *E. Spinal cord segments S2, 3 and 4.259. The sigmoid colon receives sympathetic nerves from theA. Lesser splanchnic nerveB. Vagus nerveC. Greater splanchnic nerveD. Spinal cord segments L1 and 2 *E. Spinal cord segments T5 to 9.260. The ileum receives parasympathetic nerves from theA. Vagus nerve *B. Pelvic splanchnic nervesC. Lesser splanchnic nerveD. Spinal cord segments T10 and 11E. Inferior mesenteric plexus.261. The azygos vein passes _______ in the diaphragm.A. Through the aortic hiatus *B. Through the esophageal hiatusC. Through the vena caval hiatusD. Through perforations in the cruraE. Between the slips of origin of the diaphragm from the ribs.262. The thoracic duct passes through _______ in the diaphragm.A. The esophageal hiatusB. The vena caval hiatusC. The aortic hiatus *D. Perforations in the cruraE. None of the above.263. The vagus nerves pass _______ in the diaphragm.A. Through the vena caval hiatusB. Through the aortic hiatusC. Between the slips of origin of the diaphragm from the ribsD. Through the esophageal hiatus *E. Through the perforations in the crura.264. Lymphatic spread of carcinoma of the fundus of the stomach is likely to metastasize

to theA. Internal iliac nodesB. Superior mesenteric nodesC. Inferior mesenteric nodesD. Celiac nodes *E. Right gastroepiploic nodes.265. Infection from an inflamed appendix is likely to spread via the lymphatics to theA. Superior mesenteric nodes *B. Celiac nodesC. Splenic nodesD. Inferior mesenteric nodesE. Right internal iliac nodes.266. In both sexes, the inguinal canal is formed by the

Page 33: 1intranet.tdmu.edu.ua/data/kafedra/internal/xirtop/zbtest... · Web viewModule 1. Operative surgery and topographical anatomy of the head and neck, regions and organs of the thoracic

A. Descent of the gonadB. Contraction of the gubernaculumsC. Processus vaginalis *D. Growth and descent of the ilioinguinal nerveE. Contraction of the cremaster muscle.267. The lacunar ligament isA. Formed from the conjoint tendonB. Part of the posterior wall of the rectus sheathC. Not continuous with the inguinal ligamentD. An important medial relation to the femoral ring of the femoral sheath *E. Attached to the inferior ramus of the pubis.268. Which of the following statements about the portal vein is untrue?A. It lies near the free edge of the hepatoduodenal ligamentB. It is formed by the junction of the superior mesenteric and splenic veinsC. It lies posterior to the common bile ductD. It lies posterior to the hepatic arteryE. All of the above are true *269. A surgeon wishes to anastomose the splenic and renal veins to treat portal venous

hypertension. After removing the spleen, the free end of the splenic vein is to be found in the:

A. Gastrophrenic ligamentB. Lienorenal ligament *C. Gastrosplenic ligamentD. Left edge of the lesser omentumE. None of the above places270. Advanced cirrhosis of the liver often results in portal hypertension. Which of the

following signs is not directly related to portal hypertension?A. Gastroesophageal varicesB. Hemorrhoids (varicose veins in the rectum)C. Caput medusae (dilation of the superficial umbilical veins)D. Ascites (accumulation of fluid in the abdominal cavity)E. Varicocele (varicose veins of the pampiniform plexus) *271. The foramen ovale in the skullA. Allows entrance of the spinal part of the accessory nerve into the cranial cavityB. Is located in the petrous part of the temporal boneC. Allows entrance of the middle meningeal artery into the cranial cavityD. Allows exit of the mandibular division of the trigeminal nerve *E. Allows exit of the glossopharyngeal nerve.272. The foramen spinosum in the skullA. Allows exit of the facial nerveB. Is located in the lesser wing of the sphenoidC. Allows entrance of the middle meningeal artery into the cranial cavity *D. Allows exit of the maxillary division of the trigeminal nerveE. Allows passage only of emissary veins.273. The internal acoustic meatus in the skullA. Is located in the body of the sphenoid boneB. Is located in the mastoid boneC. Allows passage of the glossopharyngeal nerveD. Allows passage of the facial nerve onlyE. Allows passage of the vestibulocochlear nerve and the facial nerve. *274. The foramen magnum in the skullA. Allows entrance of the spinal part of the accessory nerve into the cranial cavity *B. Allows exit of the spinal part of the accessory nerve out of the cranial cavityC. Is located in the sphenoid bone

Page 34: 1intranet.tdmu.edu.ua/data/kafedra/internal/xirtop/zbtest... · Web viewModule 1. Operative surgery and topographical anatomy of the head and neck, regions and organs of the thoracic

D. Is located in the temporal boneE. Allows entrance of the cranial part of the accessory nerve.275. The jugular foramen in the skullA. Is located in the petrous part of the temporal boneB. Allows exit of the hypoglossal nerveC. Is located in the middle cranial fossaD. Allows exit of the vagus nerve *E. Allows entrance of the external jugular vein.276. The middle ethmoid sinuses drain into theA. Middle meatus of the nose *B. Superior meatus of the noseC. Sphenoethmoidal recessD. Inferior meatus of the noseE. Nasolacrimal duct.277. The sphenoid sinus drains into theA. Middle meatus of the noseB. Superior meatus of the noseC. Sphenoethmoidal recess *D. Inferior meatus of the noseE. Nasolacrimal duct.278. The frontal sinus drain into theA. Middle meatus of the nose *B. Lacrimal sacC. Superior meatus of the noseD. Sphenoethmoidal recessE. Inferior meatus of the nose.279. The anterior ethmoidal sinuses drain into the A. Lacrimal sacB. Superior meatus of the noseC. Middle meatus of the nose *D. Sphenoid sinusE. Inferior meatus of the nose.280. The nasolacrimal duct drains into theA. Lacrimal sacB. Inferior meatus of the nose *C. Superior meatus of the noseD. Middle meatus of the noseE. Sphenoethmoidal recess.281. Which of these structures could not be damaged by a deep laceration superior to the

superior nuchal line?A. Occipital arteryB. Greater occipital nerveC. Nuchal ligament *D. Lesser occipital nerveE. Occipitofrontalis muscle.282. Which of the following statements about the scalp is not true?A. Loose areolar tissue occupies the subaponeurotic spaceB. The prolific arterial blood supply of the scalp is in the dense subcutaneous tissueC. The scalp is innervated by the infraorbital nerve *D. The subaponeurotic layer is sometimes referred to as the danger space of the scalpE. All of the above statements are true.283. The sensory nerves of the scalp always include branches from the following nerve

except:A. Greater occipital nerve (C2)

Page 35: 1intranet.tdmu.edu.ua/data/kafedra/internal/xirtop/zbtest... · Web viewModule 1. Operative surgery and topographical anatomy of the head and neck, regions and organs of the thoracic

B. Lesser occipital nerve (C2)C. Posterior primary ramus of C1 (least occipital nerve) *D. Supraorbital branch of the ophthalmic division of VE. Al of the above.284. A bump on the head produced a “goose egg” while a punch in the face produces a

black eye because:A. The subcutaneous tissue around the eye is very loose while that in the scalp is dense *B. There is subcutaneous muscle in the face but not the scalpC. The skin over the face is thick while that over the scalp is thinD. There are no lymphatics in the scalpE. The statement is simply a fact without anatomical explanation.

285. When trying to locate the parotid duct, a physician would consider each of the following relationships EXCEPT:A. its opening can be seen in the vestibule of the mouth opposite the upper 2nd premolar tooth B. it extends from the anterior border of the parotid gland C. it can be palpated as it crosses the face, superficial to the masseter muscle D. it is inferior to the zygomatic arch E. it is superior to the zygomatic arch *286. Which of the following statements best describes the facial vein?A. it is located within the substance of the parotid gland B. it communicates superiorly with the ophthalmic vein *C. it is more tortuous than the facial artery D. it lies anterior to the facial artery as it passes through the face E. it usually empties into the external287. The tickling sensation felt in the nasal cavity, just prior to a sneeze is probably carried in which of the following nerves?A. facial B. maxillary division of trigeminal *C. mandibular division of the trigeminal D. glossopharyngeal E. none of the above 288. Tumors of the head may grow from one region of the head to another by passing through fissures and foramina. Knowing this, if you found a tumor in the pterygopalatine fossa, it may have developed there primarily or it may have grown into the fossa from any of the following EXCEPT:A. infratemporal fossa B. cranial cavity by way of the foramen ovale *C. cranial cavity by way of the foramen rotundum D. nasal cavity E. oral cavity by way of the greater289. The 4th cranial nerve (trochlear) innervates:A. the lacrimal caruncle B. a muscle that turns the eyeball superiorly and laterally C. the lacrimal gland D. the medial part of the lower eyelid E. a muscle that turns the eyeball inferiorly *

290. During a physical examination, you have a patient stick out his/her tongue and say AAH. The muscle that is responsible for this movement is the:

A. geniohyoid B. styloglossus C. palatoglossus D. genioglossus *E. hyoglossus 291. The coronoid process belongs to which bone in the head?A. maxillary

Page 36: 1intranet.tdmu.edu.ua/data/kafedra/internal/xirtop/zbtest... · Web viewModule 1. Operative surgery and topographical anatomy of the head and neck, regions and organs of the thoracic

B. mandible *C. sphenoid D. occipital E. temporal 292. The muscles of mastication, their nerves and their vessels are located primarily in

which part of the head?A. pterygopalatine fossa B. jugular fossa C. incisive fossa D. infratemporal fossa *E. temporal fossa 293. During a sinus attack, painful sensation from the ethmoid cells is carried in which

nerve?A. meningeal B. greater petrosal C. pterygoid D. nasociliary *E. frontal 294. The facial artery gives rise to branches that supply each of the regions listed below

EXCEPT for the:A. medial angle of the orbit B. lateral nose C. region of the eyebrow *D. upper lip E. lower lip 295. All of the following structures are located within the walls or cavity of the cavernous

sinus EXCEPT for the:A. oculomotor nerve B. internal carotid artery C. ophthalmic division of the trigeminal nerve D. mandibular division of the trigeminal nerve *E. abducens nerve296. The lateral wall of the ethmoid sinus is also part of the medial wall of the:A. nasal cavity B. orbit *C. anterior cranial fossa D. oropharynx E. nasopharynx 297. The facial nerve:A. exits the cranium through the foramen ovale B. provides the primary parasympathetic supply to the parotid gland C. supplies taste fibers to the posterior 1/3 of the tongue D. supplies motor fibers to the medial pterygoid muscle E. supplies motor fibers to the stapedius *298. The tympanic plexus of nerves is found on the:A. floor of the middle ear cavity B. pyramid of the middle ear C. medial surface of the tympanic membrane D. promontory of the middle ear cavity *E. mucous membrane lining the vestibule299. Loss of lacrimation (dry eye) can be due to an injury to which nerve?A. nasociliary B. greater petrosal *C. supraorbital

Page 37: 1intranet.tdmu.edu.ua/data/kafedra/internal/xirtop/zbtest... · Web viewModule 1. Operative surgery and topographical anatomy of the head and neck, regions and organs of the thoracic

D. anterior ethmoid E. lesser petrosal 300. Upon examining a sick child, you notice pus draining from the middle meatus of the

nose. You might suspect and look for further evidence of an infection originating from the:A. nasolacrimal duct B. sphenoid sinus C. maxillary sinus *D. mastoid sinus E. posterior ethmoidal air cells 301. Destruction of which of the following would result in loss of pain from the anterior

2/3 of the tongue?A. pterygopalatine ganglion B. otic ganglion C. trigeminal ganglion *D. geniculate ganglion E. inferior ganglion of the glossopharyngeal302. An upper eyelid that droops (ptosis) - symptom that occur after a particular lesion of

which of the following nerves:A. trochlear nerve B. oculomotor nerve *C. facial nerve D. hypoglossal nerve E. none of the above303. Loss of tears over the eyeball (dry eye) - symptom that occur after a particular lesion

of which of the following nerves:A. trochlear nerve B. oculomotor nerve C. facial nerve *D. hypoglossal nerve E. none of the above304. A pupil that is small in diameter with no light reflex - symptom that occur after a

particular lesion of which of the following nerves:A. trochlear nerve B. oculomotor nerve C. facial nerve D. hypoglossal nerve E. none of the above *305. On protrusion, the tongue deviates to one side - symptom that occur after a particular

lesion of which of the following nerves:A. trochlear nerve B. oculomotor nerve C. facial nerve D. hypoglossal nerve *E. none of the above306. Pulsations felt just above the zygomatic arch and in front of the ear are from which

vessel?A. facial B. internal jugular vein C. superficial temporal artery *D. retromandibular vein E. maxillary artery 307. Which of the following accompanies the optic nerve through the optic canal?A. cranial nerves III, IV and VI B. ophthalmic nerve

Page 38: 1intranet.tdmu.edu.ua/data/kafedra/internal/xirtop/zbtest... · Web viewModule 1. Operative surgery and topographical anatomy of the head and neck, regions and organs of the thoracic

C. meninges and opthalmic artery *D. ophthalmic veins E. none of the above 308. The sensory supply to the skin over the lower eyelid comes from the:A. mental nerve B. maxillary division of the trigeminal *C. auriculotemporal nerve D. buccal branch of the trigeminal nerve E. ophthalmic division of the trigeminal309. The pterygomandibular raphe serves as a point of attachment for two important

muscles. They are:A. superior constrictor and buccinator *B. masseter and inferior constrictor C. medial pterygoid and lateral pterygoid D. lateral pterygoid and superior constrictor E. lateral pterygoid and buccinator310. The opening from the pterygopalatine fossa to the nasal cavity is the:A. pterygoid canal B. pterygomaxillary fissure C. foramen rotundum D. sphenopalatine foramen *E. pharyngeal canal 311. Impaired function of which of the following muscles would result in difficulty in

protruding the lower jaw?A. digastric B. lateral pterygoid *C. medial pterygoid D. masseter E. temporalis 312. Which nerve innervates the muscle for tight closure of the eyelids?A. facial *B. oculomotor C. sympathetic D. trigeminal E. vagus 313. An acoustic neuroma is a tumor involving the vestibulocochlear nerve as it exits the

cranial cavity. Because this tumor compresses surrounding structures or invades nearby tissues, in addition to hearing loss and equilibrium problems, a patient would most likely also demonstrate ipsilateral (same sided):

A. loss of general sensation to the face B. facial paralysis *C. paralysis of the lateral rectus muscle D. tongue paralysis E. ptosis 314. A gag reflex overcomes your patient as you lightly swab an area of the oropharynx.

What nerve carries the sensory fibers of this reflex?A. mandibular B. maxillary C. facial D. glossopharyngeal *E. vagus 315. Each of the following is characteristic of the maxillary sinus EXCEPT that:A. it is lined with mucous membrane. B. it drains through an opening under the superior concha (turbinate) *

Page 39: 1intranet.tdmu.edu.ua/data/kafedra/internal/xirtop/zbtest... · Web viewModule 1. Operative surgery and topographical anatomy of the head and neck, regions and organs of the thoracic

C. it may become infected from an abscessed tooth. D. its roof is the floor of the orbit. E. its lining is innervated by the trigeminal316. All of the following paranasal sinuses drain into the middle meatus, EXCEPT the:A. frontal B. maxillary C. sphenoid *D. anterior ethmoid E. middle ethmoid 317. A nosebleed (epistaxis) frequently occurs because of picking of the nose with the

finger at the anterior inferior portion of the nasal septum. Branches of which arteries may be involved?

A. maxillary and facial *B. facial and ophthalmicC. ophthalmic and maxillaryD. facial and maxillaryE. none of these318. A lesion of the facial nerve just after it exits from stylomastoid foramen would result

in:A. an ipsilateral (same side) loss of taste to the anterior tongue B. a decrease in saliva production in the floor of the mouth C. a sensory loss to the tongue D. an ipsilateral paralysis of facial muscles *E. a contralateral (opposite side)319. The vertical depression in the midline of the upper lip is called the:A. uvula B. frenulum C. philtrum *D. torus E. mentum 320. Depression of the jaw (opening the mouth) is accomplished by contraction of which

of the following muscles and gravity?A. lateral pterygoids B. geniohyoids C. mylohyoid D. digastrics E. all of the above *321. Arrange the following from lateral to medial: 1) infratemporal fossa; 2) nasal cavity;

3) pterygo-maxillary fissure; 4) pterygo-palatine fossa; 5) sphenopalatine foramen:A. 1 4 5 2 3 B. 4 1 5 3 2 C. 1 3 4 5 2 *D. 5 1 3 2 4 E. 1 3 5 4 2 322. A neurologist checking cranial nerve function olaces his fingers over the eyelids of

his patient and asks the patient to try to keep her eyes closed while he attempts to open them. This is obviously one test of the functional integrity of the:

A. Facial nerve *B. Motor root of the trigeminal nerveC. Abducens nerveD. Trochlear nerveE. None of these323. An injury to the mandibular nerve, just before it passes through the foramen ovale,

may give rise to one of the following effects:

Page 40: 1intranet.tdmu.edu.ua/data/kafedra/internal/xirtop/zbtest... · Web viewModule 1. Operative surgery and topographical anatomy of the head and neck, regions and organs of the thoracic

A. Loss of function of levator veli palatiniB. Loss of secretion from the submandibular gland on that sideC. Weakness of depression of the mandible against resistance *D. Loss of taste from the anterior 2/3 of the tongue on that sideE. None of the above effects could result from the injury described324. A distinguished pianist is unable to meet his concert commitments because of

difficulty of vision and pain and vesicles over the right half of his forehead and nose. You suspect herpes zoster (shingles) of the right:

A. Facial nerveB. Optic nerveC. Ophthalmic division of the trigeminal nerve *D. Maxillary division of the trigeminal nerveE. Mandibular division of the trigeminal nerve325. A patient complains of difficulty in chewing following head injury. On examination,

you find his jaw moves asymmetrically, and he has blunting of sensation over the front of the tongue and lower lip on one side. Tongue movements are normal. You suspect a fractured base of skull with a fracture line running through the:

A. Stylomastoid foramenB. Anterior condylar canalC. Foramen lacerumD. Foramen ovale *E. None of these transmits the nerve involved326. A surgeon injected local anesthetic into the mandibular division of the trigeminal

nerve as it emerges from the foramen ovale. The following sensory and/or taste loss resulted:

A. Loss of taste and sensation in the anterior 2/3 of the same side of the tongueB. Loss of taste and sensation from the posterior 1/3 of the same side of the tongueC. Loss of sensation only from the anterior 2/3 of the same side of the tongue *D. Loss of taste only from the anterior 2/3 of the same side of the tongueE. None of these is true327. A dentist operating on a left lower third molar tooth injures a nerve which lies close

to the mandible at that point. His patient subsequently sued him on the grounds that he:A. Suffered repeated cigarette burns of his lower lipB. Constantly bit his tongue *C. Could not distinguish sweet from dry sherryD. Could not protrude his tongue in the midlineE. Had a constantly dry mouth328. A patient has a sunken appearance along the ramus of the mandible and above the

zygomatic arch due to muscle atrophy. Which of the following lesions would contribute mot to cause this defect?

A. Lesion of the ophthalmic division of the trigeminal nerveB. Lession of the great auricular and the lesser occipital nervesC. Lesion of the mandibular division of the trigeminal nerve *D. Lesion of the maxillary division of the trigeminal nerveE. Lesion of the facial nerve329. In an infant, facial nerve palsy can be caused by the improper use of obstetric

forceps. The newborn anatomy which causes the nerve to be vulnerable is the:A. Small size of the parotid glandB. Absence of the mastoid process *C. Small size of the coronoid processD. Relative flatness of the parietal emminenceE. Inverted angle of the mandible330. The facial nerve carries fibers subserving all but one of the following functions:A. Secretomotor to the lacrimal gland

Page 41: 1intranet.tdmu.edu.ua/data/kafedra/internal/xirtop/zbtest... · Web viewModule 1. Operative surgery and topographical anatomy of the head and neck, regions and organs of the thoracic

B. Secretomotor to the submandibular glandC. Motor to the stapedius muscleD. Some sensation to the soft palateE. Taste to the posterior 1/3 of the tongue *331. Which of the following statements does not apply to the mandibular division of the

trigeminal nerve?A. It carries sensory innervation to the tongueB. It furnishes motor innervation to the tongue *C. It can be anesthetized intraorally to block the lower jawD. It helps to supply the duraE. It supplies the muscles of mastication332. A 75 year old patient is experiencing extreme pain in the anterior portion of the

alveolar ridge. This pain is greatest during mastications, when pressure is applied to the area. The gums appear normal. The patient states that he has worn dentures since the age of 25. which of the following seems most likely?

A. The pain is due to pressure on the buccal branch of the trigeminal nerveB. The pain is due to pressure on the inferior alveolar nerve at the mandibular foramenC. The pain is due to pressure on the mental nerve at the mental foramen *D. The pain is reffered from the nasopalatine nerveE. The pain is psychosomatic333. A surgeon injected local anesthetic into the mandibular division of the trigeminal

nerve as it emerges from the foramen ovale. All but one of the following muscles will be paralyzed:

A. The masseterB. The anterior fibers of the temporalis C. The buccinator *D. The tensor veli palatiniE. The tensor tympani334. A housewife complains that during chewing, food accumulates in the vestibule of her

mouth. You suspect a lesion of the:A. Mandibular division of the trigeminal nerveB. Facial nerve *C. Glossopharyngeal nerveD. Spinal accessory nerveE. Hypoglossal nerve335. The facial nerve supplies all but one of the following muscles:A. The buccinatorB. The risoriusC. The posterior belly of the digastricD. The stylopharyngeus *E. The stylohyoid336. A neurologist checking the function of the cranial nerves places his fingers just

below the zygomatic arches of his patient and asks the patient to bite down. If he feels muscles contract he knows that which nerve is functioning normally?

A. The facialB. The hypoglossalC. The accessoryD. The motor root of the trigeminal *E. The vagus337. The buccal branch of the mandibular division of the trigeminal nerve passes:A. Between the superior and inferior heads of the lateral pterygoidB. Anterior to the ramus of the mandibleC. Through the buccal fat padD. Lateral to the buccinator muscle

Page 42: 1intranet.tdmu.edu.ua/data/kafedra/internal/xirtop/zbtest... · Web viewModule 1. Operative surgery and topographical anatomy of the head and neck, regions and organs of the thoracic

E. All of these *338. The major sensory branch of the anterior division of the mandibular nerve is the:A. Deep posterior temporal nerveB. Inferior alveolar nerveC. Buccal nerve *D. Auriculotempora nerveE. None of these339. On resecting the lower part of the ramus of the mandible in an approach to the

infratemporal fossa a surgeon sees two nerves on the exposed surface of the medial pterygoid muscle. They are the:

A. Linqual and hypoglossal nervesB. Linqual and inferior alveolar nerves *C. Inferior alveolar and hypoglossal nervesD. Hypoglossal and glossopharyngeal nervesE. There are no nerves in this location, he is mistaking fascia for nerves340. The nerves supplying the muscles which protrude the tongue is the:A. Glossopharyngeal nerveB. Trigeminal nerveC. Facial nerveD. Vagus nerveE. Hypoglossal nerve *341. Which of the following nerves is not a branch of the ophthalmic division of the

trigeminal nerve?A. The lacrimal nerveB. The frontal nerveC. The nasociliary nerveD. The middle meningeal nerve *E. The tentorial nerve342. Injures to, or diseases of, the facial nerve may result in paralysis of muscles supplied

by the nerve. Which of the following muscles is not supplied by the facial nerve?A. The buccinatorB. The occipitofrontalisC. The stapediusD. The tensor tympani *E. All the above muscles are supplied by the facial nerve343. After an autumn night of poker, a medical student noticed that one side of a fellow

student’s face was smooth, his eye would not close, his mouth drooped, and saliva drooled out of the corner of his mouth. Further examination revealed no change in his ability to taste in the anterior 2/3 of the tongue. You suspect a lesion (unspecified damage) of the:

A. Mandibular division of the trigeminal nerveB. Glossopharyngeal nerve in the jugular foramenC. Facial nerve in the internal auditory meatusD. Facial nerve in the middle earE. Facial nerve after its exit from the skull *344. Loss of general and special sense from the posterior one-third of the tongue

following tonsillectomy is due to damage or compression to terminal portions of the:A. Vagus nerveB. Glossopharyngeal nerve *C. Chorda tympaniD. Lesser palatine nerveE. Lingual nerve345. An anatomy instructor is unable to raise one of his eyebrows in surprise when you

answer his question correctly. You diagnose injure to his:A. Frontal nerve

Page 43: 1intranet.tdmu.edu.ua/data/kafedra/internal/xirtop/zbtest... · Web viewModule 1. Operative surgery and topographical anatomy of the head and neck, regions and organs of the thoracic

B. Anterior deep temporal nerveC. Zygomaticotemporal nerveD. Zygomaticofacial nerveE. Temporal branch of the facial nerve *346. Cutaneous branches of the ophthalmic nerve (V1) include the:A. Supraorbital nerveB. Supratrochlear nerveC. Lacrimal nerveD. All of these *E. None of these347. When the inferior alveolar nerve is blocked for extraction of a tooth, one can expect

anesthesia to also involve the:A. Floor of the mouthB. Upper incisorsC. CheekD. Lower lip *E. Upper molars348. Which of the following does not open into the pterygopalatine fossa?A. Sphenopalatine foramenB. Pterygomaxillary fissureC. Foramen rotundumD. Pterygopalatine canalE. Superior orbital fissure *349. If you were to inject a local anesthetic into and around the foramen ovale, which of

the following functions would not be impaired?A. Loss of sensation to teeth of lower jaw on same sideB. Inability to chew effectively on same sideC. Interference with parotid gland functionD. Loss of taste to the anterior 2/3 of the tongue *E. All of the above would be impaired350. Tic dolourex – is a condition of unknown cause with severe pain along the maxillary

and mandibular divisions of the trigeminal nerve. This condition is sometimes relieved by alcohol injections into the trigeminal ganglion. Which of the following foramina would be used to reach the ganglion?

A. Foramen spinosumB. Stylomastoid foramenC. Foramen caecumD. Foramen ovale *E. Jugular foramen351. What is the volume of the frontal sinus at birth?A. 1 ccB. 5 ccC. 10 ccD. 15 ccE. None of these *352. The orbit is closely related to which of the following paranasal sinuses?A. The frontalB. The maxillaryC. The sphenoidD. The ethmoidE. All of these are correct *353. A patient consults you because he can feel a horizontal cord-like structure just below

his cheek bone when he clenches his teeth. You reassure him it is the:A. Mandibular nerve

Page 44: 1intranet.tdmu.edu.ua/data/kafedra/internal/xirtop/zbtest... · Web viewModule 1. Operative surgery and topographical anatomy of the head and neck, regions and organs of the thoracic

B. Facial nerveC. Long buccal nerveD. Transverse facial vein and arteryE. Parotid duct *354. Both benign and malignant tumors of the parotid gland are common. The surgeon

commonly suspects malignancy when he sees signs of possible invasion of structures in or in contact with the parotid gland. You might suspect a tumor was malignant if the patient has apparently lost the functional integrity of any but one of the following:

A. Superficial temporal arteryB. External carotid arteryC. Auriculotemporal nerveD. Facial nerveE. Any of these could be involved – they are all closely related to the gland *355. To reduce bleeding during excision of a sublingual salivary gland, a surgeon may

well elect to ligate the:A. Superior thyroid arteryB. Lingual arteryC. Facial artery *D. Maxillary arteryE. None of these give a substantial contribution to the blood supply of the sublingual gland356. A medical student trying to study for an exam because so exasperated with his

roommate who would not stop talking that he jammed a towel into his mouth. The recipient of the act found he could not close his mouth after removing the towel. He had probably suffered:

A. A rupture of his temporalis tendonB. A fracture of the neck of his mandibleC. An anterior dislocation of his temporomandibular joint *D. An injure to his mandibular nerveE. None of the above357. All of the following contribute significantly to elevating the mandible and closing the

mouth except the:A. Masseter muscleB. Medial pterygoid muscleC. Lateral pterygoid muscle *D. Anterior fibers of the temporalis muscleE. Posterior fibers of the temporalis muscle358. Muscles involved in protruding the mandible include which of the following?A. The masseter and posterior fibers of the temporalisB. The medial and lateral pterygoid muscles *C. The medial pterygoid and anterior digastric musclesD. The temporalis and genioglossus musclesE. The platysma and lateral pterygoid muscles359. A circus performer who specializes in hanging from a high wire by biting a

suspended mouth gag feels a sudden pain in her left head and jaw and falls into the safety net. On examination you find she is unable to retract the left side of her jaw. You suspect injure to the left:

A. Masseter muscleB. Medial pterygoid muscleC. Lateral pterygoid muscleD. Anterior part of the temporalis muscleE. Posterior part of the temporalis muscle *360. A frightened young lady comes to your office with a complaint of pain in her jaw

which will not close completely. After a few direct questions the site of the jaw problem is

Page 45: 1intranet.tdmu.edu.ua/data/kafedra/internal/xirtop/zbtest... · Web viewModule 1. Operative surgery and topographical anatomy of the head and neck, regions and organs of the thoracic

obviously the temporomandibular joint. What must you remember about the joint as you discuss the problem with the patient?

A. The temporomandibular joint permits both hinge and gliding movementB. Anterior dislocation occurs relatively easilyC. Posterior dislocation occurs rarelyD. All of the above *E. None of the above361. Severe bleeding following tonsillectomy used to be fairly common when this

operation was performed on almost everybody and by almost anybody. The source of the bleeding could reasonably have been a tonsillar branch of the:

A. Lingual arteryB. Descending palatine arteryC. Ascending pharyngeal arteryD. Facial arteryE. All of thee arteries *362. The sensory nerve supply to the tonsillar area is generally through to be via the:A. Glossopharyngeal nerve *B. Facial nerveC. Vagus nerveD. Mandibular nerveE. Trigeminal nerve363. The main venous drainage from the tonsil is via the:A. Retromandibular nerveB. Facial vein *C. Lingual veinD. Ascending pharyngeal veinE. Pterygoid plexus364. During a visit to the dentist, a dental burr flies off and mysteriously disappears

through the lateral wall of the palatine tonsillar fossa. It must have penetrated the:A. Stylopharyngeal muscleB. Middle constrictor muscleC. Salpingopharyngeus muscleD. Superior constrictor muscle *E. Tensor veli palatini muscle365. The lymphatic drainage from the left margin of the anterior tip of the tongue is to all

but which one of the following?A. The left juguloomohyoid nodeB. The submandibular nodesC. The submental nodesD. The jugulodigatric nodesE. All of the above receive drainage from the site *366. Which of the following statements about the nerve supply of the tongue is false?A. The vallate papillae are innervated by the glossopharyngeal (IX) nerveB. The filiform papillae are innervated by the hypoglossal (XII) nerve *C. The tip of the tongue is innervated by the trigeminal (V) nerveD. The lingual tonsil is innervated by the glossopharyngeal (IX) nerveE. All of the above are false367. The tongue is protruded from the mouth by the:A. Hyoglossus muscleB. Styloglossus muscleC. Genioglossus muscle *D. Stylohyoid muscleE. Buccinator muscle

Page 46: 1intranet.tdmu.edu.ua/data/kafedra/internal/xirtop/zbtest... · Web viewModule 1. Operative surgery and topographical anatomy of the head and neck, regions and organs of the thoracic

368. Kiesselbach’s area, on the anterior part of the nasal septum, has been blamed for a high percentage of nose “bleeds”. It receives tributaries from the:

A. Sphenopalatine and labial arteries *B. Anterior ethmoidal arteryC. Sphenopalatine and anterior ethmoidal arteriesD. Greater palatine and labial arteriesE. None of these369. The most posterior portion of the nasal septum is formed by the:A. Vomeronasal cartilageB. Vomer *C. Septal cartilageD. Nasal spine of the frontal boneE. Inferior conchae370. A rhinologist decides to anesthetize the right nasal cavity by pushing a pad of cotton

soaked in cocaine into the back of the middle meatus of the nose. He should not be too much surprised if the patient complains of:

A. Dryness of his mouthB. Inability to smileC. Loss of sensation in the lower molar teethD. Dryness of his right eye *E. Numbness over his forehead371. Which of these nerves does not supply the skin of the external nose?A. The external nasal nerveB. The infraorbital nerveC. The infratrochlear nerveD. The supratrochlear nerve *E. All of the above supply it372. A young man insults a fellow “drinker” in a bar and gets punched in the nose. The

bleeding is most likely from which of the following areas?A. The lateral wall of the inferior meatusB. The posterior third of the nasal septumC. The anterior lower portion of the nasal septum *D. The middle meatusE. The superior surface of the hard palate373. An otolaryngologist wishes to introduce a drain into the maxillary air sinus through

its orifice into the nose. In order to anesthetize the mucous membrane immediately around the opening, the blocks the:

A. Anterior ethmoid nerveB. Olfactory nervesC. Sphenopalatine ganglion *D. Infraorbital nerveE. Anterior superior alveolar nerve374. A patient had a purulent otitis media (infection of the tympanic cavity) but before he

reached a physician he had a sudden relief of his pain followed by a profuse purulent discharge from his ear. After recovery from this episode, he complained of a loud whistling noise in that ear whenever he blew his nose. This symptom strongly suggests that:

A. He had displacement of joint between the malleus and incusB. He had a ruptured tympanic membrane *C. He had a paralysis of the stapedius muscleD. None of theseE. All of these375. A patient had a purulent otitis media (infection of the tympanic cavity) but before he

reached a physician he had a sudden relief of his pain followed by a profuse purulent

Page 47: 1intranet.tdmu.edu.ua/data/kafedra/internal/xirtop/zbtest... · Web viewModule 1. Operative surgery and topographical anatomy of the head and neck, regions and organs of the thoracic

discharge from his ear. After recovery from this episode, he complained of a loud whistling noise in that ear whenever he blew his nose. This symptom strongly suggests that:

A. He had displacement of joint between the malleus and incusB. The auditory tube was patent (open) *C. He had a paralysis of the stapedius muscleD. None of theseE. All of these376. Following a motor car accident which resulted in a severe blow in the region of the

left eye and left side of the forehead, a patient complains of double vision especially when he attempts to look up. Ptosis is not evident. You suspect injury to the:

A. Upper division of the oculomotor nerveB. Lower division of the oculomotor nerveC. Trochlear nerveD. Abducent nerveE. None of the above is likely to give rise to such symptoms. You look for another cause. *377. Match the auricular anterior muscle with the appropriate motor branch of VII from

the list below:A. Temporal and/or zygomatic *B. BuccalC. Marginal mandibularD. CervicalE. None of the above378. Match the levator labii superioris muscle with the appropriate motor branch of VII

from the list below:A. Temporal and/or zygomaticB. Buccal *C. Marginal mandibularD. CervicalE. None of the above379. Match the orbicularis occuli muscle with the appropriate motor branch of VII from

the list below:A. Temporal and/or zygomatic *B. BuccalC. Marginal mandibularD. CervicalE. None of the above380. Match the platysma with the appropriate motor branch of VII from the list below:A. Temporal and/or zygomaticB. BuccalC. Marginal mandibularD. Cervical *E. None of the above381. Match the external acoustic meatus with its relationship to the parotid gland from the

list below:A. AnteromedialB. PosteromedialC. LateralD. Superior *E. Not in contact with the parotid gland382. Match the external carotid artery with its relationship to the parotid gland from the

list below:A. AnteromedialB. Posteromedial *C. Lateral

Page 48: 1intranet.tdmu.edu.ua/data/kafedra/internal/xirtop/zbtest... · Web viewModule 1. Operative surgery and topographical anatomy of the head and neck, regions and organs of the thoracic

D. SuperiorE. Not in contact with the parotid gland383. Match the internal carotid artery with its relationship to the parotid gland from the

list below:A. AnteromedialB. PosteromedialC. LateralD. SuperiorE. Not in contact with the parotid gland *384. Match the sternocleidomastoid muscle with its relationship to the parotid gland from

the list below:A. AnteromedialB. Posteromedial *C. LateralD. SuperiorE. Not in contact with the parotid gland385. Which of the following arteries is source of ophthalmic artery:

A. Common carotid arteryB. Internal carotid artery *C. External carotid arteryD. Ethmoidal arteryE. Medial cerebral artery.

386. Which of the following arteries is source of supratrochlear artery:A. Ophthalmic artery *B. Anterior cerebral arteryC. Anterior meningeal arteryD. Medial cerebral arteryE. Ethmoidal artery

387. Which of the following arteries is source of zygomaticoorbital artery:A. Ophthalmic arteryB. Temporal superficial arteryC. Facial arteryD. Maxillary artery *E. Internal carotid artery

388. Which of the following structures can be damaged during antrotomy in case of anterior border breach:

A. Facial nerve *B. Sigmoid sinusC. Cavernous sinusD. Maxillary nerveE. Mandibular nerve.

389. Which of the following structures can be damaged during antrotomy in case of posterior border breach:

A. Facial nerveB. Sigmoid sinus *C. Cavernous sinusD. Maxillary nerveE. Mandibular nerve.

390. Inflammatory process in case of purulent parotitis spreads to:A. Internal earB. Middle cranial fossaC. External acoustic meatus *D. Subcutaneous tissue of temporal regionE. Temporopterygoid space.

Page 49: 1intranet.tdmu.edu.ua/data/kafedra/internal/xirtop/zbtest... · Web viewModule 1. Operative surgery and topographical anatomy of the head and neck, regions and organs of the thoracic

391. Which of the following structures pass through the parotid gland:A. Temporal nerveB. Vagus nerveC. Facial nerve *D. Maxillary nerveE. Mandibular nerve.

392. Which of the following structures pass through the parotid gland:A. Vagus nerveB. Facial arteryC. Facial veinD. External carotid artery *E. Internal carotid artery.

393. Which of the following structures pass through the parotid gland:A. Facial veinB. External jugular veinC. Retromandibular vein *D. Internal jugular veinE. Temporal superficial vein.

394. Inflammatory process in case of purulent parotitis spreads to:A. Middle cranial fossaB. Temporal interaponeurotical tissueC. Parapharyngeal space *D. Temporopterygoid spaceE. Floor of oral cavity.

395. Complication in case of purulent parotitis:A. Damage of proper parotid fasciaB. Pus spread to the temporopterygoid spaceC. Paresis of muscles of facial expression on damaged side *D. Neuritis of vagus nerveE. Dysphagy.

396. Complication in case of purulent parotitis:A. Neuritis of vagus nerveB. Neuritis of mandibular nerveC. Bleeding of erosive vessels *D. Damage of proper parotid fasciaE. Neuritis of maxillary nerve.

397. Interfascial cellular space of temporopterygoid space locates in which of the following head regions:

A. Temporal regionB. Deep facial region *C. Buccal regionD. Occipital regionE. Oral cavity.

398. Interfascial cellular space of interpterygoid space locates in which of the following head regions:

A. Temporal regionB. Deep facial region *C. Buccal regionD. Occipital regionE. Oral cavity.

399. Parotid duct opens in which of the following structures:A. External acoustic meatusB. Vestibule of mouth *C. Nasal cavity

Page 50: 1intranet.tdmu.edu.ua/data/kafedra/internal/xirtop/zbtest... · Web viewModule 1. Operative surgery and topographical anatomy of the head and neck, regions and organs of the thoracic

D. Floor of oral cavityE. Maxillary sinus.

400. Sphenoidal sinus opens in which of the following structures:A. Superior nasal meatus *B. Middle nasal meatusC. Inferior nasal meatusD. Oral cavityE. Maxillary sinus.

401. Frontal sinus opens in which of the following structures:A. Superior nasal meatusB. Middle nasal meatus *C. Inferior nasal meatusD. Oral cavityE. Maxillary sinus.

402. Maxillary sinus opens in which of the following structures:A. Superior nasal meatusB. Middle nasal meatus *C. Inferior nasal meatusD. Oral cavityE. Sphenoidal sinus.

403. Which one of the following muscles raises the soft palate during swallowing?A. Levator veli palatine *B. PalatoglossusC. PalatopharyngeusD. All of theseE. Superior constrictor

404. The lymphatic vessels of the tongue drain primarily into the:,A. Palatine nodesB. Deep cervical nodes *C. Gingival nodesD. Facial nodesE. Parotid nodes

405. The internal carotid arteries have significant anastomotic connections with tributaries of all the following arteries except the:

A. FacialB. Internal maxillaryC. Lingual *D. Superficial temporalE. Vertebral

406. Each of the following structures empties or opens into the middle nasal meatus except the:

A. Middle ethmoidal air cellsB. Maxillary sinusC. Sphenoid sinus *D. Anterior ethmoidal air cellsE. Frontal sinus

407. The parotid duct penetrates which of the following muscles?A. MasseterB. Medial pterygoidC. Buccinator *D. Superior pharyngeal constrictorE. Levator anguli oris

408. The primary action of the muscle that originates mostly from the medial aspect of the lateral pterygoid plate is to:

Page 51: 1intranet.tdmu.edu.ua/data/kafedra/internal/xirtop/zbtest... · Web viewModule 1. Operative surgery and topographical anatomy of the head and neck, regions and organs of the thoracic

A. Retrude (retract) the mandibleB. Elevate the soft palateC. Protrude (protract) the mandibleD. Tense the soft palateE. Elevate the mandible *

409. After entering the submandibular triangle, the lingual artery passes immediately deep to which of the following muscles?

A. GenioglossusB. Anterior belly of the digastricC. MylohyoidD. Hyoglossus *E. Styloglossus

410. Pulsations felt at the lower border of the mandible just anterior to the masseter muscle are in the:

A. Superficial temporal arteryB. Lingual arteryC. Maxillary arteryD. Transverse facial arteryE. Facial artery *

411. Which one of the following bones enters into the formation of the roof of the orbit?A. PalatineB. MaxillaC. Frontal *D. ZygomaticE. Lacrimal

412. The largest structure located on the medial wall of the tympanic cavity is the:A. Fenestra vestibuliB. Facial canalC. UmboD. Promontory *E. Fenestra cochlea

413. The artery that accompanies the inferior alveolar nerve is a branch of the:A. Superficial temporalB. Maxillary *C. Posterior auricularD. FacialE. Lingual

414. Sympathetic preganglionic nerve fibers for the arteries supplying the submandibular salivary gland synapse in the:

A. Ciliary ganglionB. Superior cervical chain ganglion *C. Submandibular ganglionD. Inferior cervical chain ganglionE. Pterygopalatine ganglion

415. The muscle that rotates the muscular process of the arytenoid cartilage anteriorly, thus adducting the vocal folds, is the:

A. Transverse arytenoidsB. Posterior cricoarytenoidC. Lateral cricoarytenoid *D. ThyroarytenoidE. Aryepiglottic

416. The middle meatus receives drainage from each of the following except the:A. Maxillary sinusB. Frontal sinus

Page 52: 1intranet.tdmu.edu.ua/data/kafedra/internal/xirtop/zbtest... · Web viewModule 1. Operative surgery and topographical anatomy of the head and neck, regions and organs of the thoracic

C. Nasolacrimal duct *D. Anterior ethmoid air cellsE. Middle ethmoid air cells

417. The submandibular gland receives its primary blood supply from branches of which of the following arteries?

A. Facial *B. LingualC. Superior thyroidD. Posterior auricularE. Occipital

418. The posterior superior alveolar nerve supplies sensory fibers from the:A. Hard palateB. Soft palateC. Upper molars *D. Nasal mucosaE. Buccal mucosa

419. The bones that form the floor of the hard palate are the:A. Sphenoid and zygomaticB. Maxillary and frontalC. Ethmoid and frontalD. Palatine and maxillary *E. Palatine and ethmoid

420. The membranous labyrinth of the internal ear consists of each of the following except the:

A. Vestibule *B. SacculusC. Semicircular ductsD. UtriculusE. Cochlear duct

421. Each of the following is located between the mylohyoid and hyoglossus muscles except the:

A. Submandibuiar ductB. Lingual nerveC. Hypoglossal nerveD. Submandibuiar ganglionE. Lingual artery *

422. Which of the following muscles is involved in retraction of the mandible?A. Lateral pterygoidB. MasseterC. MylohyoidD. Medial pterygoidE. Posterior fibers of temporalis *

423. After exiting the stylomastoid foramen, the facial nerve is correctly described by each of the following except:

A. It innervates the posterior belly of the digastric and stylohyoid musclesB. It courses superficial to the retromandibular veinC. It innervates the muscle that provides tone to the cheekD. It innervates the submandibular and sublingual glands *E. It innervates the muscles that elevate the upper lip

424. Which of the following ligaments is formed from a thickening of the deep parotid fascia?

A. TemporomandibularB. StylohyoidC. Stylomandibular *

Page 53: 1intranet.tdmu.edu.ua/data/kafedra/internal/xirtop/zbtest... · Web viewModule 1. Operative surgery and topographical anatomy of the head and neck, regions and organs of the thoracic

D. SphenomandibularE. Nuchal

425. The levator anguli oris muscle is innervated by which of the following nerves?A. AuriculotemporalB. Facial *C. Ansa cervicalisD. Inferior alveolarE. Inferior palpebral nerve

426. The angular artery is the terminal part of which of the following arteries?A. Superficial temporalB. Posterior auricularC. MaxillaryD. Facial *E. Occipital

427. The infraorbital artery is one of the terminal branches of which of the following arteries?

A. FacialB. Transverse facialC. Superficial temporalD. LingualE. Maxillary *

428. The mental artery is a terminal branch of which of the following arteries?A. Superficial temporalB. Transverse facialC. Inferior alveolar *D. FacialE. Lingual

429. The retromandibular vein is formed by the confluence of the superficial temporal and which of the following veins?

A. Transverse facialB. Maxillary vein *C. FacialD. LingualE. Submental

430. The parotid duct penetrates which of the following muscles?A. MasseterB. Medial pterygoidC. Buccinator *D. Superior pharyngeal constructorE. Levator anguli oris

431. The deep portion of the parotid fascia forms which of the following?A. Stylomandibular ligament *B. Spenomandibular ligamentC. Pterygomandibular rapheD. Carotid sheathE. Buccopharyngeal fascia

432. The supraorbital artery is a branch of which of the following arteries?A. Superficial temporalB. Transverse facialC. MaxillaryD. FacialE. Ophthalmic *

433. Which of the following nerves is a cutaneous branch of the maxillary division of the trigeminal nerve?

Page 54: 1intranet.tdmu.edu.ua/data/kafedra/internal/xirtop/zbtest... · Web viewModule 1. Operative surgery and topographical anatomy of the head and neck, regions and organs of the thoracic

A. LacrimalB. InfratrochlearC. AuriculotemporalD. BuccalE. Superior labial *

434. The medial pterygoid muscle assists which of the following muscles in protrusion of the mandible?

A. MylohyoidB. Lateral pterygoid *C. GeniohyoidD. TemporalisE. Sphenomeniscus

435. A lesion of the lingual nerve immediately after it receives the chorda tympani nerve could result in each of the following except:

A. Loss of sublingual gland secretionB. Sensory loss from lower teeth *C. Loss of taste from anterior two-thirds of the tongueD. Loss of submandibular gland secretionE. Sensory loss from mucosa on floor of mouth

436. Which of the following statements correctly applies to the maxillitis:A. Inflammation of the frontal sinusB. Inflammation of the maxillary sinus *C. Inflammation of the ethmoid sinusD. Inflammation of the pharynxE. Inflammation of the nasal cavity

437. The regional lymph nodes which receive lymph directly from the lateral third of the lower lip are the:

A. SubmentalB. Superior deep cervicalC. Superficial cervicalD. Submandibular *E. Pretracheal. 438. Which of the following statements is false?A. Lymphatic drainage from the forehead is to the superficial parotid lymph nodesB. Lymphatic drainage from the lateral eyebrows is to the superficial parotid lymph nodesC. Lymphatic drainage from the external nose is to the buccal and submandibular nodesD. Lymphatic drainage from the medial ear is to the superficial parotid lymph nodes *E. Lymphatic drainage from the lower lip is to the submental lymph nodes.439. The sphenopalatine artery:A. Enters the nasal cavity through the sphenopalatine foramenB. Gives rise to the ascending palatine arteryC. Gives rise to the posterior lateral nasal arteryD. Enters the nasal cavity through the sphenopalatine foramen and gives rise to the posterior

lateral nasal artery *E. Enters the nasal cavity through the sphenopalatine foramen and gives rise to the external

nasal artery.440. The facial vein:A. Empties into the internal jugular veinB. Anastomoses with the cavernous sinus by way of the angular veinC. Has no valvesD. May receive tributaries from the external nasal, submandibular and submental veinsE. All are correct. *441. Vincent Van Gogh once cut off one of his ears. As might be expected, he lost a lot of

blood which must have come from the:

Page 55: 1intranet.tdmu.edu.ua/data/kafedra/internal/xirtop/zbtest... · Web viewModule 1. Operative surgery and topographical anatomy of the head and neck, regions and organs of the thoracic

A. Maxillary arteryB. Superficial temporal arteryC. Facial arteryD. Superficial temporal and facial arteriesE. Superficial temporal and posterior auricular arteries. *442. The maxillary artery gives rise to the:A. Inferior alveolar arteryB. Lingual arteryC. Middle meningeal arteryD. All of the above *E. None of the above.443. Which of the following anatomical structures are the drainage area of the

retroauricular (mastoid) lymph nodes:A. External auditory meatusB. Occipital scalpC. Auricle and external auditory meatus *D. Middle earE. External auditory meatus and middle ear.444. The nerve of the pterygoid canal is formed by the:A. Greater and lesser petrosal nervesB. Greater and deep petrosal nerves *C. Greater, lesser and deep petrosal nervesD. Lesser and deep petrosal nervesE. Greater petrosal nerve alone.445. The secretomotor fibers to the lacrimal gland leave the central nervous system via the

________ nerve.A. OculomotorB. TrigeminalC. Facial *D. GlossopharyngealE. None of these.446. The terminal branches of the facial nerve:A. Are motor to the muscles of masticationB. Are motor to the muscles of facial expression *C. Provide the sensory innervation of the faceD. Supply taste to the whole tongueE. None of the above.447. You have reason to suspect that your patient has a lesion to the facial nerve inside the

skull. In which of the following glands would normal secretion not suggest that you might be wrong?

A. Lacrimal glandB. Nasal glandsC. Parotid gland *D. Lingual glandsE. Submandibular gland.448. Which one of the following nerves is not sensory to the scalp behind the ear?A. The great auricular nerveB. The greater occipital nerveC. The lesser occipital nerveD. The least occipital nerveE. The occipital branch of the facial nerve. *449. The chorda tympani nerve is a branch of the:A. Geniculate ganglionB. Facial nerve *

Page 56: 1intranet.tdmu.edu.ua/data/kafedra/internal/xirtop/zbtest... · Web viewModule 1. Operative surgery and topographical anatomy of the head and neck, regions and organs of the thoracic

C. Acoustico-vestibular nerveD. Glossopharyngeal nerveE. Vagus nerve.450. If you were to bite the inside of your cheek, you would feel the pain via the:A. Mandibular nerveB. Trigeminal ganglionC. Buccal nerveD. All of the above *E. None of these.451. Which of the following does not communicate directly with the infratemporal fossa?A. Pterygomaxillary fissureB. Foramen rotundum *C. Foramen ovaleD. Foramen spinosumE. None of these.452. The stapedius muscle is supplied by the:A. Trigeminal nerveB. Facial nerve *C. Glossopharyngeal nerveD. Vagus nerveE. Accessory nerve (cranial portion).453. The tensor tympani muscle is supplied by the:A. Trigeminal nerve *B. Facial nerveC. Glossopharyngeal nerveD. Vagus nerveE. Accessory nerve (cranial portion).454. Which of the following nerves does not supply the tympanic membrane?A. The vagus nerveB. The facial nerve *C. The glossopharyngeal nerveD. The auriculotemporal nerveE. All of these supply it.455. A tumor in the internal acoustic meatus may result in loss of:A. Hearing and balanceB. Facial expressionC. Lacrimal secretionD. Taste to the anterior 2/3 of tongueE. All are correct. *456. The skin of the tip of the nose is supplied by the:A. Supratrochlear nerveB. Infratrochlear nerveC. External nasal nerve *D. Long buccal nerve, superior branchE. Infraorbital nerve.457. The digastric muscle is a two-bellied muscle that attaches by an intermediate tendon

to which of the following?A. Mandibular condyleB. Thyroid cartilageC. Cricoid cartilageD. Styloid processE. Hyoid bone *

458. The omohyoid, the sternocleidomastoid, and the posterior belly of the digastric muscle form the boundary for which of the following triangles?

Page 57: 1intranet.tdmu.edu.ua/data/kafedra/internal/xirtop/zbtest... · Web viewModule 1. Operative surgery and topographical anatomy of the head and neck, regions and organs of the thoracic

A. OccipitalB. SubmandibularC. SubmentalD. Carotid *E. Omoclavicular

459. As a rule, the isthmus of the thyroid gland crosses which of the following structures?A. Hyoid boneB. Second to fourth tracheal rings *C. Cricoid cartilageD. Thyroid cartilageE. Inferior belly of the omohyoid muscle

460. The prevertebral layer of cervical fascia forms the floor for which of the following triangles?

A. SubmentalB. Posterior cervical *C. SubmandibularD. CarotidE. Muscular

461. Which of the following muscles aids in depressing the corner of the mouth downward and widens the aperture, as in expressions of sadness or fright?

A. Orbicularis orisB. Buccinator-C. MylohyoidD. MentalisE. Platysma *

462. The cervical branch of the facial nerve innervates which of the following muscles?A. SternocleidomastoidB. GeniohyoidC. SternothyroidD. Platysma *E. Masseter

463. Which of the following veins unites with the retromandibular to form the external jugular vein?

A. Posterior auricular *B. Superficial temporalC. Transverse facialD. Internal jugularE. Facial

464. Which of the following veins crosses perpendicularly the superficial surface of the sternocleidomastoid beneath the platysma muscle?

A. Internal jugularB. Anterior jugularC. Posterior jugularD. External jugular *E. Retromandibular

465. Which of the following nerves is a dorsal ramus of the second cervical nerve?A. Great auricularB. Greater occipital *C. Lesser occipitalD. Transverse cervicalE. Supraclavicular

466. Which of the following nerves is formed by contributions from the ventral rami of cervical nerves three and four?

A. Supraclavicular *

Page 58: 1intranet.tdmu.edu.ua/data/kafedra/internal/xirtop/zbtest... · Web viewModule 1. Operative surgery and topographical anatomy of the head and neck, regions and organs of the thoracic

B. Greater occipitalC. Great auricularD. Transverse cervicalE. Occipitalis tertius

467. The superficial layer of cervical fascia splits into two sheets to enclose which of the following muscles?

A. Sternothyroid B. Anterior scaleneC. Trapezius *D. MylohyoidE. Semispinals capitis

468. Which of the following ligaments is formed from a thickening of the deep parotid fascia?

A. TemporomandibularB. StylohyoidC. Stylomandibular *D. SphenomandibularE. Nuchal

469. Which of the following fascial layers gives rise to the axillary sheath?A. Superficial layer of cervical fasciaB. Prevertebral *C. Carotid sheathD. BuccopharyngealE. Pretracheal

470. The sheath of the thyroid gland is formed from which of the following fascial layers?A. Carotid sheathB. PrevertebralC. Superficial layer of the cervical fasciaD. Pretracheal *E. Alar

471. Which of the following structures is located within the cervical visceral fasciae?A. Cervical sympathetic trunkB. Pharynx *C. External jugular veinD. Common carotid arteryE. Hypoglossal nerve

472. The largest and most important interfascial interval in the neck is which of the following spaces?

A. SuprasternalB. Retropharyngeal *C. SubmandibularD. Lateral pharyngealE. Parotid

473. The sternohyoid muscle is innervated by which of the following nerves?A. HypoglossalB. Ansa cervicalis *C. Transverse cervicalD. SupraclavicularE. Vagus

474. The superior thyroid artery is usually the first branch of which of the following arteries?

A. Common carotidB. External carotid *C. Internal carotid

Page 59: 1intranet.tdmu.edu.ua/data/kafedra/internal/xirtop/zbtest... · Web viewModule 1. Operative surgery and topographical anatomy of the head and neck, regions and organs of the thoracic

D. Subclavian arteryE. Maxillary artery

475. The inferior thyroid artery is a branch of which of the following arteries?A. Dorsal scapularB. CostocervicalC. External carotidD. Thyrocervical *E. Vertebral

476. The middle thyroid vein empties into which of the following veins?A. External jugularB. Anterior jugularC. Posterior jugularD. Internal jugular *E. Vertebral

477. Which of the following structures is embedded in the anterior sheath of the carotid sheath?

A. Sympathetic trunkB. Thyrocervical trunkC. Vertebral arteryD. Prevertebral fasciaE. Superior ramus of the ansa cervicalis *

478. The common carotid artery usually bifurcates into the external and internal carotids at the level of which of the following structures?

A. Jugular notchB. Cricoid cartilageC. Upper border of the thyroid cartilage *D. Neck of the mandibleE. Sternoclavicular joint

479. Which of the following arteries passes obliquely upward deep to the posterior belly of the digastric and the stylohyoid muscles running deep to the submandibular gland?

A. LingualB. Facial *C. MaxillaryD. Superior thyroidE. Occipital

480. Which of the following arteries arises from the posterior aspect of the external carotid at the level of the upper border of the posterior belly of the digastric?

A. FacialB. OccipitalC. Lingual *D. Posterior auricularE. Ascending pharyngeal

481. Which of the following nerves innervate the cricothyroid and the inferior constrictor muscle of the pharynx?

A. Inferior cervical cardiacB. External branch of the superior laryngeal *C. Inferior laryngealD. Recurrent laryngealE. Superior cervical cardiac

482. The superior deep cervical lymph nodes occupy which of the following cervical triangles?

A. Carotid *B. OmoclavicularC. Submandibular

Page 60: 1intranet.tdmu.edu.ua/data/kafedra/internal/xirtop/zbtest... · Web viewModule 1. Operative surgery and topographical anatomy of the head and neck, regions and organs of the thoracic

D. OccipitalE. Submental

483. Which of the following ganglia is commonly located at the level of the second cervical vertebra?

A. Stellate ganglionB. Inferior cervical ganglionC. Vertebral ganglionD. Middle cervical ganglionE. Superior cervical ganglion *

484. Which of the following ganglia is commonly located at the level of the cricoid cartilage?

A. Superior ganglion of the vagusB. Inferior ganglion of the glossopharyngealC. OticD. Middle cervical *E. Submandibular

485. Which of the following ganglia is commonly located at the base of the transverse process of the seventh cervical vertebrae?

A. PterygopalatineB. SubmandibularC. Cervicothoracic *D. VertebralE. Geniculate

486. The posterior belly of the digastric muscle is innervated by which of the following nerves?

A. TrigeminalB. Facial *C. VagusD. Ansa subclaviaE. Hypoglossal

487. The subclavian vein joins the internal jugular vein to form the brachiocephalic vein at which of these structures?

A. The outer border of the first ribB. Behind the acromioclavicular jointC. Behind the coracoclavicular jointD. In front of the coracohumeral ligamentE. Behind the sternal end of the clavicle *

488. The costocervical trunk usually gives rise to which of the following arteries?A. Highest intercostal *B. Inferior thyroidC. SuprascapularD. Transverse cervicalE. Ascending cervical

489. Which of the following laryngeal cartilages has a triangular base with vocal and muscular processes?

A. CricoidB. CorniculateC. Arytenoid *D. CuneiformE. Epiglottis

490. Which of the following structures contributes to the formation of the vocal ligaments?

A. Thyrohyoid membraneB. Cricotracheal ligament

Page 61: 1intranet.tdmu.edu.ua/data/kafedra/internal/xirtop/zbtest... · Web viewModule 1. Operative surgery and topographical anatomy of the head and neck, regions and organs of the thoracic

C. Quadrangular membraneD. Conus elasticus *E. Hyoepiglottic ligament

491. Which of the following structures constitutes the vestibular ligament of the false vocal fold?

A. Quadrangular membrane *B. Median cricothyroid ligamentC. Thyrohyoid membraneD. Thyroepiglottic ligamentE. Cricotracheal ligament

492. The space between the apposed vocal folds and arytenoid cartilages is known as the:A. GlottisB. Rima glottidis *C. VestibuleD. Rima vestibuliE. Piriform recess

493. Which of the following muscles of the larynx is abductor of the vocal ligament?A. Posterior cricoarytenoid *B. Lateral cricoarytenoidC. Transverse arytenoidD. ThyroarytenoidE. Cricothyroid

494. Which of the following muscles of the larynx increases tension on the vocal folds?A. Cricothyroid *B. Lateral cricoarytenoidC. Posterior cricoarytenoidD. ThyroarytenoidE. Transverse arytenoid

495. The vocalis muscles are composed of the internal fibers of which of the following muscles?

A. Lateral cricoarytenoidB. CricothyroidC. Thyroarytenoid *D. Posterior cricoarytenoidE. Oblique arytenoid

496. The principal sensory nerve of the larynx is the:A. Recurrent laryngealB. Inferior laryngealC. Superior laryngeal *D. GlossopharyngealE. Cervical sympathetic trunk

497. Which of the muscles of the larynx is innervated by the external branch of the superior laryngeal nerve?

A. Lateral cricoarytenoidB. Posterior cricoarytenoidC. ThyroarytenoidD. Transverse arytenoidsE. Cricothyroid *

498. The superior laryngeal artery is a branch of which of the following arteries?A. LingualB. Superior thyroid *C. Costocervical trunkD. Thyrocervical trunkE. Transverse cervical

Page 62: 1intranet.tdmu.edu.ua/data/kafedra/internal/xirtop/zbtest... · Web viewModule 1. Operative surgery and topographical anatomy of the head and neck, regions and organs of the thoracic

499. The trachea begins at the level of which of the following structures?A. Hyoid boneB. Thyroid cartilageC. Fourth cervical vertebraD. Cricoid cartilage *E. Second cervical vertebra

500. The carina is part of which of the following structures?A. Hyoid boneB. EpiglottisC. Trachea *D. LarynxE. Pharynx

501. The pharynx terminates at the level of which of the following structures? A. Hyoid bone B. Second cervical vertebraC. Thyroid cartilageD. Cricoid cartilage *E. Jugular notch

502. The pharyngobasilar fascia contributes to which of the following layers of the pharyngeal wall?

A. Mucous membraneB. Submucosa *C. Longitudinal muscle layerD. Circular muscle layerE. Buccopharyngeal fascia

503. The middle pharyngeal constrictor arises from which of the following structures?A. Pterygomandibular rapheB. Cricoid cartilageC. Thyroid cartilageD. Torus tubariusE. Hyoid bone *

504. Which of the following muscles enters the pharyngeal wall in the gap between the origins of the middle and superior pharyngeal constrictor muscles?

A. Stylopharyngeus *B. PalatopharyngeusC. SalpingopharyngeusD. ThyrohyoidE. Sternohyoid

505. Which of the following muscles of the pharynx is innervated by the glossopharyngeal nerve?

A. Superior pharyngeal constrictorB. SalpingopharyngeusC. Stylopharyngeus *D. PalatopharyngeusE. Middle pharyngeal constrictor

506. Which of the following ganglia is a peripheral ganglion in the course of the parasympathetic innervation of the parotid gland?

A. CiliaryB. PterygopalatineC. SubmandibularD. Otic *E. Geniculate

507. Which of the following nerves supplies parasympathetic fibers through the otic ganglion to the parotid gland?

Page 63: 1intranet.tdmu.edu.ua/data/kafedra/internal/xirtop/zbtest... · Web viewModule 1. Operative surgery and topographical anatomy of the head and neck, regions and organs of the thoracic

A. VagusB. Glossopharyngeal *C. FacialD. HypoglossalE. Accessory

508. Which of the following nerves innervates the genioglossus muscle?A. Hypoglossal *B. Ansa cervicalisC. GlossopharyngealD. VagusE. Trigeminal

509. The esophagus begins at the level of which of the following structures?A. Hyoid boneB. Thyroid cartilageC. Fourth cervical vertebraD. Cricoid cartilage *E. Jugular notch

510. The masseteric fascia is formed from which of the following?A. Superficial layer of cervical fascia *B. Carotid sheathC. Prevertebral fasciaD. BuccopharyngealE. Pretracheal fascia

511. The innervation of the masseter muscle is provided by which of the following nerves?

A. Buccal branch of the facialB. Buccal branch of the trigeminalC. Maxillary division of the trigeminalD. Inferior alveolar nerveE. Mandibular division of the trigeminal *

512. The suboccipital triangle contains which of the following structures?A. Vertebral artery *B. Lesser occipital nerveC. Spinal accessory nerveD. Occipital arteryE. Posterior auricular artery

513. In addition to hearing loss and balance disturbances, a tumor in the internal acoustic meatus may be responsible for all the following signs and symptoms except:

A. Dry eye from loss of secretion of the lacrimal glandB. Loss of secretion of the parotid gland on one side *C. Loss of secretion of the submandibular and sublingual glands on one sideD. Dry nasal mucosa form loss of secretion of the nasal glands on one sideE. Facial paralysis

514. Damage to the external laryngeal nerve during thyroid surgery will result in the patient's inability to:

A. Relax the vocal cordsB. Rotate the arytenoid cartilagesC. Tense the vocal cords *D. Widen the rima glottidisE. Abduct the vocal cords

515. A lesion of the otic ganglion would be expressed clinically by a loss of:A. Function of the sublingual glandB. Taste on the posterior one-third of the tongueC. Sensory innervation to the parotid gland

Page 64: 1intranet.tdmu.edu.ua/data/kafedra/internal/xirtop/zbtest... · Web viewModule 1. Operative surgery and topographical anatomy of the head and neck, regions and organs of the thoracic

D. Secretion of the parotid gland *E. Function of the stylopharyngeus muscle

516. A tonsillar abscess may extend posteriorly through the pharyngeal wall into the:A. Pleural cavityB. Anterior triangle of the neckC. Suprasternal spaceD. Pharyngeal tonsilE. Retropharyngeal space *

517. Which muscle may be paralyzed by a superficial knife wound to the posterior triangle to the neck?

A. Trapezius *B. SternocleidomastoidC. Anterior scaleneD. Splenius capitisE. Posterior scalene

518. Which of the following damaged ends of the cervical vein need to press for arrest of bleeding:

A. Central *B. PeripheralC. BothD. Middle of a veinE. None of the above

519. To indicate the orientators for brachial plexus anesthesia:A. Sternal end of the clavicleB. Middle of the clavicle *C. Acromial end of the clavicleD. Jugular notch E. Carotid triangle

520. Which of the following structures locates in the antescalenus cellular space:A. Venous angle *B. Phrenical nerveC. Vagus nerveD. Brachial plexusE. Inferior thyroid artery

521. Sheath for the scalenus muscles is formed by which of the following neck fasciae:A. IB. IIC. IIID. IVE. V *

522. Submandibular gland situated between laminae of which of the following neck fasciae:

A. IB. II *C. IIID. IVE. V

523. From which of the following neck cellular spaces pus passes to the posterior mediastinum:

A. Retrovisceral *B. ParapharyngealC. Interaponeurotical suprasternaleD. SubmandibularE. Previsceral

Page 65: 1intranet.tdmu.edu.ua/data/kafedra/internal/xirtop/zbtest... · Web viewModule 1. Operative surgery and topographical anatomy of the head and neck, regions and organs of the thoracic

524. The IV neck fascia forms the sheath for neurovascular bundle. To name its structures:

A. Common carotid artery, internal jugular vein, vagus nerve *B. Subclavian vein, brachial plexus, common carotid arteryC. Subclavian artery, common carotid artery, vagus nerveD. Brachial plexus, vagus nerve, common carotid arteryE. Jugular vein, vagus nerve, common carotid artery

525. The V neck fascia forms the sheath for neurovascular bundle. To name its structures:A. Vagus nerve, common carotid arteryB. Internal jugular vein, vagus nerveC. External jugular vein, phrenic nerveD. Subclavian artery, brachial plexus *E. Phrenic nerve, vagus nerve

526. Which of the following veins passes within the submandibular triangle:A. Internal jugularB. Facial *C. MentalD. Superior thyroidE. External jugular

527. Which of the following nerves situated between the common carotid artery and internal jugular vein:

A. LingualB. FacialC. Vagus *D. Reccurent laryngealE. Sympathetic trunk (superior ganglion)

528. Where can passes pus from the bed of submandibular gland:A. Floor of the oral cavity *B. Previsceral cellular spaceC. Retrovisceral cellular spaceD. Anterior mediastinumE. Posterior mediastinum

529. Which of the following neck fasciae divides the neck region on anterior and posterior parts:

A. IB. II *C. IIID. IVE. V

530. To indicate the injection point in case of vagosympathetic blockade after Vyshnevskyy:

A. Middle of the posterior edge of sternocleidomastoid muscle *B. Middle of the anterior edge of sternocleidomastoid muscleC. Jugular notchD. Lateral surface of the esophagusE. None of the above

531. Which of the following arteries haven’t branches in neck region:A. Internal carotid artery *B. External carotid arteryC. Common carotid arteryD. Thyrocervical trunkE. None of the above

532. Which of the following structures forms the neck superior border:A. Mandible *

Page 66: 1intranet.tdmu.edu.ua/data/kafedra/internal/xirtop/zbtest... · Web viewModule 1. Operative surgery and topographical anatomy of the head and neck, regions and organs of the thoracic

B. Alba lineC. Superior temporal lineD. Digastric muscleE. Vagus nerve

533. Which of the following arteries realizes the arterial supply of the pharynx:A. Superior laryngealB. Ascending pharyngeal *C. SublingualD. Ascending palatineE. Descending palatine

534. Which of the following arteries realizes the arterial supply of the cervical part of esophagus:

A. Thyrocervical trunkB. Costocervical partC. Superior thyroid arteryD. Inferior thyroid artery *E. Inferior laryngeal artery

535. The esophagus is innervated by which of the following nerves:A. Reccurent laryngeal *B. Superior laryngealC. AccessoryD. HypoglossalE. Phrenic

536. The sympathetic trunk locates under which of the following neck fasciae:A. IB. IIC. IIID. IVE. V *

537. The following statements concerning the structures in the neck are correct except which?

A. The parotid salivary gland contains within its substance the facial nerve and the external carotid artery

B. The parotid duct opens into the mouth opposite the upper second molar tooth C. As the trachea descends through the neck it rests posteriorly on the vertebral column *D. The nerve to the mylohyoid muscle innervates the anterior belly of the digastric muscleE. The hypoglossal nerve innervates the muscle of the tounge.538. Which of the following nerves might be injured when tying the inferior thyroid artery

during operations on the thyroid gland?A. The sympathetic trunkB. The internal laryngeal nerveC. The descendens cervicalisD. The recurrent laryngeal nerve *E. The superior laryngeal nerve.539. To where do the submental lymph nodes drain?A. The superficial cervical lymph nodesB. The deep cervical lymph nodes *C. The submandibular lymph nodesD. The tracheobronchial lymph nodesE. The anterior cervical lymph nodes.540. Which of the following anatomical structures forms one of the Pyrogov’s triangle

side?A. N. hypoglossus *B. M. geniohyoideus

Page 67: 1intranet.tdmu.edu.ua/data/kafedra/internal/xirtop/zbtest... · Web viewModule 1. Operative surgery and topographical anatomy of the head and neck, regions and organs of the thoracic

C. M. genioglossusD. M. styloglossusE. M. sternocleidomastoideus.

541. The retrovisceral space is boarded by the:A. I, II and III neck fasciaeB. IV, V neck fasciae *C. Two laminae of the IV neck fasciaeD. I, III neck fasciaeE. II, III neck fasciae.

542. The phrenic nerve on the neck region is located in theA. Interscalenic spaceB. Antescalenic space *C. Omovertebral triangleD. Omotracheal triangleE. Carotid triangle.543. Which of the following anatomical structures closely attaches to the posterior surface

of the external edge of thyroid gland?A. Internal jugular veinB. Parathyroid glandsC. Common carotid artery *D. Inferior laryngeal nerveE. Vagus nerve.544. Which tracheal cartilages is cutted during inferior tracheostomy?A. 3-4 *B. 1-2C. 2-3D. 5-6E. 7-8.545. How many branches originate from the internal carotid artery on the neck region?A. 6B. 9C. 1D. No one *E. 3.546. Which of the following arteries are located within the bed of the submandibular

gland?A. SubmandibularB. AngularC. Facial *D. BuccalE. Internal carotid.547. Which of the following structures lies parallel and immediately deep to the carotid

sheath in the neck?A. Superior ramus of the ansa cervicalisB. TracheaC. Sympathetic trunk *D. Internal jugular veinE. Vagus nerve

548. Each of the following arteries is a direct branch of the external carotid artery except the:

A. OccipitalB. FacialC. Posterior auricularD. Maxillary

Page 68: 1intranet.tdmu.edu.ua/data/kafedra/internal/xirtop/zbtest... · Web viewModule 1. Operative surgery and topographical anatomy of the head and neck, regions and organs of the thoracic

E. Infraorbital *549. The inferior root (ramus) of the ansa cervicalis contains nerve fibers derived from

the:A. First and second cervical spinal nervesB. Second and third cervical spinal nerves *C. Third and fourth cervical spinal nervesD. First, second, and third cervical spinal nervesE. Hypoglossal nerve

550. Each of the following is related to the walls of the laryngeal part of the pharynx except the:

A. Piriform recessB. Cricoid cartilageC. Inferior pharyngeal constrictor muscleD. Arytenoid cartilageE. Palatine tonsil *

551. Each of the following is found within the carotid sheath except the:A. Vagus nerveB. Internal carotid arteryC. Common carotid arteryD. Internal jugular veinE. Cervical sympathetic trunk *

552. The thyroid gland:A. Is ensheathed by the prevertebral layer of the deep cervical fasciaB. Has an isthmus which lies anterior to the 2nd, 3rd & 4th tracheal rings *C. Is drained only by the middle thyroid veinsD. Is related to the hypoglossal nerve on each sideE. Is ensheathed by the superficial cervical fascia553. The thyroid gland:A. Is ensheathed by the prevertebral layer of the deep cervical fasciaB. Is closely related to the recurrent laryngeal nerves on both sides *C. Is drained only by the middle thyroid veinsD. Is related to the hypoglossal nerve on each sideE. Is ensheathed by the superficial cervical fascia554. Recurrent laryngeal nervesA. Arises from tenth cranial nerve in the jugular foramenB. Supply all muscles of the vocal cordsC. Remain safe if inferior thyroid arteries are ligated away from the thyroid glandD. Are so named because they curve around larynx *E. Sensory to larynx and pharynx555. Recurrent laryngeal nervesA. Arises from tenth cranial nerve in the jugular foramenB. Supply all muscles of the vocal cordsC. Remain safe if inferior thyroid arteries are ligated away from the thyroid glandD. Sensory to larynx and pharynxE. If damaged on both sides the vocal fords are in cadaveric position *556. Recurrent laryngeal nerveA. Supplies all the muscles of larynxB. Not damaged if the ligation of inferior thyroid artery is done away from its originC. Arises from tenth cranial nerve in the jugular foramenD. Sensory to larynx and pharynxE. Receives it name because it supplies the larynx *557. Recurrent laryngeal nerveA. Arises from tenth cranial nerve in the jugular foramenB. Supply all muscles of the vocal cords

Page 69: 1intranet.tdmu.edu.ua/data/kafedra/internal/xirtop/zbtest... · Web viewModule 1. Operative surgery and topographical anatomy of the head and neck, regions and organs of the thoracic

C. Remain safe if inferior thyroid arteries are ligated away from the thyroid glandD. Sensory to larynx and pharynxE. Give branches to cardiac plexus *558. At the level of cricoid cartilage following are found:A. Junction of esophagus with pharynx *B. Entrance of superior thyroid artery into thyroid glandC. C8 vertebra exactly oppositeD. Isthmus of thyroid glandE. Bifurcation of common carotid artery559. At the level of cricoid cartilage following are found:A. Junction of the larynx with the trachea *B. Entrance of superior thyroid artery into thyroid glandC. C8 vertebra exactly oppositeD. Isthmus of thyroid glandE. Bifurcation of common carotid artery560. At the level of cricoid cartilage following are found:A. Isthmus of thyroid glandB. Entrance of superior thyroid artery into thyroid glandC. C8 vertebra exactly oppositeD. Middle cervical sympathetic ganglion *E. Bifurcation of common carotid artery561. At the level of cricoid cartilage following are found:A. Isthmus of thyroid glandB. Inferior thyroid artery enters the thyroid gland *C. C8 vertebra exactly oppositeD. Entrance of superior thyroid artery into thyroid glandE. Bifurcation of common carotid artery562. Thyroid gland

A. Enclosed in prevertebral fasciaB. Isthmus is related to 2nd-3rd tracheal rings *C. Is drained only by the middle thyroid veinsD. Is related to the hypoglossal nerve on each sideE. Is ensheathed by the superficial cervical fascia

563. Thyroid glandA. Enclosed in prevertebral fasciaB. Related to recurrent laryngeal nerve on both sides *C. Is drained only by the middle thyroid veinsD. Is related to the hypoglossal nerve on each sideE. Is ensheathed by the superficial cervical fascia

564. Thyroid glandA. Enclosed in prevertebral fasciaB. Is ensheathed by the superficial cervical fasciaC. Is drained only by the middle thyroid veinsD. Is related to the hypoglossal nerve on each sideE. Drain its venous blood partly thru middle thyroid vein *

565. Thyroid glandA. Enclosed in prevertebral fasciaB. Is related to the hypoglossal nerve on each sideC. Is drained only by the middle thyroid veinsD. Related to inferior constrictor *E. Is ensheathed by the superficial cervical fascia

566. Thyroid glandA. Is ensheathed by the superficial cervical fascia B. Moves on swallowing *

Page 70: 1intranet.tdmu.edu.ua/data/kafedra/internal/xirtop/zbtest... · Web viewModule 1. Operative surgery and topographical anatomy of the head and neck, regions and organs of the thoracic

C. Is drained only by the middle thyroid veinsD. Is related to the hypoglossal nerve on each sideE. Enclosed in prevertebral fascia

567. Thyroid glandA. Is supplied by the branches of external carotid and thyrocervical trunk *B. Enclosed in prevertebral fasciaC. Is drained only by the middle thyroid veinsD. Is related to the hypoglossal nerve on each sideE. Is ensheathed by the superficial cervical fascia

568. Thyroid glandA. Is related to the hypoglossal nerve on each side B. Is ensheathed by the superficial cervical fasciaC. Is drained only by the middle thyroid veinsD. Enclosed in prevertebral fasciaE. Has the inferior thyroid vein draining usually into the left brachiocephalic vein*

569. The following general statements concerning structures in the head and neck are correct except which?

A. The sensory nerve supply to the mucous membrane lining the upper part of the trachea is from the recurrent laryngeal nerve

B. The spinal part of the accessory nerve can be injured easily as it crosses the posterior triangle of the neck

C. The afferent sensory nerve fibers for the gag reflex are contained in the glossopharyngeal nerveD. The afferent sensory nerve fibers for the cough reflex are contained in the vagus nerveE. The sternocleidomastoid and the trapezius muscles receive their innervation from the cranial

part of the accessory nerve *570. A 59-year-old woman with a small swelling below the chin was seen by her

physician. At physical examination, a single, hard swelling could be palpated in the submental triangle. It was mobile on the deep tissues and not tethered to the skin. A diagnosis of a malignant secondary deposit in a submental lymph node was considered. Using your knowledge of anatomy, where would you look for the primary carcinoma?

A. The posterior third of the tongueB. The maxillary sinusC. The angle of the mouthD. The center of the lower lip *E. The lining of the cheek571. A 64-year-old man was seen by his physician for a hard-based ulcer on the right

lateral edge of the anterior two thirds of the tongue. Which group of nodes should be examined by the physician for possible evidence of metastases?

A. Superficial cervical nodesB. Submental nodesC. Submandibular nodes *D. Parotid nodesE. Buccal nodes

572. A patient having lunch accidentally bit the inside of her left cheek. To which lymph nodes are intecting bacteria likely to spread?

A. Mastoid nodesB. Parotid nodesC. Submental nodesD. Superficial cervical nodesE. Submandibular nodes *

573. A patient has a boil on the tip of her nose. To which lymph nodes does the lymph from the skin of the infected area drain?

A. Submandibular nodes *B. Submental nodes

Page 71: 1intranet.tdmu.edu.ua/data/kafedra/internal/xirtop/zbtest... · Web viewModule 1. Operative surgery and topographical anatomy of the head and neck, regions and organs of the thoracic

C. Parotid nodesD. Superficial cervical nodesE. Tracheobronchial nodes

574. The ______ passes between the sixth and seventh cervical vertebrae.A. Vertebral arteryB. Seventh cervical spinal nerve *C. Sixth cervical spinal nerveD. Vertebral veinE. Eighth cervical spinal nerve575. A 69-year-old woman complaining of a burning pain over the left shoulder and upper

part of the left arm was seen by her physician. The pain had started approximately 2 weeks previously and had progressively worsened. The pain was made worse by moving the neck. At physical examination, the patient showed hyperesthesia of the skin over the lower part of the left deltoid muscle and down the lateral side of the arm. In addition, her left deltoid and biceps brachii muscles were weaker than those on the right side. At radiologic examination, extensive osteoarthritic changes of the vertebrae with spur formation on the bodies of the fourth, fifth, and sixth cervical vertebrae were seen. The weakness of the left deltoid and biceps brachii muscles could have resulted from pressure on which nerve roots?

A. The posterior nerve roots of C5 and 6B. The anterior nerve roots of C5 and 6 *C. The anterior nerve roots of C7 and 8D. The anterior nerve roots of C8 and T1E. None of these576. Which of the following anatomical structures takes part in forming of “dangerous

area”, that located on posterior surface of lateral lobes of thyroid gland?A. Superior thyroid arteryB. Inferior thyroid artery *C. Superior laryngeal arteryD. Inferior laryngeal arteryE. Vagus nerve577. Which of the following anatomical structures takes part in forming of “dangerous

area”, that located on posterior surface of lateral lobes of thyroid gland?A. Parathyroid glands *B. Superior laryngeal arteryC. Inferior laryngeal arteryD. Superior thyroid arteryE. Isthmus of thyroid gland578. Which of the following anatomical structures takes part in forming of “dangerous

area”, that located on posterior surface of lateral lobes of thyroid gland?A. Vagus nerveB. Superior laryngeal nerveC. Recurrent laryngeal nerve *D. Superior laryngeal arteryE. Inferior laryngeal artery579. At which of the following distance from superior polus of lateral lobes of thyroid

gland the superior thyroid artery divides into its branches?A. 0,5 cmB. 1 cm *C. 2 cmD. 2,5 cmE. 3 cm580. In which direction you should cut the cricothyroid ligament during conicotomy?A. LongitudinalB. Transversal *

Page 72: 1intranet.tdmu.edu.ua/data/kafedra/internal/xirtop/zbtest... · Web viewModule 1. Operative surgery and topographical anatomy of the head and neck, regions and organs of the thoracic

C. ObliqueD. Z-shapedE. None of these581. Right recurrent laryngeal nerve runsA. Along lateral wall of esophagusB. Along posterior wall of esophagusC. Along anterior wall of esophagusD. Between trachea and esophagus *E. None are correct582. Left recurrent laryngeal nerve runsA. Along lateral wall of esophagusB. Along posterior wall of esophagusC. Along anterior wall of esophagus *D. Between trachea and esophagusE. None are correct583. Which of the following arteries attaches to the trachea at level of jugular notch?A. Arch of aorta *B. Isthmus af aortaC. Common carotid arteryD. Subclavian arteryE. Brachiocephalic trunk584. Which of the following arteries runs along anterior wall of trachea at level of

sternoclavicular junction?A. Arch of aortaB. Isthmus af aortaC. Common carotid artery *D. Subclavian arteryE. Brachiocephalic trunk585. Which of the following venous structures is located on anterior wall of trachea down

from the isthmus of thyroid gland?A. Inferior thyroid veinB. Unpair thyroid plexus *C. Inferior laryngeal veinD. Tracheal veinsE. Jugular venous arch

586. Blood supplying of trachea is realized by:A. Thyrocervical trunkB. Costocervical trunkC. Inferior thyroid artery *D. Inferior laryngeal arteryE. Ascending cervical artery

587. Innervation of trachea is realized by:A. Lingual nerveB. Facial nerveC. Vagus nerveD. Recurrent laryngeal nerve *E. Superior laryngeal nerve

588. Blood supplying of pharynx is realized by:A. Superior laryngeal arteryB. Ascending pharyngeal artery *C. Sublingual arteryD. Sphenopalatine arteryE. Superior thyroid artery

589. Blood supplying of pharynx is realized by:

Page 73: 1intranet.tdmu.edu.ua/data/kafedra/internal/xirtop/zbtest... · Web viewModule 1. Operative surgery and topographical anatomy of the head and neck, regions and organs of the thoracic

A. Superior laryngeal arteryB. Sphenopalatine arteryC. Sublingual arteryD. Ascending palatine artery *E. Superior thyroid artery

590. Blood supplying of pharynx is realized by:A. Descending palatine artery *B. Superior laryngeal arteryC. Sublingual arteryD. Sphenopalatine arteryE. Superior thyroid artery

591. Which of the following nerves takes part in forming of pharyngeal plexus that is located on posterior and lateral walls of pharynx?

A. Vagus nerve *B. Facial nerveC. Lingual nerveD. Superior laryngeal nerveE. Recurrent laryngeal nerve

592. Which of the following nerves takes part in forming of pharyngeal plexus that is located on posterior and lateral walls of pharynx?

A. Recurrent laryngeal nerveB. Facial nerveC. Lingual nerveD. Superior laryngeal nerveE. Sympathetic trunk *

593. Which of the following nerves takes part in forming of pharyngeal plexus that is located on posterior and lateral walls of pharynx?

A. Superior laryngeal nerveB. Facial nerveC. Lingual nerveD. Glossopharyngeal nerve *E. Recurrent laryngeal nerve

594. Origin of esophagus is placed at which of the following distance from anterior teeth?A. 5 cmB. 10 cmC. 15 cm *D. 20 cmE. 25 cm595. Length of cervical part of esophagus isA. 3 cmB. 4 cmC. 5 cm *D. 6 cmE. 7 cm596. Which of the following arteries crosses the lateral wall of esophagus at level of its

origin?A. Superior thyroid arteryB. Inferior thyroid artery *C. Superior laryngeal arteryD. Inferior laryngeal arteryE. Inferior pharyngeal artery597. Blood supplying of the cervical part of esophagus is realized by:

A. Thyrocervical trunkB. Costocervical trunk

Page 74: 1intranet.tdmu.edu.ua/data/kafedra/internal/xirtop/zbtest... · Web viewModule 1. Operative surgery and topographical anatomy of the head and neck, regions and organs of the thoracic

C. Superior thyroid arteryD. Inferior thyroid artery *E. Inferior laryngeal artery

598. Innervation of the cervical part of esophagus is realized by:A. Recurrent laryngeal nerve *B. Superior laryngeal nerveC. Sympathetic trunkD. Glossopharyngeal nerveE. Vagus nerve

599. Which of the following thyroid arteries runs parallel and inward to external branch of superior laryngeal nerve?

A. Superior thyroid artery *B. Inferior thyroid arteryC. Lowest thyroid arteryD. Superior and inferior thyroid arteriesE. Inferior and lowest thyroid arteries600. Ascending branch of which of the following thyroid arteries runs parallel to phrenic

nerve:A. Superior thyroid arteryB. Inferior thyroid artery *C. Lowest thyroid arteryD. None of theseE. All of these601. Which of the following thyroid arteries is located within pretracheal space and may

be damaged during inferior tracheostomy:A. Superior thyroid arteryB. Inferior thyroid arteryC. Lowest thyroid artery *D. None of theseE. All of these602. Branches of which of the following nerves takes part in forming of nervous plexus of

thyroid gland:A. Recurrent laryngeal nerve *B. Accesory nerveC. Glossopharyngeal nerveD. Vagus nerveE. None of these603. Branches of which of the following nerves takes part in forming of nervous plexus of

thyroid gland:A. Vagus nerveB. Accesory nerveC. Glossopharyngeal nerveD. Sympathetic trunk *E. None of these604. Branches of which of the following nerves takes part in forming of nervous plexus of

thyroid gland:A. Glossopharyngeal nerveB. Accesory nerveC. Superior laryngeal nerve *D. Vagus nerveE. None of these605. Which part of sternocleidomastoid muscle is guide for access during surgeries on

cervical part of esophagus?A. Anterior of the left

Page 75: 1intranet.tdmu.edu.ua/data/kafedra/internal/xirtop/zbtest... · Web viewModule 1. Operative surgery and topographical anatomy of the head and neck, regions and organs of the thoracic

B. Posterior of the left*C. Anterior of the rightD. Posterior of the rightE. Middle of the muscle

606. Emergency laryngotomy performed between ___________ cartilages.A. Thyroid and cricoid *B. Arytenoid and corniculateC. Thyroid and arytenoidD. Cricoid and corniculateE. Arytenoid and cricoid607. Bifurcation of common carotid artery is projected at superior edge of which of the

following cartilages?A. Thyroid *B. ArytenoidC. CricoidD. CorniculateE. Epiglottic608. Branch of which of the following nerves forms inferior cervical ansa (together with

lower root of cervical plexus)?A. Lingual nerveB. Marginal branch of facial nerveC. Hypoglossal nerve *D. Transverse nerve of neckE. Thyrohyoid nerve609. Which of the following nerves participates in forming of sinocarotid zone located in

bifurcation of common carotid artery?A. Vagus nerve *B. Facial nerveC. Lingual nerveD. Superior laryngeal nerveE. Recurrent laryngeal nerve610. Which of the following nerves participates in forming of sinocarotid zone located in

bifurcation of common carotid artery?A. Hypoglossal nerveB. Facial nerveC. Glossopharyngeal nerve *D. Superior laryngeal nerveE. Recurrent laryngeal nerve611. Which of the following nerves participates in forming of sinocarotid zone located in

bifurcation of common carotid artery?A. Sublingual nodeB. Facial nerveC. Lingual nerveD. Superior laryngeal nerveE. Sympathetic trunk *612. The first branch of external carotid artery on the neck is:A. Superior laryngeal arteryB. Superior thyroid artery *C. Lingual arteryD. Facial arteryE. Occipital artery613. Which of the following branches arises from external carotid artery at level of major

corn of hyoid bone?A. Superior laryngeal artery

Page 76: 1intranet.tdmu.edu.ua/data/kafedra/internal/xirtop/zbtest... · Web viewModule 1. Operative surgery and topographical anatomy of the head and neck, regions and organs of the thoracic

B. Lingual arteryC. Facial artery *D. Occipital arteryE. Posterior auricular artery

614. Which of the following branches of external carotid artery runs along its wall up?A. Superior laryngeal arteryB. Ascending pharyngeal artery *C. Facial arteryD. Occipital arteryE. Posterior auricular artery

615. Superior node of the sympathetic trunk is intersected by vagus nerve at level of _________ cervical vertebrae.

A. IB. IIC. III *D. IVE. V616. Which of the following nerves is source of superior cervical cardial nerve formation?A. Facial nerveB. Vagus nerveC. Recurrent laryngeal nerveD. Sympathetic trunk (superior node) *E. Accessory nerve617. Why you should ligate the neck veins before its cutting?A. for prevention air embolism *B. for prevention bleedingC. this rule does not apply to the neck veinsD. it’s important just for main veinsE. you should ligate just tributaries of external jugular vein618. What is danger of postoperative bleeding from the wounds of the anterior region of

neck?A. Possible compression of the tracheaB. Possible compression of vagus nerveC. Development of septic complicationsD. Development of air embolismE. All of these are important *619. Where is place for cervical vago-sympathetic block after O. Vyshnevskyy?A. Between sternocleidomastoid musclesB. In the corner between clavicle and external edge of sternocleidomastoid muscleC. Jugular notchD. 3 cm above the mid-clavicleE. Intersection of the posterior edge of sternocleidomastoid with external jugular vein *620. Which of the following parts of thyroid gland should be remained in case of subtotal

subfascial resection after O. Nikolaeff?A. Anterior-lateralB. IsthmusC. Superior polusD. Inferior polusE. Posterior-lateral *621. Which surgical instrument is used for dilatation of tracheal wound in case of

tracheostomy?A. Dilator by JansenB. Dilator by PasoffC. S-shaped laminae hook by Farabeuf

Page 77: 1intranet.tdmu.edu.ua/data/kafedra/internal/xirtop/zbtest... · Web viewModule 1. Operative surgery and topographical anatomy of the head and neck, regions and organs of the thoracic

D. Retractor by Troussean *E. Miculitcz retractor622. To abort an attack of paroxysmal tachycardia, an enterprising medical student

massaged the carotid sinus to reduce the heart rate. The landmark which guided him to the sinus was the:

A. Inferior border of the mandibleB. Hyoid boneC. Laryngeal prominence *D. Cricoid cartilageE. Suprasternal notch

623. Which of the following is not a branch of the superior thyroid artery?A. Infrahyoid arteryB. Sternocleidomastoid arteryC. Superior laryngeal arteryD. Ascending cervical artery *E. Cricothyroid artery

624. The thyroid gland can be examined in which of the following triangles of the neck?A. submental B. glandular C. carotid D. submandibular (or digastric) E. muscular (or visceral) *

625. The cough reflex, as do all reflexes, has a sensory and a motor part to it. What nerve carries the sensory part of the cough reflex?

A. internal laryngeal nerve *B. external laryngeal nerve C. trigeminal nerve D. facial nerve E. maxillary nerve

626. A structure that can easily be injured during ligation of the superior thyroid artery is the:

A. inferior laryngeal branch of the recurrent laryngeal nerve B. ascending pharyngeal artery C. superior parathyroid gland D. transverse colli nerve E. external branch of the superior laryngeal *

627. A surgeon removing a thyroid gland wishe to be sure that he is not removing the parathyroid glands. If he cannot find the superior parathyroids it may be becaue they are:

A. In the thoraxB. Not presentC. Embedded in the thyroidD. Easily mistaken for lymph nodesE. Any of these *

628. All of the following are surgical hazards during a thyroidectomy because of their anatomical relationships except the:

A. Accessory nerve *B. Recurrent laryngeal nerveC. Internal jugular veinD. Parathyroid glandsE. External branch of the superior laryngeal nerve

629. A young woman chokes on a piece of rare steak in a restaurant. A doctor friend cannot dislodge the steak and performs an emergency tracheotomy. Bleeding ensues. The bleeding was most likely due to perforation of the:

A. Communicating arch of the anterior jugulars and inferior thyroid veins *

Page 78: 1intranet.tdmu.edu.ua/data/kafedra/internal/xirtop/zbtest... · Web viewModule 1. Operative surgery and topographical anatomy of the head and neck, regions and organs of the thoracic

B. Inferior thyroid arteryC. Common carotid arteryD. External carotid arteryE. Communicating arch of the anterior jugulars and superior thyroid veins

630. Which of the following statements about the ansa cervicalis is false?A. It lies in front of the carotid sheathB. It is formed by a loop between the descendens hypoglossi (superior ramus) and the

descendens cervicalis (inferior ramus)C. The descendens cervicalis contains no fibers originating in the medulla oblongataD. The descendens hypoglossi contains fibers originating in the medulla oblongata *E. All of these statements are false631. The cervical plexus contributes to the nerve supply of all of the following except the:A. DiaphragmB. Infrahyoid muscleC. Trapezius muscleD. Intrinsic muscles of the tongue *E. Sternocleidomastoid muscle632. The superior cervical ganglion gives:A. White rami to C1, 2, 3, and 4B. A branch to the middle cervical ganglion, the ansa subclaviaC. Visceral branches to the esophageal plexusD. Brances to the internal carotid plexus *E. None of the above633. A superficial cervical block is carried out by injecting local anesthetic along the

middle one-third of the posterior border of the sternocleidomastoid muscle. The nerves blocked should include the:

A. Greater occipital and posterior auricular nerves *B. Greater auricular and posterior auricular nervesC. Posterior auricular nerveD. Transverse cervical nerveE. None of these634. A surgeon attempting a left phrenic nerve resection as it lies on the front of the

scalenus anterior muscle at the root of the neck may encounter the:A. Transverse cervical and suprascapular arteries and veins, thoracic duct *B. Vertebral artery, thoracic ductC. Transverse cervical veins, thoracic ductD. Vertebral and transverse cervical arteriesE. Inferior cervical ganglion of the sympathetic trunk635. To denervate the pressure receptors in the carotid sinus one must cut the:A. Glossopharyngeal nerve *B. Vagus nerveC. Accessory nerveD. Ansa hypoglossiE. Cervical sympathetic trunk below the middle cervical ganglion636. The recurrent laryngeal nerve:A. Runs in the groove between the esophagus and the tracheaB. Enters the larynx through the cricothyroid membraneC. Supplies motor fibers to the thyroarytenoideus muscleD. All are correct *E. None of these637. The vocal folds extend between the:A. Thyroid and cricoid cartilagesB. Cricoid and arytenoid cartilagesC. Thyroid and arytenoid cartilages *

Page 79: 1intranet.tdmu.edu.ua/data/kafedra/internal/xirtop/zbtest... · Web viewModule 1. Operative surgery and topographical anatomy of the head and neck, regions and organs of the thoracic

D. Arytenoid and epiglottic cartilagesE. None of these638. The motor nerve to the cricothyroid muscle is the:A. Internal branch of the superior laryngeal nerveB. Glossopharyngeal nerveC. Recurrent laryngeal nerveD. Cervical branch of the facial nerveE. External branch of the superior laryngeal nerve *639. All of the following are surgical hazards during a thyroidectomy because of their

anatomical relationships except the:A. Accessory nerve *B. Recurrent laryngeal nerveC. Internal jugular veinD. Parathyroid glandsE. External branch of the superior laryngeal nerve640. If a surgeon removing a parathyroid adenoma were to injure the nerve which

constitutes a major hazard in this procedure, one of the muscles which would be paralyzed would be the:

A. Middle pharyngeal constrictorB. DiaphragmC. Lateral thyroarytenoid *D. ThyrohyoidE. Mylohyoid641. The middle constrictor of the pharynx arises partly from the:A. Oblique line of the thyroid cartilageB. Thyrohyoid membraneC. Body of the hyoid boneD. Stylohyoid ligament *E. Posterior margin of the cricoid cartilage642. Which of the following contribute branches to the pharyngeal plexus of nerves?A. The superior cervical ganglionB. The glossopharyngeal nerveC. The vagus nerveD. All of these *E. None of these643. The stylopharyngeus muscle is a landmark in locating the glossopharyngeal nerve.

The nerve lies:A. Lateral the muscleB. Middle to the muscleC. Posterior to the muscleD. All of these are correct depending upon the level of dissection *E. All are not correct644. Transection of the superior laryngeal nerve would result in paralysis of the:A. Posterior cricoarytenoid muscleB. Thyroarytenoid muscleC. Thyrohyoid muscleD. Cricothyroid muscle *E. Middle pharyngeal constrictor645. A patient complains of laryngeal stridor (crowing) on exertion. On laryngoscopic

examination you observe that the normal looking vocal cords are adducted. You suspect that the abductors of the vocal cords are inoperative. These are the:

A. Cricothyroid muscleB. Thyroarytenoid muscleC. Anterior cricoarytenoid muscle

Page 80: 1intranet.tdmu.edu.ua/data/kafedra/internal/xirtop/zbtest... · Web viewModule 1. Operative surgery and topographical anatomy of the head and neck, regions and organs of the thoracic

D. Posterior cricoarytenoid muscle *E. All of these646. If the right recurrent laryngeal nerve were injured during a thyroidectomy, laryngeal

muscles which would be functioning would include the:A. Right cricothyroid muscleB. Left thyroarytenoid muscleC. Left cricothyroid muscleD. All of these *E. None of these647. The thyroid gland receives blood from the:A. External carotid artery and costocervical trunkB. External carotid and subclavian arteries *C. Internal and external carotid arteriesD. Internal carotid and subclavian arteriesE. Subclavian artery and costocervical trunk648. Indication for vago-sympathetic block after Vyshnevskyy is:A. Pleuro-pulmonary shock *B. Hepatic failureC. Postoperative shock in case of injure of pelvic organsD. Renal failureE. Cardiac failure649. Indication for vago-sympathetic block after Vyshnevskyy is:A. Hepatic failureB. Postoperative shock in case of injure of pelvic organsC. Postoperative shock in case of injure of thoracic and abdominal cavities*D. Renal failureE. Cardiac failure650. Complication after subtotal subfascial resection of thyroid gland is:A. Vagus nerve damageB. Bleeding *C. Horner’s syndromeD. Tracheoesophageal fistulaE. Tracheitis651. Complication after subtotal subfascial resection of thyroid gland is:

A. Vagus nerve damageB. Phrenic nerve damageC. Horner’s syndromeD. Excision of parathyroid glands *E. Difficult swallowing

652. Complication after subtotal subfascial resection of thyroid gland is:A. Vagus nerve damageB. Recurrent nerve damage * C. Laryngeal edemaD. Fat embolismE. Laryngeal cyst

653. Complication after subtotal subfascial resection of thyroid gland is:A. Vagus nerve damageB. Compression of recurrent nerve by hematoma *C. Pharynx reflexD. Retropharyngeal abscessE. Difficult swallowing

654. Complication after subtotal subfascial resection of thyroid gland is:A. Vagus nerve damageB. Asphyxia *

Page 81: 1intranet.tdmu.edu.ua/data/kafedra/internal/xirtop/zbtest... · Web viewModule 1. Operative surgery and topographical anatomy of the head and neck, regions and organs of the thoracic

C. Lacunar anginaD. Laryngeal anginaE. All of these

655. Complication after subtotal subfascial resection of thyroid gland is:A. Vagus nerve damageB. Violation of voice (hoarseness, aphonia) *C. Laryngeal stenosisD. Chondroperichondritis of larynxE. Laryngeal angina

656. Complication after subtotal subfascial resection of thyroid gland is:A. Vagus nerve damageB. Air embolism *C. Lacunar anginaD. Laryngeal anginaE. Pharynx reflex

657. Complication after subtotal subfascial resection of thyroid gland is:A. Vagus nerve damageB. Recurrent toxic goiter *C. Chondroperichondritis of larynxD. Horner’s syndromeE. Tracheoesophageal fistula658. Complication after subtotal subfascial resection of thyroid gland is:

A. BleedingB. Excision of parathyroid glandsC. Recurrent nerve damage D. Compression of recurrent nerve by hematomaE. All are correct *

659. Complication after subtotal subfascial resection of thyroid gland is:A. AsphyxiaB. Violation of voice (hoarseness, aphonia)C. Air embolismD. Recurrent toxic goiterE. All are correct *660. The main method of surgical treatment of nodular goiter is:

A. Subtotal subfascial resection of thyroid lobe with node *B. Resection of thyroid glandC. StrumectomyD. All methods can be usedE. None of the above

661. The main method of surgical treatment of nodular goiter is:A. Resection of thyroid glandB. StrumectomyC. Enucleation *D. HemityreoidectomyE. All are correct

662. The main method of surgical treatment of nodular goiter is:A. Resection of thyroid glandB. StrumectomyC. HemityreoidectomyD. Enucleation with resection *E. None of these

663. The main method of surgical treatment of nodular goiter is:A. Subtotal subfascial resection of thyroid lobe with nodeB. Enucleation

Page 82: 1intranet.tdmu.edu.ua/data/kafedra/internal/xirtop/zbtest... · Web viewModule 1. Operative surgery and topographical anatomy of the head and neck, regions and organs of the thoracic

C. Enucleation with resectionD. All methods can be used *E. None of the above

664. The main method of surgical treatment of diffuse goiter is:A. Subtotal subfascial resection of thyroid gland after Nykolaeff *B. Resection of thyroid glandC. EnucleationD. HemityreoidectomyE. Enucleation with resection

665. Length of tracheal incision is equal to diameter of which of the following instruments?

A. Tracheostomy cannula *B. Tracheal dilatorC. Wound dilatorD. BronchoscopeE. Artificial airway

666. Indication for tracheostomy:A. Damage of larynx or trachea if it is found to be impossible to remove of asphyxia *B. Tracheoesophageal fistulaC. Chondroperichondritis of larynxD. Compression of recurrent nerve by hematomaE. Laryngeal cyst

667. Indication for tracheostomy:A. Laryngeal cystB. Stenosis of larynx or trachea *C. Tracheoesophageal fistulaD. Laryngeal anginaE. Pharynx reflex

668. Indication for tracheostomy:A. Lacunar anginaB. Laryngeal cystC. Chondroperichondritis of larynxD. False croup *E. Tracheoesophageal fistula669. Indication for tracheostomy:A. Foreign bodies of larynx *B. Tracheoesophageal fistulaC. Chondroperichondritis of larynxD. Laryngeal cystE. Pharynx reflex670. Indication for tracheostomy:A. Pharynx reflexB. Laryngeal anginaC. Violation of voice (hoarseness, aphonia)D. Traumatic brain injury *E. Tracheoesophageal fistula671. Indication for tracheostomy:A. Damage of larynx or trachea if it is found to be impossible to remove of asphyxiaB. Stenosis of larynx or tracheaC. False croupD. Foreign bodies of larynxE. All of these *672. Which of the following instruments is necessary for performance of tracheostomy?A. One-tooth sharp hook *

Page 83: 1intranet.tdmu.edu.ua/data/kafedra/internal/xirtop/zbtest... · Web viewModule 1. Operative surgery and topographical anatomy of the head and neck, regions and organs of the thoracic

B. S-shaped laminae hook by FarabeufC. Artificial airwayD. Kocher’s probeE. Buiallsky’s spatula673. Which of the following instruments is necessary for performance of tracheostomy?A. Wound dilatorB. Tracheostomy cannula *C. S-shaped laminae hook by FarabeufD. Artificial airwayE. Kocher’s probe674. Which of the following instruments is necessary for performance of tracheostomy?A. Wound dilatorB. Buiallsky’s spatulaC. Blunt hook for isthmus of thyroid gland *D. S-shaped laminae hook by FarabeufE. Artificial airway675. Which of the following instruments is necessary for performance of tracheostomy?A. One-tooth sharp hookB. Tracheostomy cannulaC. Blunt hook for isthmus of thyroid glandD. Tracheal dilatorE. All are correct *676. Which of the following symptoms is evidence of correctly performed vago-

sympathetic blockade after Vyshnevskyy?A. Narrowing of pupilsB. Hyperemia of faceC. Hyperemia of tunica albuginea of eyeD. Retraction of eyeballE. All are correct *677. Which of the following symptoms is evidence of correctly performed vago-

sympathetic blockade after Vyshnevskyy?A. Dilation of pupilsB. Narrowing of pupils *C. Dilation of interpalpebral spaceD. ExophthalmosE. Tachycardia678. Which of the following symptoms is evidence of correctly performed vago-

sympathetic blockade after Vyshnevskyy?A. Dilation of pupilsB. Increase of arterial pressureC. Hyperemia of face *D. Dilation of interpalpebral spaceE. Exophthalmos679. Which of the following symptoms is evidence of correctly performed vago-

sympathetic blockade after Vyshnevskyy?A. Dilation of pupilsB. BradycardiaC. ExophthalmosD. Hyperemia of tunica albuginea of eye *E. Dilation of interpalpebral space680. Which of the following symptoms is evidence of correctly performed vago-

sympathetic blockade after Vyshnevskyy?A. Decrease of arterial pressureB. Dilation of pupils

Page 84: 1intranet.tdmu.edu.ua/data/kafedra/internal/xirtop/zbtest... · Web viewModule 1. Operative surgery and topographical anatomy of the head and neck, regions and organs of the thoracic

C. Dilation of interpalpebral spaceD. Narrowing of interpalpebral space *E. Exophthalmos681. Which of the following symptoms is evidence of correctly performed vago-

sympathetic blockade after Vyshnevskyy?A. Retraction of eyeball *B. Dilation of pupilsC. BradycardiaD. Dilation of interpalpebral spaceE. Exophthalmos682. The following statements concerning the structures in the neck are correct except

which?A. The parotid salivary gland contains within its substance the facial nerve and the external

carotid arteryB. The parotid duct opens into the mouth opposite the upper second molar tooth C. As the trachea descends through the neck it rests posteriorly on the vertebral column *D. The nerve to the mylohyoid muscle innervates the anterior belly of the digastric muscleE. The hypoglossal nerve innervates the muscle of the tounge.683. Which of the following nerves might be injured when tying the inferior thyroid artery

during operations on the thyroid gland?A. The sympathetic trunkB. The internal laryngeal nerveC. The descendens cervicalisD. The recurrent laryngeal nerve *E. The superior laryngeal nerve.684. To where do the submental lymph nodes drain?A. The superficial cervical lymph nodesB. The deep cervical lymph nodes *C. The submandibular lymph nodesD. The tracheobronchial lymph nodesE. The anterior cervical lymph nodes.685. Which of the following anatomical structures forms one of the Pyrogov’s triangle

side?A. N. hypoglossus *B. M. geniohyoideusC. M. genioglossusD. M. styloglossusE. M. sternocleidomastoideus.686. The retrovisceral space is boarded by the:A. I, II and III neck fasciaeB. IV, V neck fasciae *C. Two laminae of the IV neck fasciaeD. I, III neck fasciaeE. II, III neck fasciae.687. The phrenic nerve on the neck region is located in theA. Interscalenic spaceB. Antescalenic space *C. Omovertebral triangleD. Omotracheal triangleE. Carotid triangle.688. Which of the following anatomical structures closely attaches to the posterior surface

of the external edge of thyroid gland?A. Internal jugular veinB. Parathyroid glands

Page 85: 1intranet.tdmu.edu.ua/data/kafedra/internal/xirtop/zbtest... · Web viewModule 1. Operative surgery and topographical anatomy of the head and neck, regions and organs of the thoracic

C. Common carotid artery *D. Inferior laryngeal nerveE. Vagus nerve.689. Which tracheal cartilages is cutted during inferior tracheostomy?A. 3-4 *B. 1-2C. 2-3D. 5-6E. 7-8.690. How many branches originate from the internal carotid artery on the neck region?A. 6B. 9C. 1D. No one *E. 3.691. Which of the following arteries are located within the bed of the submandibular

gland?A. SubmandibularB. AngularC. Facial *D. BuccalE. Internal carotid.692. Which of the following statements concerning intercostals nerves is correct?

A. There are 11 pairs of thoracic spinal nervesB. The thoracic spinal nerves are commonly called subcostal nervesC. The ventral rami of the thoracic spinal nerves supply muscle, bone, joint, and skin of

the backD. They supply the parietal pleura and are mainly sensory nervesE. They are the ventral rami of the thoracic spinal nerves *

693. Which of the following structures passes through the aortic hiatus of the diaphragm?A. Vagus nerveB. EsophagusC. Inferior vena cavaD. Sympathetic trunkE. Azygos vein *

694. All of the following veins drain into the coronary sinus except the:A. Great cardiac veinB. Middle cardiac veinC. Oblique vein of left atriumD. Small cardiac veinE. Anterior cardiac veins *

695. The major venous return system of the heart, the coronary sinus, empties into the:A. Inferior vena cavaB. Left atriumC. Superior vena cavaD. Right ventricleE. Right atrium *

696. Which of these statements is true in relation to intercostal nerves?A. In the intercostal space, they run between the internal and external intercostals

musclesB. They are located in the costal groove above the artery and veinC. They are all confined to the thoraxD. They are entirely cutaneous to the thoracic wallE. The upper six nerves terminate as anterior cutaneous branches *

Page 86: 1intranet.tdmu.edu.ua/data/kafedra/internal/xirtop/zbtest... · Web viewModule 1. Operative surgery and topographical anatomy of the head and neck, regions and organs of the thoracic

697. All the following structures empty into the right atrium of the heart except the:A. Coronary sinusB. Inferior vena cavaC. Superior vena cavaD. Anterior cardiac veinsE. Pulmonary veins *

698. The lateral boundary of the superior mediastinum is the:A. Lateral border of the sternumB. T-1 to T-4 vertebraeC. Sternal angleD. T-2 to T-5 vertebraeE. Mediastinal pleura *

699. Most of the sternocostal surface of the heart is formed by the:A. Right atriumB. Left ventricleC. Right auricleD. Left atriumE. Right ventricle *

700. Each of the following arteries is a branch of the internal thoracic except the:A. PericardiophrenicB. MusculophrenicC. Superior epigastricD. Anterior intercostalsE. Superior phrenic *

701. Which of the following structures passes posterior to the root of the right lung?A. Hemiazygos veinB. Right phrenic nerveC. Thoracic aortaD. Right recurrent laryngeal nerveE. Right vagus nerve *

702. Each of the following vessels empties into the coronary sinus except the:A. Great cardiac veinB. Middle cardiac veinC. Small cardiac veinD. Posterior vein of the left ventricleE. Anterior cardiac vein *

703. Each of the following structures is found in the posterior mediastinum except the:A. Vagus nerveB. Azygos veinC. Descending aortaD. EsophagusE. Phrenic nerve *

704. The brachiocephalic veins receive venous blood directly from each of the following except the:

A. Subclavian veinB. Internal jugular veinC. Inferior thyroid veinD. Internal thoracic veinE. External jugular vein *

705. Each of the following is related to the right ventricle except the:A. Interventricular septumB. Trabeculae cameaeC. Anterior papillary muscleD. Septomarginal band

Page 87: 1intranet.tdmu.edu.ua/data/kafedra/internal/xirtop/zbtest... · Web viewModule 1. Operative surgery and topographical anatomy of the head and neck, regions and organs of the thoracic

E. Bicuspid valve *706. The greater splanchnic nerve usually contains nerve fibers derived from each of the

following spinal nerves except:A. T-5B. T-9C. T-8D. T-7E. T-10 *

707. The left coronary artery bifurcates into the circumflex branch and the:A. Left marginal branchB. Left ventricular branchC. Right marginal branchD. Posterior interventricular branchE. Anterior interventricular branch *

708. The posterior intercostal artery in the fifth intercostal space arises from theA. Musculophrenic arteryB. Internal thoracic arteryC. Costocervical trunkD. Subclavian arteryE. Thoracic aorta *

709. All of the following can be found in the coronary sulcus (atrioventricular groove) except the

A. Right coronary arteryB. Circumflex branch of the left coronary arteryC. Great cardiac veinD. Coronary sinusE. Middle cardiac vein *

710. The most anterior of the following structures in the superior mediastinum is the:A. Aortic archB. Left common carotid arteryC. Right common carotid arteryD. Phrenic nerveE. Brachiocephalic vein *

711. Each of the following arteries is a branch of the descending thoracic aorta except the:A. Posterior intercostalB. EsophagealC. BronchialD. PericardialE. Left subclavian *

712. Within the superior mediastinum, the anterior surface of the esophagus is in contact with the:

A. Anterior longitudinal ligament of the vertebral columnB. Thoracic ductC. ThymusD. Arch of the aortaE. Trachea *

713. The thoracic duct is correctly described by each of the following except it:A. Is the largest lymphatic channel in the bodyB. Arises in a dilatation known as the cistema chyli within the abdomenC. Returns to the bloodstream lymph from all of the body below the diaphragm and

from the left half of the body above the diaphragmD. Courses through the posterior mediastinum between the aorta and the azygos veinE. Diverges in the superior mediastinum to the right side of the esophagus, towards the

right superior aperture of the neck *

Page 88: 1intranet.tdmu.edu.ua/data/kafedra/internal/xirtop/zbtest... · Web viewModule 1. Operative surgery and topographical anatomy of the head and neck, regions and organs of the thoracic

714. On the diaphragmatic surface of the heart, the posterior interventricular sulcus separates which of the following chambers?

A. Right ventricle-right atriumB. Left ventricle-left atriumC. Left atrium-right atriumD. Left ventricle-right atriumE. Left ventricle-right ventricle *

715. The parietal pleura is described by each of the following terms except:A. DiaphragmaticB. CervicalC. CostalD. MediastinalE. Basal *

716. The blood supply to the anterior two-thirds of the interventricular septum is provided by the:

A. Marginal branch of the right coronary arteryB. Marginal branch of the left coronary arteryC. Circumflex branch of the left coronary arteryD. Posterior interventricular arteryE. Anterior interventricular artery *

717. Which one of the following structures in the posterior mediastinum is found immediately posterior to the left atrium and pericardium?

A. Vagus nervesB. Azygos veinC. Thoracic ductD. Right pulmonary arteryE. Esophagus *

718. Each of the following statements correctly describes the costodiaphragmatic recess except:

A. It is an inferior extension of the pleural cavityB. It represents an area where the costal and diaphragmatic layers of parietal pleura can

come into apposition with each otherC. In quiet respiration, it is usually found deep to the eighth and ninth intercostals

spaces in the midaxillary lineD. It contains a thin film of serous fluidE. It is a sub-compartment of the thoracic cavity *

719. The most superior structure at the root of the left lung is the:A. Superior pulmonary veinB. Primary bronchusC. Inferior pulmonary veinD. Bronchiolar arteryE. Pulmonary artery *

720. The horizontal fissure of the right lung separates theA. Superior and inferior lobesB. Inferior and middle lobesC. Superior lobe from the cardiac notchD. Superior lobe and lingulaE. Superior and middle lobes *

721. Each of the following is found at the horizontal plane of the sternal angle except the:A. Superior termination of the fibrous pericardiumB. Intervertebral disk between the fourth and fifth thoracic vertebraeC. Beginning and termination of aortic archD. Bifurcation of trachea

Page 89: 1intranet.tdmu.edu.ua/data/kafedra/internal/xirtop/zbtest... · Web viewModule 1. Operative surgery and topographical anatomy of the head and neck, regions and organs of the thoracic

E. Bifurcation of the brachiocephalic artery into the right subclavian and right common carotid arteries *

722. The part of the rib that articulates with the transverse process of the vertebra is the:A. Superior articular facetB. Inferior articular facetC. AngleD. NeckE. Tubercle articular facet *

723. The superior vena cava is formed:A. Deep to the right second costal cartilageB. Deep to the right sternoclavicular jointC. Deep to the right subclavian arteryD. Anterior to the aortic archE. Deep to the right first costal cartilage *

724. Deep to the fourth intercostal space on the right side of the sternum is the:A. Superior vena cavaB. Right brachiocephalic veinC. Left brachiocephalic veinD. Brachiocephalic trunkE. Right atrium *

725. The esophageal hiatus transmits the esophagus and the:A. Superior phrenic arteryB. Thoracic ductC. Greater splanchnic nerveD. Lesser splanchnic nerveE. Vagal nerve trunks *

726. The musculophrenic artery is a branch of which of the following arteries?A. Inferior epigastricB. Superficial epigastricC. Superficial circumflexD. Deep external pudendalE. Internal thoracic *

727. The aortic hiatus of the diaphragm is located at the level of which vertebra?A. Fourth cervicalB. Sixth cervicalC. Fifth thoracicD. Fourth lumbarE. 12th thoracic *

728. The mediastinum contains all the following structures except the:A. HeartB. Pulmonary arteriesC. TracheaD. EsophagusE. Lungs *

729. Anatomically components of the proper thoracic wall include the:A. Pectoral musclesB. Serratus anteriorC. TrapeziusD. Latissimus dorsiE. Intercostal muscles *

730. The superior thoracic aperture can be described correctly by which of these statements?

A. It is large and irregularB. It is bounded by the third thoracic vertebra

Page 90: 1intranet.tdmu.edu.ua/data/kafedra/internal/xirtop/zbtest... · Web viewModule 1. Operative surgery and topographical anatomy of the head and neck, regions and organs of the thoracic

C. It includes the costal archD. It involves costal cartilages of the tenth ribE. Its plane slopes downward and forward *

731. The sternal angle is found at which of these locations?A. Jugular notchB. Xiphoid processC. Level with the fourth costal cartilageD. Level with the lower border of the sixth thoracic vertebraE. Manubriosternal joint *

732. Ossification of the parts of the body of the sternum usually is complete by age (in years):

A. OneB. ThreeC. SixD. 15E. 21 *

733. Which of the following defines true ribs?A. All 12 pairsB. Lower five pairsC. 10th and 11th pairsD. 12th pairE. Upper seven pairs *

734. All the following are correct statements about rib fractures exceptA. Most frequently they occur as a result of compression forces on the thoraxB. Most often they occur just anterior to the costal angleC. Broken rib ends tend to spring outwardD. Fractured bone ends may injure the lungsE. Splinting the chest wall is essential treatment *

735. Which of these statements correctly describes intercostal muscles?A. External intercostals begin anteriorlyB. External intercostal membrane is posteriorC. Fibers of external intercostals slant upward and backwardD. Innermost intercostals are the best developed of the intercostalsE. Fibers of internal intercostals run upward and forward *

736. Innervation of the thoracic wall can be described, correctly by all the following statements except:

A. It receives a nerve supply from spinal nerves T1-12B. Cutaneous innervation of skin over paravertebral regions of the thorax is provided by

dorsal rami of spinal nervesC. Ventral rami of T1-11 are called intercostals nervesD. The ventral ramus of T12 is the subcostal nerveE. It receives no nerve supply from cervical nerves *

737. Which of these statements is true in relation to intercostal nerves?A. In the intercostal space, they run between the internal and external intercostal

musclesB. They are located in the costal groove above the artery and veinC. They are all confined to the thoraxD. They are entirely cutaneous to the thoracic wallE. The upper six nerves terminate as anterior cutaneous branches *

738. Which of the following statements is correct regarding the blood vessels of thethoracic wall?

A. In the intercostal space, the vessels run just below the respective intercostal nerveB. Branches of the vessels vary widely from those of the intercostal nervesC. Posterior intercostal arteries are branches of the internal thoracic artery

Page 91: 1intranet.tdmu.edu.ua/data/kafedra/internal/xirtop/zbtest... · Web viewModule 1. Operative surgery and topographical anatomy of the head and neck, regions and organs of the thoracic

D. Branches of the descending thoracic aorta become anterior intercostal arteriesE. Superficial structures of the thorax are served by intercostal vessels *

739. All the following are true statements about the anterior thoracic artery except:A. It descends behind the subclavian veinB. It divides into two terminal branchesC. It gives branches to the mediastinumD. The musculophrenic artery is one of its terminal branchesE. It is a branch of the arch of the aorta *

740. Innervation of pleura can be described correctly by all of these statements except:A. Costal pleura is supplied by intercostal nervesB. The central portion of diaphragmatic pleura is supplied by the phrenic nerveC. Peripheral diaphragmatic pleura is supplied by intercostal nervesD. Mediastinal pleura is supplied by the phrenic nerveE. The pain of pleurisy is mediated by autonomic nerves *

741. Which of the following statements is correct in relation to internal anatomy of the lung?

A. Arteries and veins provide the major lung frameworkB. Bronchi branch symmetricallyC. Bronchi are hollow tubes without particular wall supportD. Pulmonary vessels show no particular relationship to bronchial branchingE. Bronchopulmonary segments are the anatomic units *

742. Which of these items is true regarding external anatomy of the lung?A. The upper tapered end of the lung is its baseB. The root of the lung is located at its baseC. Lobes are comparable to bronchopulmonary segmentsD. Each lung has three lobesE. Visceral pleura covers all lung surfaces *

743. Which of these statements is correct regarding pulmonary circulation of the lung?A. It is the main blood supply to the bronchiB. It is the main blood supply to the connective tissue of the lungC. The pulmonary trunk goes directly to the left lungD. Pulmonary veins enter the right atrium of the heartE. The pulmonary trunk arises from the right ventricle *

744. The ligamentum arteriosum is located correctly by which of the following?A. Between the ductus arteriosus and the right pulmonary arteryB. Between the left pulmonary vein and the aortaC. Between the right pulmonary vein and the pulmonary trunkD. Between the left bronchial artery and the aortic archE. Between the left pulmonary artery and the aortic arch *

745. All the following are correct statements about pulmonary veins except:A. Two veins pass from the hilum of each lungB. They show more variation than do the pulmonary arteriesC. They are formed by confluence of capillaries in the lungD. Their primary tributaries are related to particular bronchopulmonary segmentsE. Usually they enter the right atrium of the heart *

746. The bronchial arteries may arise from all the following except the:A. Descending aortaB. Right intercostal arteryC. Arch of the aortaD. Subclavian arteryE. Anterior thoracic artery *

747. Which of the following statements describes nerve supply of the lungs correctly?A. Sympathetic fibers in the pulmonary plexus are preganglionic fibersB. Vagal fibers in the pulmonary plexus are postganglionic fibers

Page 92: 1intranet.tdmu.edu.ua/data/kafedra/internal/xirtop/zbtest... · Web viewModule 1. Operative surgery and topographical anatomy of the head and neck, regions and organs of the thoracic

C. The vagus innervates the smooth muscle in walls of pulmonary vessetsD. Sympathetic fibers control constriction of the bronchiE. Visceral afferents from the lung have been demonstrated only in the vagus nerve *

748. All of the following statements about the pericardial sac are true except:A. The outer layer is fibrousB. Epicardium completely invests the heartC. The pericardial sac and its contents comprise the middle mediastinumD. The pericardial sac is fused to the central tendon of the diaphragmE. The fibrous pericardium lubricates the moving surfaces of the heart *

749. Which of these items correctly describes the heart?A. All the great veins enter its apexB. The apex points forward and toward the rightC. The diaphragmatic surface is formed largely by the right ventricle and atriumD. The coronary sinus occupies the posterior interventricular sulcusE. Its base is made largely of the left atrium and a portion of the right atrium *

750. The right atrium includes all of these structures except the:A. Tricuspid valveB. Crista terminalisC. Musculi pectinatiD. Fossa ovalisE. Trabeculae cameae *

751. Which of these statements correctly describes the azygos venous system?A. Normally it drains into the inferior vena cavaB. It is located entirely on the right side of the vertebral columnC. It receives no blood from thoracic visceraD. It has a number of valvesE. Primarily it drains blood from the body wall *

752. All of these items correctly described the thoracic duct except:A. It returns lymph from the greater part of the body to the venous systemB. It is the upward continuation of the cisterna chyliC. In most of its course it lies behind the esophagusD. It contains valvesE. It ends at the confluence of the right subclavian and brachiocephalic veins *

753. Which of the following statements is correct in relation to thoracic splanchnic nerves?

A. Their fibers relay in the sympathetic ganglion chainB. They are part of the cardiac plexusC. Usually they are five in numberD. They consist of parasympathetic nerve fibersE. They are composed predominantly of preganglionic fibers *

754. Correct description of structure of the sternum includes which of the following statements?

A. The jugular notch is located at the lower border of the manubriumB. The sternal angle is located at the articulation of the body and the xiphoid processC. The body consists of five fused stemebraeD. The xiphoid process consists of bone thicker than that of the bodyE. The upper border of the body is located at the level of the costal cartilage of the

second rib *755. Ribs may be described correctly by all the following except:

A. Every rib articulates with the vertebral columnB. The upper seven pairs of ribs are called vertebrosternalC. Ribs 8, 9, and 10 are called vertebrochondral ribsD. Floating ribs are the last two pairsE. Ribs one through 12 are called true ribs *

Page 93: 1intranet.tdmu.edu.ua/data/kafedra/internal/xirtop/zbtest... · Web viewModule 1. Operative surgery and topographical anatomy of the head and neck, regions and organs of the thoracic

756. Which of these facts about ribs is not true?A. The sternal end of each arch lies at a lower level than the vertebral endB. Ribs and cartilages increase in length progressively from first to seventh ribC. The transverse diameter of the thorax increases progressively from first to eighth ribD. The ninth is the most obliquely placed ribE. The typical rib takes an upward slope *

757. All of these statements about intercostal arteries are correct except:A. The upper two posterior intercostal arteries arise from the supreme intercostal arteryB. The lower nine posterior intercostal arteries arise from the aortaC. Intercostal arteries run under the shelter of a costal grooveD. Intercostal arteries accompany each intercostal nerveE. The superior epigastric artery supplies anterior intercostal arteries *

758. Which of the following structures is not located in the mediastinum?A. Heart and pericardiumB. TracheaC. Vessels proceeding to and from the heartD. Vagus nervesE. Lungs *

759. All of the following are parts of the parietal pleura except:A. CostalB. MediastinalC. DiaphragmaticD. CervicalE. Pulmonary *

760. All of these statements describe the aortic arch correctly except:A. It lies in the superior mediastinumB. It begins at the level of the sternal angleC. It gives off the left common carotid arteryD. It gives off the right and left coronary arteriesE. It gives off the right subclavian artery *

761. Correct description of the recurrent laryngeal nerves includes which of the following?

A. They are branches of the phrenic nerveB. They branch from the sympathetic trunkC. Their neuronal cell bodies are in the cervical spinal cordD. The right nerve recurs around the superior vena cavaE. The left nerve recurs around the ligamentum arteriosum *

762. Each segmental bronchus together with the portion of lung it supplies is called:A. Primary segmentB. Lobar segmentC. Epiarterial segmentD. Alveolar segmentE. Bronchopulmonary segment *

763. Correct information about bronchial arteries includes all the following items except:A. They arise by a stem from the aortaB. They supply the pulmonary pleuraC. They supply the bronchiD. They run through the interlobar structuresE. The pressure within them is low *

764. Nerves of the lungs and pleura include all of these except:A. Branches of the vagus contribute to the pulmonary plexusB. Branches of the thoracic sympathetic ganglia 1 through 5 help to form the pulmonary

plexusC. Sensory vagal fibers constitute the afferent limb of the respiratory reflex arc

Page 94: 1intranet.tdmu.edu.ua/data/kafedra/internal/xirtop/zbtest... · Web viewModule 1. Operative surgery and topographical anatomy of the head and neck, regions and organs of the thoracic

D. Efferent vagal fibers are secretomotorE. Visceral pleura has many afferent nerves sensitive to mechanical stimulation *

765. The tracheal bifurcation can be seen at which of these levels?A. T8 in the erect subjectB. T6 during inspirationC. T12 during expirationD. T2-3 in the supine cadaverE. T4-5 in the supine living subject *

766. If the phrenic nerve was cut close to its origin, which of these effects could be seen?A. Loss of bronchoconstrictionB. Loss of power in intercostal musclesC. Difficulty in expirationD. Loss of the respiratory reflex arcE. Loss of sensation in the middle diaphragm *

767. The brachiocephalic veins are formed from the union of the:A. External jugular and inferior vena cavaB. Ductus arteriosum and superior vena cavaC. Azygos vein and the axillary veinD. Pulmonary vein and inferior vena cavaE. Internal jugular and subclavian *

768. All the following structures empty into the right atrium of the heart except the:A. Coronary sinusB. Inferior vena cavaC. Superior vena cavaD. Anterior cardiac veinsE. Pulmonary veins *

769. In the adult, which of the following structures is not prominent in the anterior mediastinum?

A. Loose areolar tissueB. Lymph vessels and nodesC. FatD. Sternopericardial ligamentsE. Thymus gland *

770. The thoracic aorta gives off all of these branches except:A. Bronchial arteriesB. Pericardial arteriesC. Diaphragmatic arteriesD. One pair of subcostal arteriesE. The first pair of posterior intercostal arteries *

771. Which of these statements correctly describes the position of the esophagus?A. It passes anterior to the left principal bronchusB. It descends on the left of the aortic archC. It runs in the anterior mediastinumD. It passes through the hiatus formed by the median arcuate ligament of the diaphragmE. It begins at the level of the cricoid cartilage *

772. Which of the following statements is true of the trachea?A. It descends behind the esophagusB. Its posterior surface is convexC. During inspiration, its bifurcation ascendsD. It contains 0-shaped bars of cartilageE. It ends at the level of the sternal angle *

773. All of the following statements correctly describe the left vagus nerve except:A. It gives off the left recurrent laryngeal nerveB. Posterior to the left bronchus, it breaks up into the left pulmonary plexus

Page 95: 1intranet.tdmu.edu.ua/data/kafedra/internal/xirtop/zbtest... · Web viewModule 1. Operative surgery and topographical anatomy of the head and neck, regions and organs of the thoracic

C. It forms part of the esophageal plexusD. It descends from the neck posterior to the left common carotid arteryE. It supplies the left side of the diaphragm *

774. Which of the following correctly describes the aortic arch?A. It passes to the right of the tracheaB. It receives the ligamentum arteriosum from the right pulmonary arteryC. It gives off the right subclavian arteryD. It gives off the left brachiocephalic trunkE. It begins posterior to the sternal angle *

775. The superior vena cava returns blood from all of these structures except the:A. HeadB. NeckC. Upper limbD. Thoracic wallE. Lungs *

776. All of the following statements correctly describe the brachiocephalic veins except:A. Each is formed by the union of the internal jugular and the subclavian veinsB. They unite to form the superior vena cavaC. Each vein receives the internal thoracic veinD. They arise posterior to the medial ends of the clavicleE. They contain valves to prevent backflow of blood *

777. All of these structures occupy the superior mediastinum except the:A. ThymusB. Aortic archC. TracheaD. EsophagusE. Heart and pericardium *

778. Which of the following statements is true regarding the coronary arteries?A. Sharp lines of demarcation exist between their distribution to right and left ventriclesB. Most of the blood in these arteries returns to the left atriumC. Variations of these arteries are uncommonD. They are infrequent sites of arteriosclerosisE. They arise from the right and left aortic sinuses *

779. Which of these statements correctly describes sensation of the heart?A. Referred pain from the heart rarely occursB. Heart ischemia rarely results in generation of painful stimuliC. Afferent pain fibers from the heart run in the vagus nerveD. Usually pain from the heart is felt only in the left chestE. The heart is insensitive to cold, heat, and touch *

780. All of the following correctly describe innervation of the heart except:A. Sympathetics increase rate of heart beatB. Parasympathetics reduce rate and force of heart beatC. The impulse conducting system of the heart is controlled by the autonomic nervous

systemD. The parasympathetic cardiac nerves supply three branches to each sideE. Vagal stimulation dilates coronary arteries *

781. Which of the following is not a characteristic of the left ventricle?A. Its wall is much thicker than that of the right ventricleB. Its interior is covered by trabeculae cameaeC. The chordae tendinae of papillary muscles are distributed to cusps of the

atrioventricular valveD. The aorta arises from its anterior uppermost partE. It forms the base of the heart *

782. Characteristics of the left atrium consist of all the following except:

Page 96: 1intranet.tdmu.edu.ua/data/kafedra/internal/xirtop/zbtest... · Web viewModule 1. Operative surgery and topographical anatomy of the head and neck, regions and organs of the thoracic

A. It forms most of the base of the heartB. It contains a few musculi pectinatiC. Much of this atrium lies posterior to the right atriumD. The auricle overlaps the root of the pulmonary trunkE. It receives the pulmonary arteries *

783. Which of the following is characteristics of the right ventricle:A. Ussualy it has only two pappilary musclesB. It receives blood through the mitral valveC. It has internal muscular ridges, the musculi pectinatyD. It contains the fossa ovalisE. It gives origin to the pulmonary trunk *

784. Which of the following correctly describes chambers of the heart:A. The coronary sulcus separates the two ventriclesB. The right ventricle forms the right border of the heartC. The valve of the superior vena cava directs blood downwardD. The interventricular septum contains the fossa ovalisE. The superior vena cava opens into the right atrium *

785. The heart may correctly be described by all the following except:A. An apex formed by the tip of the left ventricleB. A diaphragmatic surface formed by both ventriclesC. A base formed by the atriaD. A location in the middle mediastinumE. An anterior surface formed mainly by the left atrium *

786. The epicardium receives its arterial blood supply from the:A. PericardiophrenicB. MusculophrenicC. Superior phrenicD. BronchialE. Coronary arteries *

787. Which of these statements is true of the fibrous pericardium?A. It has no close relationship with the central tendon of the diaphragmB. It moves freely within the thoracic cavityC. Its base is pierced by the aortaD. It has no attachment to the sternumE. It extends upward to the level of the sternal angle *

788. Which of these statements correctly describes lymphatic drainage of the lungs?A. Usually both bronchomediastinal lymph trunks terminate in the thoracic ductB. Only one lymphatic plexus is involved in this drainageC. Little transfer of lymph drainage from side to side occursD. Rarely is this lymph drainage responsible for transfer of cancer cells to other organsE. No lymph vessels are located in the walls of the pulmonary alveoli *

789. In what way does the root of the right lung differ from that of the left lung?A. In numbers of pulmonary veinsB. In numbers of primary bronchiC. In the presence of a pulmonary plexus of nervesD. In numbers of bronchial veinsE. In numbers of pulmonary arteries *

790. Characteristics of the left lung include which of the following?A. It is heavier than the right lungB. It is composed of three lobesC. The azygos vein arches over its rootD. It has a horizontal fissureE. The cardiac notch is found on its superior lobe *

Page 97: 1intranet.tdmu.edu.ua/data/kafedra/internal/xirtop/zbtest... · Web viewModule 1. Operative surgery and topographical anatomy of the head and neck, regions and organs of the thoracic

791. The central part of the parietal diaphragmatic pleura is supplied by which of these nerves?

A. IntercostalsB. VagusC. ParasympatheticsD. SympatheticsE. Phrenic *

792. All of the following statements concerning the atrioventricular valves are true, except:

A. The valves are attached to the anuli fibrosiB. The right atrioventricular (tricuspid) valve is formed by the posterior, inferior and

septal cuspsC. Each cusp receives chordae tendinae from more than one papillary muscleD. The chordae tendinae of the mitral valve are thicker than those of the tricuspidE. The left atrioventricular (mitral) valve is formed by the septal and left cusps *

793. The pericardial sinuses areof considerable interest to the cardiac surgeon. When the surgeon’s finger is in the transverse sinus, which of the following structures is not related to the sinus in the manner noted?

A. Aorta, anterior to the sinusB. Pulmonary artery, anterior to the sinusC. Left atrium, posterior to the sinusD. Superior left pulmonary vein, posterior to the sinusE. Superior vena cava, anterior to the sinus *

794. Which of the following statements about the pericardial sinuses is untrue?A. Only two major vessels, both arteries, lie anterior to an instruments in the transverse

sinusB. The left atrium lies anterior to an instrument in the oblique sinusC. The superior and inferior venae cavae lie in the right wall of the obligue sinusD. A sharp instrument passed through the left part of the posterior wall of the transverse

sinus could enter the left atriumE. None of these statements is true *

795. During cardiac surgery, which of the following vessels could be compressed between a finger placed in the transverse sinus and an anteriorly placed thumb?

A. The superior vena cavaB. The inferior vena cavaC. The descending aortaD. The right pulmonary veinE. The pulmonary artery *

796. The chamber contributing most to the anterior (ventral) aspect of the heart is the:A. Right atriumB. Left atriumC. Left ventricleD. All four chambers contribute about equallyE. Right ventricle *

797. The right border of the heart is formed by the:A. Right ventricleB. Left atriumC. Right and left atriaD. Right atrium and right ventricleE. Right atrium *

798. Which of the following is in direct posterior relation to the left atrium?A. The right atriumB. The tracheaC. The aorta

Page 98: 1intranet.tdmu.edu.ua/data/kafedra/internal/xirtop/zbtest... · Web viewModule 1. Operative surgery and topographical anatomy of the head and neck, regions and organs of the thoracic

D. The transverse pericardial sinusE. The esophagus *

799. The base of the heart corresponds to the:A. Diaphragmatic surface of the heartB. Left ventricle and left atriumC. Right ventricle and right atriumD. None of theseE. Left and right atria *

800. A physician has detected something while examining a patient’s heart. He is now listening carefully over the fourth intercostal space immediately to the left of the sternum. His suspicion has evidently been directed toward the:

A. Mitral valveB. Aortic valveC. Pulmonic valveD. Valve of the coronary sinusE. Tricuspid valve *

801. You hear a loud systolic murmur (i.e., while the ventricles are contracting) most clearly over the right second chondrosternal junction. You suspect:

A. Pulmonary stenosis (narrowing of the pulmonary valve)B. Mitral stenosis (narrowing of the mitral valve)C. Tricuspid stenosis (narrowing of the tricuspid valve)D. A patent ductus arteriosusE. Aortic stenosis (narrowing of the aortic valve) *

802. During a surgical procedure involving the coronary arteries, you would find the circumflex artery in the:

A. Anterior atrioventricular grooveB. Posterior interventricular grooveC. Right atrioventricular sulcusD. Left atrioventricular sulcusE. Posterior interatrial sulcus *

803. The left coronary artery:A. Arises from the anterior surface of the aortaB. Ordinarily supplies the posterior interventricular branchC. Ordinarily supplies the diaphragmatic portion of the right ventricleD. All theseE. None of these *

804. A man past 50 years old suddenly collapsed and died while shoveling snow. A massive area of necrotic heart muscle was found over the anterior wall of the right ventricle with the involvement of the anterior wall of the left ventricle as well. Assuming that this was the result of acute coronary arterial occlusion, the occlusion was probably of the:

A. Left coronary arteryB. Circumflex branchC. Right coronary arteryD. Coronary sinusE. Anterior interventricular artery *

805. The artery of the sinuatrial node is usually a branch of the:A. Posterior interventricular arteryB. Anterior interventricular arteryC. Anterior ventricular arteryD. Left coronary arteryE. Right coronary artery *

806. The coronary sinus receives blood from the veins listed below, except the:A. Small cardiac veinB. Middle cardiac vein

Page 99: 1intranet.tdmu.edu.ua/data/kafedra/internal/xirtop/zbtest... · Web viewModule 1. Operative surgery and topographical anatomy of the head and neck, regions and organs of the thoracic

C. Oblique vein of the left atriumD. Great cardiac veinE. Anterior cardiac veins *

807. All of the following arteries are branches of the right coronary artery, except the:A. Nodal branchB. Anterior ventricular branchesC. Posterior interventricular branchD. Right marginal branchE. Anterior interventricular branch *

808. The apex of the heart usually receives most of its blood supply from the:A. Circumflex branch of the left coronary arteryB. Posterior interventricular arteryC. Marginal branch of the right coronary arteryD. Main division of the right coronary arteryE. Anterior interventricular artery *

809. The great cardiac vein contributes to the drainage of all of the chambers of the heart except the:

A. Left atriumB. Right ventricleC. Left ventricleD. It contributes to the drainage of all of the chambersE. Right atrium *

810. Which of the following is usually a branch of the left coronary artery?A. Nodal branchB. Anterior ventricular branchesC. Posterior interventricular branchD. Right marginal branchE. Anterior interventricular branch *

811. The vein which can be seen in the anterior interventricular sulcus with the interventricular branch of the left coronary artery is the:

A. Anterior cardiac veinB. Small cardiac veinC. Middle cardiac veinD. Left marginal veinE. Great cardiac vein *

812. The following statements concerning an intercostal space are correct except which?A. The anterior intercostals arteries of the lower five intercostals spaces are branches of

the musculophrenic arteryB. The sensory fibers in the lower five intercostals nerves supply the skin of lateral

thoracic and anterior abdominal wallsC. The posterior intercostals arteries of the lower nine spaces are branches of the

thoracic aortaD. The intercostals nerves and blood vessels run between the internal and the innermost

intercostals muscles.E. Throughout an intercostals space, the intercostals nerves and blood vessels lie close

to the upper border of the lower rib *813. The following statements concerning the structure of the heart are correct except

which?A. The trabeculae carneae are internal surface structures of both the left and the right

ventriclesB. The coronary arteries are functional end arteriesC. The sinuatrial node is supplied by the right and sometimes the left coronary arteryD. The four pulmonary veins open through the posterior wall of the left atrium and there

are no valves.

Page 100: 1intranet.tdmu.edu.ua/data/kafedra/internal/xirtop/zbtest... · Web viewModule 1. Operative surgery and topographical anatomy of the head and neck, regions and organs of the thoracic

E. The pericardial cavity is the potential space between the fibrous and the serous pericardia *

814. The following statements regarding the innervation of thoracic structures are correct except which?

A. The lung and visceral pleura are innervated by the autonomic nerves and are not sensitive to sensation of temperature, touch, and pressure

B. The sensory nerve supply to the mucous membrane of the lower part of the trachea is from the vagus and the recurrent laryngeal nerves

C. The nerve supply of the pericardium is the phrenic nervesD. The sinuatrial node is supplied by sympathetic and parasympathetic nerves via the

cardiac plexuses.E. The motor innervation of the diaphragm is provided by the third, fourth, and fifth

cervical spinal nerves and by the lower six intercostals nerves *815. The following statements concerning thoracic structures are correct except which?

A. The carina is the name given to the site of bifurcation of the tracheaB. The ligamentum arteriosum is the remains of the ductus arteriosusC. The ductus arteriosus is formed from the sixth left pharyngeal archD. The thymus receives its arterial supply mainly from the internal thoracic artery.E. The thymus lies in the middle mediastinum *

816. The following events occur on inhalation except which?A. The diaphragm descendsB. The external intercostals muscles contractC. The ribs are raisedD. The vertical dimension of the thoracic cavity increases.E. The abdominal muscles contract and push the abdominal viscera cranially *

817. When passing a needle through the chest wall and into the pleural cavity in the midaxillary line, the following structures will be pierced except which?

A. The external intercostal muscleB. The skinC. The parietal pleuraD. The internal intercostal muscle.E. The levator costarum *

818. The following statements concerning the bronchopulmonary segments are correct except which?

A. It is a subdivision of a lung lobeB. It is surrounded by connective tissueC. It has a segmental bronchus, a segmental artery, lymph vessels, and autonomic

nervesD. When diseased, it can be removed surgically as a structural unit.E. It is pyramidal in shape, with its apex toward the lung surface *

819. The following statements concerning the main bronchi are correct except which?A. The right main bronchus is wider than the left main bronchusB. The right main bronchus is shorter than the left main bronchusC. The right main bronchus is longer than the left main bronchusD. The left main bronchus passes to the left in front of the esophagusE. The left main bronchus gives off the superior lobar bronchus before entering the

hilum of the lung. *820. The following structures open into the right atrium except which?

A. The superior vena cavaB. The coronary sinusC. The anterior cardiac veinD. The inferior vena cavaE. The right pulmonary veins. *

Page 101: 1intranet.tdmu.edu.ua/data/kafedra/internal/xirtop/zbtest... · Web viewModule 1. Operative surgery and topographical anatomy of the head and neck, regions and organs of the thoracic

821. The conducting system of the heart is composed of the following structures except which?

A. The Purkinje plexusB. The sinuatrial nodeC. The atrioventricular bundleD. The atrioventricular node.E. The deep cardiac plexus *

822. In repairing a patent interventricular septum, a cardiac surgeon must beware of catching the stem of the atrioventricular bundle in his stitches. The most dangerous area is _________ to the septal defect.

A. AnteriorB. SuperiorC. InferiorD. The atrioventricular bundle is not near the septal defectE. Posterior *

823. All of the following structures are anterior to the pericardium except the:A. ThymusB. Sternal lymph nodesC. Internal thoracic arteryD. Transversus thoracis muscleE. Phrenic nerve *

824. The only one of the following structures which lies in contact with the fibrous pericardium is the:

A. Azygos veinB. Thoracic sympathetic gangliaC. Thoracic ductD. Bodies of the thoracic vertebraeE. Phrenic nerve *

825. An aneurysm (sac-like dilatation) on the ascending aorta also involves the anterior sinus. Which of the following is anatomically unlikely?

A. Erosion into the pulmonary arteryB. Involvement of the right coronary arteryC. Rupture into the pericardiumD. Extension forward to erode the sternumE. Pressure on the recurrent laryngeal nerve causing hoarseness *

826. The right ventricle includes all of the structures listed below, except the:A. Conus arteriosus pulmonarisB. Trabeculae carneaeC. Anterior, septal and posterior papillary musclesD. Crista supraventricularisE. Aortic ostium *

827. All of the following are closely applied to the anterior surface of the esophagus except the:

A. TracheaB. Left bronchusC. PericardiumD. DiaphragmE. All of these are closely applied to its anterior surface *

828. The sternal angle serves as a landmark for locating the:A. First ribB. Third ribC. Fourth ribD. None of the ribsE. Second rib *

Page 102: 1intranet.tdmu.edu.ua/data/kafedra/internal/xirtop/zbtest... · Web viewModule 1. Operative surgery and topographical anatomy of the head and neck, regions and organs of the thoracic

829. All of the following are useful surface landmarks for thoracic structures except the:A. Sternal angleB. Suprasternal notchC. Anterior axillary foldD. Xiphisternal junctionE. Nipple in the female *

830. A stab wound with a two inch blade in the left fifth interspace close to the edge of the sternum would most likely penetrate the:

A. Left atriumB. Right atriumC. Left ventricleD. LiverE. Right ventricle *

831. A stab wound just lateral to and at the level of the right nipple, passing six inches straight posteriorly would penetrate all of the following except the:

A. Pectoralis major muscleB. Pectoralis minor muscleC. Middle lobe of the right lungD. Lower lobe of the right lungE. Right lobe of the liver *

832. A perforating wound involving the pleural cavity is likely to result in a pneumothorax. The right pleural sac could be punctured by a stab wound in any of the following places except the:

A. Ninth intercostal space – midaxillary lineB. Level of the sternal angle at right border of the sternumC. Ninth intercostal space – midscapular lineD. Anterior part of the posterior triangle of the neck an inch above the clavicleE. Ninth intercostal space – midclavicular line *

833. A man was shot with a 22 caliber rifle. The bullet entered his chest through the right fourth intercostal space in the midclavicular line and emerged at the inferior angle of his right scapula. He can be expected to have:

A. Hemorrhage into his pericardiumB. Horner’s syndrome from injury to his right sympathetic chainC. A traumatic aneurysm of the thoracic aortaD. Perforation of the diaphragm with injurry to the liverE. Collapse of the right lung *

834. A bullet entering the chest in the midline at the sternal angle and striking the disc between the fourt and fifth thoracic vertebrae would be expected to perforate which of the following last?

A. The ascending aortaB. The bifurcation of the tracheaC. The thymusD. None of these would be perforatedE. The esophagus *

835. The apex of the heart is located in the left:A. Third interspaceB. Fourth interspaceC. Sixth interspaceD. None of theseE. Fifth interspace *

836. If, during quiet breathing, you locate the inferior border of the right lung at the eighth rib in the midaxillary line, the border is:

A. Two spaces lower than normalB. One space lower than normal

Page 103: 1intranet.tdmu.edu.ua/data/kafedra/internal/xirtop/zbtest... · Web viewModule 1. Operative surgery and topographical anatomy of the head and neck, regions and organs of the thoracic

C. One space higher than normalD. Two spaces higher than normalE. In normal position *

837. During a sternal marrow puncture in the lower half of the manubrium the needle inadvertently was inserted too far and produced 20 ml of blood. The source of the blood was probably the:

A. Left atriumB. Pulmonary veinC. Azygos veinD. Right atriumE. Aorta *

838. A horizontal anteroposterior bullet wound at the left edge of the sternum in the fifth interspace would most likely penetrate the:

A. Right atriumB. Superior vena cavaC. Inferior vena cavaD. It would be unlikely to perforate any of theseE. Right ventricle *

839. A patient with a wound from a 22 caliber bullet in the tenth intercostal space in the right midaxillary line was X-rayed and the bullet was found to have penetrated medially about two inch inches. It is almost certain that the bullet was lodged in the:

A. Costophrenic recessB. Right lungC. Posterior mediastinumD. Right atriumE. Liver *

840. A physician wishes to drain fluid from a pleural recess on the right side. With the patient in maximum expiration he can place his needle at the lateral border of the erector spinae in the:

A. Eighth intercostal spaceB. Ninth intercostal spaceC. Subcostal spaceD. Cannot be done from behindE. Eleventh intercostal space *

841. A horizontal anteroposterior bullet wound at the right edge of the sternum in the fifth interspace would most likely penetrate the:

A. Right ventricleB. Superior vena cavaC. Inferior vena cavaD. It would be unlikely to perforate any of theseE. Right atrium *

842. A physician wishing to drain fluid from the pericardial cavity by approaching from the front must be careful to avoid all but which one of the following?

A. The pleuraB. The internal thoracic vesselsC. The intercostal vesselsD. All of these are at risk and must be carefully avoidedE. The phrenic nerve *

843. All of the following are located partly or entirely above the level of the sternal angle (Louis) except the:

A. Arch of the aortaB. Left recurrent laryngeal nerveC. Left brachiocephalic veinD. Remains of the thymus gland

Page 104: 1intranet.tdmu.edu.ua/data/kafedra/internal/xirtop/zbtest... · Web viewModule 1. Operative surgery and topographical anatomy of the head and neck, regions and organs of the thoracic

E. Pulmonary valve *844. All of the following are related to the sternal angle, except the:

A. Bifurcation of the tracheaB. Arch of the aortaC. Attachment of the second rib to the sternumD. Termination of the azygos veinE. Termination of the hemiazygos vein *

845. A chest X-ray of a one-year-old child shows a broad shadow occupying the superior mediastinum. The child is free from any symptoms of disease. The shadow is probably that of:

A. His noncalcified sternumB. His congenitally abnormal heartC. His double aortic archD. A tumorE. His thymus *

846. All of the following structures are located in the superior mediastinum, except the:A. ThymusB. Aortic archC. Thoracic ductD. EsophagusE. Azygos vein *

847. The density of the shadow of the superior mediastinum on a chest x-ray is determined by the size of all but which one of the following structures in the normal adult?

A. The superior vena cavaB. The brachiocephalic arteryC. The left common carotid arteryD. The left subclavian arteryE. All of these contribute to the shadow *

848. If the intercostal muscles are paralyzed, the most obvious effect which can be observed by a physician in the living subject is that:

A. Breathing is impossibleB. Breathing is obviously extremely difficultC. Movement of the liver in respiration is decreasedD. Expiration, but not inspiration, is impairedE. The intercostal spaces move in during inspiration *

849. The volume of the thorax is increased through the action of all of the following muscles except the:

A. External intercostalsB. Levator costarumC. DiaphragmD. Serratus anteriorE. Rectus abdominis *

850. Inspiration is accomplished by all of the following except:A. Elevation of the rib cage with an increase in the cross-sectional area of the thoracic

cavityB. Contraction of the diaphragmC. The fact that the pleural cavities are completely closed and contain no airD. Atmospheric pressureE. Contraction of the abdominal wall musculature *

851. All of the following muscles can be involved in respiratory movements except the:A. Serratus anteriorB. Scalenus anteriorC. SubcostalsD. Transversus thoracis

Page 105: 1intranet.tdmu.edu.ua/data/kafedra/internal/xirtop/zbtest... · Web viewModule 1. Operative surgery and topographical anatomy of the head and neck, regions and organs of the thoracic

E. Psoas major *852. In several clinical condition, patients use all the major and accessory muscles of

respiration. Which of the following is not a muscle of inspiration?A. Pectoralis majorB. Scalenus anteriorC. Erector spinaeD. Sarratus anteriorE. All of these can assist in inspiration *

853. Which of the following muscles is not used in forced respiration?A. Scalenus anteriorB. SternomastoidC. Rectus abdominisD. Serratus anteriorE. All of these are used *

854. In a case where the eleventh rib on the left side was fractured by a blow, what critical situation might arise?

A. Torn diaphragmB. Punctured pleuraC. Punctured spleenD. Punctured kidneyE. All of these may arise *

855. Which of the following structures is adjacent to the left lung?A. The superior vena cavaB. The inferior vena cavaC. The arch of azygos veinD. The right phrenic nerveE. None of these *

856. If you were to open the right pleural cavity by a vertical incision parallel to the border of the sternum, and slide your hand in dorsally between the mediastinal pleura and the medial surface of the lung below the right pulmonary veins, you would be stopped by encountering the:

A. Vertebral columnB. DiaphragmC. Thymus glandD. AortaE. Pulmonary ligament *

857. The two pleural cavities are in contact:A. Above the aortic archB. Behind the left atriumC. Underneath the aortic arch (“the aortic window”)D. They are not in contactE. Behind the sternal angle *

858. The trachea bifurcates at the level of the _________ vertebral body:A. Seventh cervicalB. Second thoracicC. Sixth thoracicD. Eighth thoracicE. Fourth thoracic *

859. If you were to explore the interval between the mediastinal pleura and the fibrous pericardium anterior (ventral) to the root of the lung, the only one of the following structures you would expect to encounter would be the:

A. Greater splanchnic nerveB. Inferior vena cavaC. Internal thoracic artery

Page 106: 1intranet.tdmu.edu.ua/data/kafedra/internal/xirtop/zbtest... · Web viewModule 1. Operative surgery and topographical anatomy of the head and neck, regions and organs of the thoracic

D. Deep cardiac plexusE. Phrenic nerve *

860. Which of the following structures is least likely to be involved in the local spread of a carcinoma of the apex of the right lung?

A. The vagus nerveB. The right phrenic nerveC. The right sympathetic trunkD. The first thoracic nerve to the brachial plexusE. The thoracic duct *

861. The right lung:A. Is smaller than the left lungB. Has a lingula which is homologous to the middle lobe of the left lungC. Has a cardiac notch in addition to an impressionD. Lies adjacent to the aorta near the hilusE. Has an epiarterial bronchus *

862. In lung surgery which of the following may be of use as guide to distinguish the line of separation between adjacent bronchopulmonary segments?

A. the pulmonary arteries and the bronchiB. the pulmonary arteries and pulmonary veinsC. the pulmonary arteries and the connective tissue septaD. there is no line of separation, adjacent bronchopulmonary segments merely diffuse

into one anotherE. the pulmonary veins and the connective tissue septa *

863. Diseases of the lungs sometimes lead to involvement of the structures in contact with them. Which of the following structures is not adjacent to the right lung?

A. the superior vena cavaB. the inferior vena cavaC. the arch of azygos veinD. the right phrenic nerveE. all of these are adjacent to the right lung *

864. Structures which are in contact with, and groove, the left lung include all of the following except the:

A. aortic archB. left subclavian arteryC. esophagusD. all of these are in contact with the left lungE. inferior vena cava *

865. The mediastinal surface of the right lung presents impressions corresponding to all of the following structures except the:

A. esophagusB. right brachiocephalic veinC. azygos veinD. superior vena cavaE. all of these make impressions on the right lung *

866. An infant is brought to you having “swallowed” an open safety pin. Respiratory distress suggests to you that it may have entered the trachea. this is a dangerous situation because of the close relationship between the trachea and all but which one of the following?

A. the arch of the aortaB. the brachiocephalic arteryC. the esophagusD. the superior vena cavaE. all of these are closely related *

867. The thoracic duct:

Page 107: 1intranet.tdmu.edu.ua/data/kafedra/internal/xirtop/zbtest... · Web viewModule 1. Operative surgery and topographical anatomy of the head and neck, regions and organs of the thoracic

A. is without valvesB. is thick walledC. lies to the left of the aorta at the level of T8D. lies deep to the lateral arcuate ligamentE. passes through the aortic opening of the diaphragm *

868. Which of the following lymph nodes are most likely to compress the left recurrent laryngeal nerve if they become involved in malignant disease?

A. posterior thoracicB. retrosternalC. infraclavicularD. broncho-pulmonaryE. tracheobronchial or paratracheal *

869. The anterior intercostal arteries are branches of the:A. ascending aortaB. descending aortaC. superior epigastric arteryD. lateral thoracic arteryE. internal thoracic artery *

870. The drainage of the superior vena cava includes blood from the:A. second intercostals spaceB. right quadratus lumborumC. left lungD. tracheaE. all of these *

871. The vessels with the shortest intrathoracic course is the:A. superior vena cavaB. ascending aortaC. azygos veinD. left brachiocephalic veinE. inferior vena cava *

872. The concavity of the arch of the aorta is in close or direct relationship with all of the following except the:

A. right pulmonary arteryB. left bronchusC. left recurrent laryngeal nerveD. ligamentum arteriosumE. right superior pulmonary vein *

873. Coarctation of the aorta is a congenital constriction which usually occurs just distal to the ligamentum arteriosum. Which of the following findings would you expect in such a case?

A. blood pressure unequal in the two armsB. blood pressure higher in the right arm and leg than in the left arm and legC. blood pressure higher in the legs than in the armsD. blood pressure equal wherever takenE. blood pressure higher in the arms than in the legs *

874. An aneurysm of the arch of the aorta could produce all of the following except:A. hoarsenessB. sternal erosionC. dullness to percussion in the second intercostals space on the left side of the sternumD. dyspnoea (difficulty in breathing)E. it could produce any or all of the above *

875. Branches of the internal thoracic artery furnish blood supply to the:A. breastB. pericardium

Page 108: 1intranet.tdmu.edu.ua/data/kafedra/internal/xirtop/zbtest... · Web viewModule 1. Operative surgery and topographical anatomy of the head and neck, regions and organs of the thoracic

C. diaphragmD. abdominal musclesE. all are correct *

876. The vagus nerve supplies fibers to each of the plexus below except the:A. cardiac plexusB. pharyngeal plexusC. pulmonary plexusD. esophageal plexusE. brachial plexus *

877. From your knowledge of the sensory innervation of the pericardium, you might expect pain from the pericardium to be referred to the area of skin which has the same segmental innervation. This is predominantly the:

A. external auditory canalB. medial side of the arm and forearmC. epigastriumD. there is no sensory innervation of the pericardiumE. side of the neck and upper shoulder *

878. The thoracic portion of the sympathetic trunk becomes involved in many disease processes. To which of the structures listed below is it not closely related?

A. the neck of the first ribB. the second thoracic nerveC. the posterior intercostals arteriesD. the parietal pleuraE. the esophagus *

879. The heart receives parasympathetic innervation by way of the:A. hypoglossal nerveB. greater splanchnic nerveC. phrenic nerveD. superior cardiac nervesE. vagus nerve *

880. Which branch of which vagus nerve hooks around the aortic arch adjacent to the ligamentum arteriosum?

A. The right recurrent laryngeal nerveB. The left superior laryngeal nerveC. The anterior vagal trunkD. The vagal cardiac branchesE. The left recurrent laryngeal nerve *

881. The nerve passing anterior to the root of the left lung is the:A. Vagus nerveB. Left recurrent laryngeal nerveC. Sympathetic trunkD. None of these is anterior to the lung rootE. Phrenic nerve *

882. The pain accompanying episodes of pleurisy arises from nerve endings in the:A. Visceral pleuraB. Mediastinal pleuraC. Pulmonary ligamentD. All of theseE. Parietal pleura *

883. Surgical treatment of patent ductus arteriosus involves closure of the ductus. Of the following, the structure nearest to the operative field is the:

A. Greater splanchnic nerveB. Phrenic nerveC. Sympathetic trunk

Page 109: 1intranet.tdmu.edu.ua/data/kafedra/internal/xirtop/zbtest... · Web viewModule 1. Operative surgery and topographical anatomy of the head and neck, regions and organs of the thoracic

D. None of these is near the ductus arteriosusE. Left recurrent laryngeal nerve *

884. Which of the following nerves is not related to the arch of the aorta?A. The left vagusB. The left cervical cardiac branches of the vagusC. The left cervical cardiac branches of the sympathetic plexusD. The left phrenic nerveE. The left greater splanchnic nerve *

885. A patient whom you believe to have a cancer of the lung because of the finding of cancer cells in the sputum, complains of pain over the left shoulder. This would lead you to suspect invasion of the mediastinum immediately:

A. Above the root of the left lungB. Below the root of the left lungC. Anterior to the root of the left lungD. None of theseE. All of these *

886. During unaccustomed exercise, pain may be produced in the diaphragm. The innervation of the peripheral margin of the diaphragm from which this pain may be felt is by the:

A. Phrenic nervesB. Splanchnic nervesC. Vagus nervesD. All of these nervesE. Intercostal nerves *

887. All of the following levels for opening in the diaphragm are correct except the:A. Esophageal hiatus (T10)B. Aortic hiatus (T12)C. Thoracic duct (T12)D. All of these are trueE. Inferior vena cava (T12) *

888. The central tendon of the diaphragm is pierced by the:A. AortaB. Right greater splanchnic nerveC. EsophagusD. Right vagus nerveE. Inferior vena cava *

889. Which of the following statements in regard to the diaphragm are true?A. It has attachments to the sternumB. It has attachments to the lower six ribsC. It has attachments to the bodies to the first three lumbar vertebraeD. The pericardium is fused to its superior surfaceE. All of these are true *

890. Which of the following are in contact with the diaphragm?A. The middle lobe of the right lungB. The lingulaC. The right ventricleD. The let ventricleE. All of these are in contact with the diaphragm *

891. A horizontal stab wound with a short blade in the left second intercostal space close to the sternum is most likely to injure the:

A. Left brachiocephalic veinB. Left atriumC. Left ventricleD. Right ventricle

Page 110: 1intranet.tdmu.edu.ua/data/kafedra/internal/xirtop/zbtest... · Web viewModule 1. Operative surgery and topographical anatomy of the head and neck, regions and organs of the thoracic

E. Aorta *892. Which of the following is true about Meckel's diverticulum?

A. It is usually in the duodenumB. It lies on the mesenteric side of the intestinal wallC. It is due to the persistence of the mullerian ductD. It is usually asymptomatic *E. It is present in more than 30% of the population

893. Which of the following is true about blood flow through the liver?A. The liver has a triple blood supply from the hepatic artery, the cystic artery, and the

portal veinB. The caudate and quadrate lobes derive their blood supply in part from the right

hepatic arteryC. Blood supply from the portal vein alone would not provide the liver with sufficient

oxygenation *D. The hepatic sinusoids are lined with a continuous endothelium and substantial basal

laminaE. All of these are true

894. Which of the following statements concerning the anterior abdominal wall is true? A. Between the costal margin and the umbilicus, the posterior layer of the sheath of the

rectus abdominis muscle is formed by the aponeurosis of the internal oblique and transversus abdominis muscles *

B. The deep inguinal ring is in the aponeurosis of the transversus abdominis muscleC. The external spermatic fascia is continuous with the transversalis fasciaD. The internal spermatic fascia is derived from the internal oblique aponeurosisE. The anterior wall of the inguinal canal is formed primarily by the inguinal and

lacunar ligaments895. Which of the following statements concerning the inferior epigastric artery is true?

A. It lies medial to a direct inguinal herniaB. lt lies lateral and posterior to an indirect inguinal herniaC. It is a branch of the internal iliac arteryD. It is a route of collateral circulation in coarctation of the aorta *E. It anastomoses with the musculophrenic artery

896. Each of the following receives a contribution from the external abdominal oblique muscle or its fascia except the:

A. Lacunar ligamentB. External spermatic fasciaC. Inguinal ligamentD. Anterior sheath of the rectus abdominisE. Falx inguinalis *

897. The lateral boundary of the inguinal traingle is the:A. Inguinal ligamentB. Rectus abdominis muscleC. Ischiopubic ramiD. Urogenital diaphragmE. Inferior epigastric artery *

898. The superior mesenteric artery supplies each of the following except the:A. AppendixB. Rectum *C. PancreasD. CecumE. Transverse colon

899. The posterior boundary of the epiploic foramen is formed by the:A. DuodenumB. Lesser omentum

Page 111: 1intranet.tdmu.edu.ua/data/kafedra/internal/xirtop/zbtest... · Web viewModule 1. Operative surgery and topographical anatomy of the head and neck, regions and organs of the thoracic

C. StomachD. Inferior vena cava *E. Transverse colon

900. The hepatic portal vein is correctly described by each of the following except:A. It drains most of the venous blood from the gastrointestinal tractB. It is fanned by the confluence of the superior mesenteric and splenic veinsC. It is located in the hepatoduodenal ligamentD. It forms posterior to the head of the pancreas *E. It forms the anterior border of the epiploic foramen

901. Posterior to the second part of the duodenum is the:A. Transverse colonB. Left lobe of the liverC. Hilum of the right kidney *D. Superior mesenteric arteryE. Gallbladder

902. Postganglionic fibers from the celiac plexus and ganglion distribute with the:A. Middle colic arteryB. Left colic arteryC. Inferior pancreaticoduodenal arteryD. Left gastric artery *E. Right colic artery

903. Part of the anterior wall of the omental bursa is formed by the:A. PancreasB. Lesser omentum *C. Lenorenal ligamentD. MesenteryE. Coronary ligament

904. The falciform ligament:A. Contains the ligamentum venosumB. Is located between the liver and the diaphragmC. Separates the caudate lobe from the quadrate lobe of the liverD. Contains the ligamentum teres of the liver *E. Contains the ductus venosum

905. All of the following statements correctly describe the deep inguinal ring except:A. It is a diverticulum of the transversalis fasciaB. It is immediately lateral to the inferior epigastric arteryC. It is the site of direct inguinal hernias *D. It transmits the round ligament of the uterusE. It is reinforced anteriorly by fibers of the external abdominal oblique and internal

abdominal oblique906. The vagus nerve provides parasympathetic innervation to the gut as far as the:

A. Duodenal-jejunal flexureB. Ileocecal junctionC. Right colic flexureD. Left colic flexure *E. Upper third of the rectum

907. Which of the following nerves traverses the superficial inguinal ring?A. IliohypogastricB. SubcostalC. Ilioinguinal *D. PudendalE. Obturator

908. The upper five slips of origin for the external abdominal oblique muscle interdigitate with which of the following muscles?

Page 112: 1intranet.tdmu.edu.ua/data/kafedra/internal/xirtop/zbtest... · Web viewModule 1. Operative surgery and topographical anatomy of the head and neck, regions and organs of the thoracic

A. Pectoralis majorB. Rectus abdominisC. Latissimus dorsiD. Serratus anterior *E. Subscapularis

909. Which of the following structures is not a specialization of the external oblique aponeurosis?

A. Inguinal ligamentB. Lacunar ligamentC. Intercrural fibersD. Internal spermatic fascia *E. Medial and lateral crura

910. The thickened, rolled-under portion of the external oblique aponeurosis, which is stretched between the anterior superior spine of the ilium and the pubic tubercle, is known as the:

A. Inguinal ligament *B. Lacunar ligamentC. Intercrural fibersD. Rectus sheathE. Linea alba

911. The lacunar ligament represents the more medial, rolled-under fibers of which of the following?

A. Medial cruraB. Reflected inguinal ligamentC. Fundiform ligamentD. Pectineal ligamentE. Inguinal ligament *

912. In the lower one fourth of the abdomen, the internal abdominal oblique aponeurosis does which of the following?

A. Splits to send one sheet anterior and one posterior to the rectus abdominis muscleB. Fails to split and passes to the median line entirely posterior to the rectus abdominis

muscle.C. Fails to split and passes to the median line entirely anterior to the rectus abdominis

muscle *D. DisappearsE. Gives rise to the internal spermatic fascia

913. The aponeurosis of the transversus abdominis muscle contributes to which of the following?

A. Inguinal ligamentB. Lacunar ligamentC. Falx inguinalis *D. External spermatic fasciaE. Superficial inguinal ring

914. Which of the following statements correctly applies to the tendinous intersections?A. They are firmly adherent to both layers of the rectus sheathB. The lowest is at the level of the symphysis pubisC. The highest is near the xiphoid process *D. The lowest is at the level of the arcuate lineE. There are usually ten tendinous intersections

915. Which of the following statements correctly applies to the pyramidalis muscle?A. It is a large, well-developed muscleB. It is always presentC. It is contained within the rectus sheath *D. It is posterior to the rectus abdominis muscle

Page 113: 1intranet.tdmu.edu.ua/data/kafedra/internal/xirtop/zbtest... · Web viewModule 1. Operative surgery and topographical anatomy of the head and neck, regions and organs of the thoracic

E. It is innervated by the tenth thoracic nerve916. The transversalis fascia contributes to which of the following structures?

A. Deep inguinal ring *B. Cremaster muscle and fasciaC. Inguinal ligamentD. Pectineal ligamentE. External spermatic fascia

917. The innervation of the muscles of the abdominal wall is provided by which of the following cord segments?

A. Thoracic 3 through sacral 3B. Thoracic 5 through lumbar 2C. Thoracic 1 through sacral 5-D. Thoracic 7 through lumbar 4 *E. Lumbar 1 through sacral 5

918. The tenth intercostal nerve enters the rectus sheath at the level of which of the following structures?

A. Xiphoid processB. Umbilicus *C. Pyramidalis muscleD. Pubic tubercleE. Symphysis pubis

919. The four lumbar arteries arise from which of the following vessels?A. Internal thoracicB. Internal iliacC. FemoralD. Aorta *E. External iliac

920. Which of the following structures does not traverse the inguinal canal?A. Inferior epigastric artery *B. Ilioinguinal nerveC. Genital branch of the genitofemoral nerveD. Cremasteric arteryE. Internal spermatic fascia

921. The esophagogastric junction is located at the level of which of the following vertebrae?

A. Seventh cervicalB. 11th thoracic *C. Second lumbarD. Fifth lumbarE. Sixth thoracic

922. Which of the following arteries is a branch of the celiac trunk?A. GastroduodenalB. Proper hepaticC. Common hepatic *D. Right gastroepiploicE. Cystic

923. The cystic artery arises from which of the following arteries?A. SplenicB. Left gastroepiploicC. Right gastricD. Right hepatic *E. Gastroduodenal

924. The left gastroepiploic artery arises from which of the following arteries?A. Left hepatic

Page 114: 1intranet.tdmu.edu.ua/data/kafedra/internal/xirtop/zbtest... · Web viewModule 1. Operative surgery and topographical anatomy of the head and neck, regions and organs of the thoracic

B. GastroduodenalC. Left gastricD. Common hepaticE. Splenic *

925. Which of the following statements correctly applies to the innervation of the stomach?

A. The sacral plexus provides the parasympathetic fibersB. The anterior vagal trunk carries preganglionic visceral efferents onlyC. The stomach has no sympathetic innervationD. The vagal trunks contain preganglionic visceral efferent and general visceral afferent

fibers *E. The postganglionic parasympathetic cell bodies are located in the celiac plexus

926. Which of the following statements correctly applies to the spleen?A. The sixth, seventh, and eighth ribs are in relation to the spleenB. The spleen is retroperitonealC. The spleen develops in the ventral mesogastriumD. The spleen normally descends below the costal marginE. The spleen rests on the left flexure of the colon *

927. Which of the following statements correctly applies to the first part of the duodenum?

A. It is surrounded by the hepatoduodenal ligament *B. It is related to the caudate lobe of the liverC. The common bile duct passes ventralD. It is located at the level of the third lumbar vertebraE. It has circular folds in its interior

928. The second portion of the duodenum is crossed by which of the following structures?A. Right renal arteryB. Transverse colon *C. Right ureterD. Portal veinE. Superior mesenteric vein

929. Which of the following arteries crosses the anterior aspect of the third part of the duodenum?

A. Proper hepaticB. Left colicC. Superior mesenteric *D. Inferior mesentericE. Splenic

930. Which of the following statements correctly applies to the greater duodenal papilla?A. The location for the terminal opening of the accessory pancreatic ductB. It is located in the interior of the third part of the duodenumC. It is continued below by the longitudinal fold of the duodenum *D. It is superior to the lesser duodenal papillaE. It opens into the duodenojejunal flexure

931. The lower left portion of the head of the pancreas is inserted behind which of the following arteries?

A. Left gastroepiploicB. Common hepaticC. Left colicD. Inferior mesentericE. Superior mesenteric *

932. The superior mesenteric and splenic veins unite to form the portal vein behind which of the following structures?

A. First part of the duodenum

Page 115: 1intranet.tdmu.edu.ua/data/kafedra/internal/xirtop/zbtest... · Web viewModule 1. Operative surgery and topographical anatomy of the head and neck, regions and organs of the thoracic

B. Transverse colonC. SpleenD. Neck of the pancreas *E. Duodenojejunal junction

933. The tail of the pancreas enters which of the following structures?A. Epiploic foramenB. Lienorenal ligament *C. Suspensory ligament of the duodenumD. Paracolic fossaE. Left coronary ligament

934. The pancreatica magna artery is a branch of which of the following arteries?A. Common hepaticB. Inferior mesentericC. Superior mesentericD. Left gastroepiploicE. Splenic *

935. Which of the following structures represents the obliterated remains of umbilical vein?

A. Ligamentum teres hepatis *B. Ligamentum venosumC. Ductus arteriosusD. Falciform ligamentE. Porta hepatis

936. The hepatoduodenal ligament transmits which of the following structures?A. Hepatic veinB. Main pancreatic ductC. Portal vein *D. Ligamentum venosumE. Superior mesenteric vein

937. The cystic artery usually arises from which of the following arteries?A. SplenicB. GastroduodenalC. Right gastroepiploicD. Right hepatic *E. Celiac trunk

938. The hepatic veins drain into which of the following veins?A. PortalB. CoronaryC. Inferior vena cava *D. Superior mesentericE. Splenic

939. The portal vein ascends to the liver in the free margin of which of the following structures?

A. MesocolonB. Greater omentumC. MesenteryD. Lesser omentum *E. Falciform ligament

940. The superior mesenteric artery supplies all of the small intestine except which of the following?

A. Proximal part of the duodenum *B. Duodenojejunal junctionC. Jejunoileal junctionD. Distal end of the ileum

Page 116: 1intranet.tdmu.edu.ua/data/kafedra/internal/xirtop/zbtest... · Web viewModule 1. Operative surgery and topographical anatomy of the head and neck, regions and organs of the thoracic

E. Descending portion of the duodenum941. Which of the following statements correctly apply to the middle colic artery?

A. It takes origin from the celiac trunkB. It supplies the cecumC. It anastomoses with the inferior pancreaticoduodenal arteryD. It is a branch of the superior mesenteric artery *E. It primarily supplies the left colic flexure

942. Both the vagal parasympathetic innervation and the thoracic splanchnic sympathetic innervation of the gastrointestinal tract terminate at which of the following?

A. Duodenojejunal junctionB. Junction of the middle and left thirds of the transverse colon *C. Jejunoileal junctionD. Ileocecal junctionE. Distal one third of the sigmoid colon

943. Epiploic appendages are located on which of the following structures?A. DuodenumB. StomachC. IleumD. JejunumE. Sigmoid *

944. Which of the following statements correctly applies to the vermiform appendix?A. It is usually retroperitonealB. It has a small mesentery *C. It receives its blood supply from the inferior mesenteric arteryD. It is usually located in a retrocecal positionE. It receives its innervation from the pelvic splanchnic nerve

945. The transverse mesocolon is attached posteriorly to which of the following structures?

A. Hepatoduodenal ligamentB. SpleenC. Second portion of the duodenum *D. Lesser omentumE. Gastrocolic ligament

946. The pampiniform plexus is located in which of the following locations?A. PancreasB. KidneyC. SpleenD. Inguinal canal *E. Liver

947. Which of the following layers of fasciae is associated with the diaphragm?A. AlarB. SuperficialC. Transversalis *D. BuccopharyngealE. Innominate

948. The right suprarenal vein drains into which of the following veins?A. Right renalB. Inferior mesentericC. Superior mesentericD. PortalE. Inferior vena cava *

949. Which of the following statements correctly applies to the sigmoid colon?A. It begins at the brim of the pelvis *B. It has no mesocolon

Page 117: 1intranet.tdmu.edu.ua/data/kafedra/internal/xirtop/zbtest... · Web viewModule 1. Operative surgery and topographical anatomy of the head and neck, regions and organs of the thoracic

C. It continues as the rectum at the level of the fifth sacral segmentD. It receives its blood supply from the left colic arteryE. It has no teniae coli

950. Which of the following statements correctly applies to the small intestine?A. The upper three fifths is considered jejunumB. The lower three fifths contains aggregated lymph nodules *C. None of it is retroperitonealD. The parasympathetic innervation is provided by the pelvic splanchnic nervesE. The blood supply is provided by both the superior and inferior mesenteric arteries

951. Which of the following statements correctly applies to the gall bladder?A. The submucosal layer is well developedB. The mucous membrane is thrown into circular foldsC. Its epithelium concentrates the contents of the gall bladder *D. It produces bileE. It lies to the left of the falciform ligament

952. Which of the following statements correctly applies to the falciform ligament?A. It represents the inferior limit of the common mesenteryB. It encloses the ligamentum teres of the liverC. It extends from the umbilicus to the liver *D. It contains the common bile ductE. It does not extend over the diaphragmatic surface of the liver

953. The anterior surface of the liver lies against all of the following structures except:A. DiaphragmB. Costal marginC. Xiphoid processD. Abdominal wallE. Spleen *

954. Which of the following structures is situated between the celiac trunk and the superior mesenteric artery?

A. Duodenum and pancreas *B. Spleen and stomachC. Transverse colon and ileumD. Stomach and cecumE. Pancreas and jejunum

955. Which of the following statements correctly applies to the pancreas?A. It extends from the right kidney to the spleenB. It is inferior to the stomachC. It is crossed by the transverse mesocolon *D. It overlies the fourth lumbar vertebraE. Its uncinate process extends behind the inferior mesenteric vessels

956. Which of the following statements correctly apply to the fourth portion of the duodenum?

A. It is located at the level of the first lumbar vertebraB. It is entirely retroperitonealC. The root of the mesentery begins at the duodenojejunal flexure *D. It is in direct continuity with the pylorus of the stomach E. It overlies the hilum of the right kidney

957. All of the following statements correctly apply to the duodenum except which?A. It is the shortest portion of the small intestineB. It is usually the breadth of twelve fingersC. It is the fixed portion of the small intestineD. It is suspended by a mesentery *E. It is the first portion of the small intestine

Page 118: 1intranet.tdmu.edu.ua/data/kafedra/internal/xirtop/zbtest... · Web viewModule 1. Operative surgery and topographical anatomy of the head and neck, regions and organs of the thoracic

958. Which of the following statements correctly applies to the innervation of the stomach?

A. The parasympathetic innervation enhances muscular movement *B. The sympathetic innervation exerts greater influence on the secretion of waterC. The sympathetic innervation exerts greater influence on the secretion of hydrochloric

acidD. The parasympathetic innervation has the major influence in the secretion of enzymesE. Afferents principally accompany the parasympathetic system

959. The inferior boundary of the epiploic foramen is which of the following structures? A. Inferior vena cavaB. Hepatoduodenal ligamentC. Caudate lobe of the liverD. Lesser omentumE. First part of the duodenum *

960. The lesser omentum includes which of the following ligaments?A. PhrenicocolicB. CoronaryC. Hepatogastric *D. GastrocolicE. Gastrolienal

961. The lesser peritoneal sac is closed off from the greater peritoneal sac except for the communication through which of the following?

A. Aortic hiatusB. Esophageal hiatusC. Caval foramenD. Deep inguinal ringE. Epiploic foramen *

962. Meckel's diverticulum is an occasional feature of which of the following structures?A. DuodenumB. CecumC. Ileum *D. JejunumE. Liver

963. The transpyloric line is located at the level of which of the following vertebrae?A. Fourth lumbarB. Second sacralC. First lumbar *D. Fifth thoracicE. Tenth thoracic

964. The upper one third of the free intestine occupies which of the following areas of the abdomen?

A. Lower left quadrantB. Lower right quadrantC. Upper left quadrant *D. Upper right quadrantE. Middle of the abdominal cavity

965. Components of the superior hypogastric plexus include all but which one of the following?

A. Preganglionic sympathetic fibersB. Postganglionic sympathetic fibersC. Visceral afferents for painD. Visceral efferents from T10-L2E. Parasympathetic fibers *

966. The ovarian artery is a branch of the:

Page 119: 1intranet.tdmu.edu.ua/data/kafedra/internal/xirtop/zbtest... · Web viewModule 1. Operative surgery and topographical anatomy of the head and neck, regions and organs of the thoracic

A. Internal iliacB. External iliacC. Inferior epigastricD. Abdominal aorta *E. Uterine artery

967. Which of these statements describes the superior hypogastric plexus correctly?A. It lies above the bifurcation of the aortaB. It contains mostly parasympathetic nervesC. It is a downward prolongation of the preaortic plexus *D. It is joined by the first and second lumbar splanchnic nervesE. Its large branches do not contribute to the pelvic plexus

968. Which of the following statements about the umbilical vein is correct?A. It closes immediately at the time of birthB. The obliterated vein becomes the ligamentum teres of the liver *C. The obliterated vein becomes the falciform ligamentD. The obliterated vein becomes the median umbilical ligamentE. It drains into the common iliac vein

969. A 55 year old woman dies of septicemia after going to the emergency department with severe abdominal pain. She lost 15 kg in the past 2 months and complained of severe pain after meals. Postmortem findings including severe aortic atherosclerosis and a segment of necrotic bowel extending from the splenic flexure to the sigmoid colon. A blockage in which of the following arteries would best support these findings?

A. Common iliac arteriesB. Common hepatic arteryC. Superior mesenteric arteryD. Inferior mesenteric artery *E. Splenic artery

970. A patient is diagnosed as having intestinal angina (thrombotic occlusion of the superior mesenteric artery near its origin). If significant collateral circulation is not present or fails to develop, all the following parts of the intestinal tract may become gangrenous except the:

A. Ascending colonB. AppendixC. Descending colon *D. IleumE. Transverse colon

971. If the inferior mesenteric vein was ligated (tied off), blood could still reach the liver because of anastomoses between the:

A. Splenic and inferior mesenteric veinsB. Left colic and middle colic veins *C. Right colic and middle colic veinsD. Ileocolic and right colic veinsE. Left gastroepiploic and splenic veins

972. Each of the following statements concerning inguinal hernia is correct except:A. It is more common than femoral herniaB. An indirect hernia passes through the deep inguinal ringC. There are two types of hernia, indirect and directD. The deep inguinal ring is medial to the inferior epigastric artery *E. In the direct type of inguinal hernia, the inferior epigastric artery is lateral to the

herniating mass973. Herniation of abdominal contents into an unobliterated vaginal process and within

the coverings of the spermatic cord results in which of the following hernias?A. UmbilicalB. Indirect inguinal *

Page 120: 1intranet.tdmu.edu.ua/data/kafedra/internal/xirtop/zbtest... · Web viewModule 1. Operative surgery and topographical anatomy of the head and neck, regions and organs of the thoracic

C. Direct inguinalD. LumbarE. Femoral

974. The inferior epigastric artery is lateral to the herniating mass in which of the following hernia?

A. Direct inguinal *B. Indirect inguinalC. FemoralD. UmbilicalE. Lumbar

975. Which of the following nerves innervates the muscles of the anterior abdominal wall:A. Intercostals *B. PhrenicC. VagusD. PudendalE. Femoral

976. Which of the following anatomical structures forms the inferior wall of the inguinal canal:

A. Lacunar ligamentB. Pectineal ligamentC. Linea alba baseD. Inguinal ligament *E. Inferior edge of transverse muscle

977. The boundary of the external foramen of the inguinal canal is the:A. Aponeurosis of abdominal external oblique muscle *B. Internal abdominal oblique muscle C. Transverse muscle of abdomenD. Linea alba E. Duglas' line

978. Which of the following anatomical structures forms anterior wall of the rectus muscle sheath on ribs level:

A. Aponeurosis of abdominal external oblique muscle B. Aponeurosis of abdominal internal oblique muscle C. Aponeurosis of transverse muscle of abdomenD. Cartilages of V-VII ribs *E. Pyramidal muscle

979. Lateral umbilical fold is formed by which of the following arteries:A. Umbilical *B. IntercostalC. Superior epigastricD. Inferior epigastricE. Cremaster

980. Between which of the following anatomical structures of the anterior abdominal wall pass intercostals nerves:

A. Abdominal external oblique muscle and internal oblique abdominal muscleB. Abdominal internal oblique muscle and transverse muscle of abdomen*C. Superficial fascia and proper fasciaD. Endoabdominal fascia and parietal peritoneumE. Rectus muscle and pyramidal muscle

981. Which of the following anatomical structures forms the posterior wall of the inguinal canal:

A. Transverse fascia *B. Aponeurosis of abdominal internal oblique muscle C. Aponeurosis of transverse muscle of abdomen

Page 121: 1intranet.tdmu.edu.ua/data/kafedra/internal/xirtop/zbtest... · Web viewModule 1. Operative surgery and topographical anatomy of the head and neck, regions and organs of the thoracic

D. Rectus muscle E. Linea alba

982. Hernial gate in case of direct inguinal hernia:A. Inguinal space *B. Internal foramen of inguinal canalC. External foramen of inguinal canalD. Midline defectsE. Supravesical fossa

983. Aponeurosis which of the following muscles forms the inguinal ligament:A. Abdominal external oblique muscle *B. Abdominal internal oblique muscle C. Transverse muscle of abdomenD. Rectus muscle E. Pyramidal muscle

984. Which of the following anatomical structures forms posterior wall of the rectus muscle sheath below the umbilicus:

A. Transverse fascia *B. Aponeurosis of abdominal external oblique muscle C. Aponeurosis of abdominal internal oblique muscle D. Aponeurosis of transverse muscle of abdomenE. Cartilages of V-VII ribs

985. The boundaries of the supravesical fossa is formed by the:A. Middle umbilical fold and medial umbilical fossa *B. Medial and lateral umbilical fossaC. Medial inguinal fold and epigastric vesselsD. Rectus muscle and Shpigel's lineE. Rectus muscle and medial inguinal fold

986. The superior boundary of the inguinal region:A. Lin. spinarum *B. Lin. costarumC. Lin. albaD. Lin. spinoumbilicalisE. Lin. pararectalis

987. Hernial gate in case of umbilical hernia:A. Alba line defectsB. Spigel's lune defectsC. Umbilical ring *D. Inguinal spaceE. Medial fossa

988. Which of the following anatomical structures forms anterior wall of the rectus muscle sheath in lower part:

A. Aponeuroses of abdominal wide muscles *B. Transverse fascia C. Cartilages of V-VII ribs D. Pyramidal muscleE. Parietal peritoneum

989. The boundaries of medial inguinal fossa are formed by which of the following anatomical structures:

A. Lateral umbilical fold and epigastric vesselsB. Medial and lateral umbilical folds *C. Lateral inguinal fossa and pyramidal muscleD. Middle and medial umbilical foldsE. Rectus muscle and pyramidal muscle

990. On which level the alba line is widest:

Page 122: 1intranet.tdmu.edu.ua/data/kafedra/internal/xirtop/zbtest... · Web viewModule 1. Operative surgery and topographical anatomy of the head and neck, regions and organs of the thoracic

A. Near the xyphoid processB. On 2-3 sm lower than umbilicusC. On 3-4 sm upper than umbilicusD. On umbilicus level *E. Near the symphysis

991. Which of the following anatomical structures forms posterior wall of the rectus muscle sheath in lower part:

A. Transverse fascia *B. Aponeurosis of abdominal external oblique muscle C. Aponeurosis of abdominal internal oblique muscleD. Cartilages of V-VII ribsE. Superficial lamina of aponeurosis of abdominal internal oblique muscle

992. Tendon of which of the following abdominal muscles forms the Spigel’s line:A. External obliqueB. Internal obliqueC. Transverse *D. RectusE. Pyramidal

993. Which of the following veins connects the subcutaneous veins of umbilical region with the portal vein:

A. Paraumbilical veins *B. Hepatic veinsC. Gastric veinsD. Superior epigastric veinE. Inferior epigastric vein

994. Hernial gate in case of congenital inguinal hernia:A. Femoral ringB. External ring of inguinal canalC. Inguinal spaceD. Internal ring of inguinal canal *E. Alba line defects

995. Sliding hernia differ from the ordinary hernia by which of the following components:A. Hernial fluidB. Hernial gateC. One wall of the hernial sac is formed by hollow organ wall *D. Signs of tunics inflammation E. hernial contents

996. What is paramedian incision of the anterior abdominal wall:A. Along medial edge of the left abdominal rectus muscle *B. Along lateral edge of the left abdominal rectus muscleC. Along medial edge of the right abdominal rectus muscleD. Along lateral edge of the right abdominal rectus muscleE. Along alba line above the umbilicus

997. What is transrectal incision of the anterior abdominal wall:A. Along lateral edge of the right abdominal rectus muscleB. Through the abdominal rectus muscle *C. Along alba line above the umbilicusD. Along alba line below the umbilicusE. Along alba line above the umbilicus

998. What is combined incisions:A. Laparotomy with thoracotomy *B. Transverse incisions with oblique incisions of the abdominal wallC. Two different incisions of the abdominal wallD. Oblique incisions with longitudinal incisions of the abdominal wall

Page 123: 1intranet.tdmu.edu.ua/data/kafedra/internal/xirtop/zbtest... · Web viewModule 1. Operative surgery and topographical anatomy of the head and neck, regions and organs of the thoracic

E. None of the above999. Gaster divides on which of the following parts:

A. 5 *B. 3C. 4D. 2E. 6

1000. Gaster is innervate by:A. Right vagus nerve, celiac plexusB. Left vagus nerve, celiac plexusC. Left and right vagus nervesD. Celiac plexusE. Vagus nerve, celiac plexus *

1001. During 24 hours liver produces :A. 1 liter of bile *B. 2 liters of bile C. 1,5-2 liters of bile D. 0.5 liter of bile E. 300-400 ml of bile

1002. Gall bladder consists of which of the following parts:A. Fundus, bodyB. Fundus, body, neck, ductusC. Neck, ductusD. Fundus, neck, ductusE. Fundus, body, neck *

1003. External ring of inguinal canal is formed by which of the following anatomical structures:

A. Aponeurosis of transverse muscle of abdomenB. Aponeurosis of abdominal external oblique muscle *C. Aponeurosis of abdominal internal oblique muscleD. Alba lineE. Pyramidal muscle

1004. The external boundary of umbilical region:A. Lateral edge of rectus muscle *B. Medial edge of rectus muscle C. Spigel's lineD. Duglas' lineE. Alba line

1005. The inferior boundary of inguinal region:A. Alba lineB. External edge of rectus muscleC. Internal edge of rectus muscleD. Inguinal ligament *E. Pyramidal muscle

1006. Hernial sac in case of anterior abdominal wall hernias is formed by the:A. Visceral peritoneumB. Parietal peritoneum *C. Aponeurosis of abdominal external oblique muscle D. Superficial fasciaE. Proper fascia

1007. Which of the following hernias frequently can be incarceretive in female:A. UmbilicalB. InguinalC. Femoral *

Page 124: 1intranet.tdmu.edu.ua/data/kafedra/internal/xirtop/zbtest... · Web viewModule 1. Operative surgery and topographical anatomy of the head and neck, regions and organs of the thoracic

D. MidlineE. Obturator

1008. The boundaries of lateral inguinal fossa are formed by which of the following anatomical structures:

A. Lateral umbilical fold and inguinal ligament *B. Medial and lateral umbilical foldsC. Inguinal ligament and pyramidal muscleD. Middle and medial umbilical foldsE. Rectus muscle and pyramidal muscle

1009. Which of the following anatomical structures locates in umbilical region:A. Lig. teres hepatis *B. Spigel's lineC. Duglas' lineD. Superior epigastric arteryE. None of the above

1010. Through which of the following anatomical structures on internal surface of the anterior abdominal wall passes hernial sac in case of indirect inguinal hernia:

A. Supravesical fossaB. Medial inguinal fossa *C. Lateral inguinal fossaD. Inguinal spaceE. None of the above

1011. Which of the following anatomical structures passes through the inguinal canal in male:

A. Testicular artery *B. Obturator arteryC. Pudendal arteryD. Femoral nerveE. Sciatic nerve

1012. Which of the following anatomical structures passes through the inguinal canal in female:

A. Femoral nerveB. Sciatic nerveC. Round ligament of uterus *D. Lacunar ligamentE. Wide ligament of uterus

1013. Which of the following anatomical structures passes through the inguinal canal in female:

A. Vagus nerveB. Sciatic nerveC. Femoral nerveD. Ilioinguinal nerve *E. None of the above

1014. The peritoneum is stitched by the:A. Continuous catgut suture together with the transverse fascia *B. Interrupted silk suture together with the transverse fascia C. Continuous catgut suture D. Continuous silk suture together with the transverse fascia E. Interrupted silk suture

1015. What is stomy:A. Incision of an organ cavityB. Connection of an organ cavity with another organ cavityC. Connection of an organ cavity with outdoor environment *D. Excision of an organ part

Page 125: 1intranet.tdmu.edu.ua/data/kafedra/internal/xirtop/zbtest... · Web viewModule 1. Operative surgery and topographical anatomy of the head and neck, regions and organs of the thoracic

E. Stitching of a hollow organ wound1016. What is Lamber's suture:

A. Blanket sutureB. "Danger" intestinal sutureC. Sero-muscular intestinal suture *D. InternalE. Continuous suture

1017. Czerny's suture lies through which of the following wall layers:A. Serous, muscular, submucous, mucous *B. Serous, muscularC. Muscular, submucous, mucous D. Submucous, mucous E. Serous

1018. Length of the abdominal part of esophagus:A. 2 cmB. 1-1,5 cmC. 1,5-2 cm *D. 2-2,5 cmE. 3 cm

1019. Venous drainage from the abdominal part of esophagus:A. Left gastric vein *B. Right gastric veinC. Portal veinD. Phrenic veinsE. Hemiazygos vein

1020. Which of the following gaster parts produces the hydrochloric acid:A. BodyB. FundusC. Body, fundus *D. All partsE. Pyloric part

1021. Celiac trunk divides on which of the following arteries:A. Splenic, common hepaticB. Left and right gastric, hepaticC. Left and right gastric, common hepatic, splenicD. Left gastric, splenic, common hepatic *E. Left and right gastric, splenic

1022. Common hepatic artery divides on which of the following vessels:A. Left and right hepaticB. Proper hepatic, gastoduodenal *C. Left and right hepatic, cysticD. Left and right hepatic, gastoduodenal E. Proper hepatic, gastric, duodenal

1023. What is Gartman's recess:A. Dilatation of gall bladder neck *B. Dilatation of gall bladder ductusC. Dilatation of gall bladder fundusD. Dilatation of all parts of gall bladderE. None of the above

1024. Pancreas divides on which of the following parts:A. Head, fundus, bodyB. Head, body, tail *C. Head, neck, bodyD. Head, neck, fundus

Page 126: 1intranet.tdmu.edu.ua/data/kafedra/internal/xirtop/zbtest... · Web viewModule 1. Operative surgery and topographical anatomy of the head and neck, regions and organs of the thoracic

E. Head, neck, tail 1025. Splenic artery arises from the:

A. Celiac trunk *B. Abdominal aortaC. Left gastric arteryD. Right gastric arteryE. Common hepatic artery

1026. Sources of arterial supply of duodenum:A. Celiac trunk and superior mesenteric artery *B. Celiac trunk C. Superior mesenteric artery D. Superior and inferior mesenteric arteriesE. Short gastric branches

1027. Small intestine consists of:A. 3 parts *B. 2 parts C. 4 parts D. 2-3 parts E. 1 parts

1028. Arterial supply of jejunum and ileum:A. Superior mesenteric artery, celiac trunkB. Inferior mesenteric arteryC. Superior mesenteric artery *D. Celiac trunkE. Inferior mesenteric artery, celiac trunk

1029. How many teniae has the large intestine:A. 1B. 2C. 3 *D. 4E. No one

1030. Peritoneum attitude to the ascending colon:A. Mesoperitoneal *B. IntraperitonealC. RetroperitonealD. In retroperitoneal spaceE. In accoding to filling

1031. Peritoneum attitude to the descending colon:A. Mesoperitoneal *B. IntraperitonealC. RetroperitonealD. In retroperitoneal spaceE. In accoding to filling

1032. Peritoneum attitude to the transverse colon:A. MesoperitonealB. Intraperitoneal *C. RetroperitonealD. In retroperitoneal spaceE. In accoding to filling

1033. Peritoneum attitude to the sigmoid colon:A. MesoperitonealB. RetroperitonealC. Intraperitoneal *D. In retroperitoneal space

Page 127: 1intranet.tdmu.edu.ua/data/kafedra/internal/xirtop/zbtest... · Web viewModule 1. Operative surgery and topographical anatomy of the head and neck, regions and organs of the thoracic

E. In accoding to filling1034. Sources of arterial supply of large intestine:

A. Superior and inferior mesenteric arteries *B. Celiac trunk and superior mesenteric arteryC. Celiac trunk D. Celiac trunk and inferior mesenteric arteryE. Superior and inferior mesenteric arteries, celiac trunk

1035. Superior mesenteric artery divides on the:A. Right colic, middle colic B. Iliocolic, right colicC. Iliocolic, right colic, middle colic *D. Ilial, colic, middle colicE. Left and right colic

1036. Inferior mesenteric artery divides on the:A. Left colic, sigmoid, superior rectal *B. Iliocolic, right colic, middle colic C. Right colic, middle colic D. Ilial, colic, middle colicE. Left colic, middle colic

1037. What is Riolan's arch:A. Anastomosis between the superior and inferior mesenteric arteries *B. Anastomosis between the right and middle colic arteries C. Anastomosis between the ilial and colic arteriesD. Anastomosis between the celiac trunk and superior mesenteric artery E. Anastomosis between the celiac trunk and inferior mesenteric artery

1038. Congenital inguinal hernias can be:A. SlidingB. IrreducibleC. Indirect and directD. Indirect *E. Direct

1039. What is hernioplasty in case of indirect inguinal hernias:A. Strengthening of the posterior wall of inguinal canalB. Strengthening of the anterior wall of inguinal canal *C. Strengthening of the posterior and anterior walls of inguinal canalD. Strengthening of the medial inguinal fossaE. Strengthening of the lateral inguinal fossa

1040. What is vagotomy:A. Transection of the vagus nerve *B. Transection of the celiac plexus branchesC. Stitching of the vagus nerveD. Reinnervation by the vagus nerve branchesE. Anchoring of the vagus nerve

1041. Which types of vagotomy do you know:A. Left and right B. TotalC. Left and right selectiveD. Stem, selective, selective proximal *E. All of the above

1042. Gaster divides into which of the following parts:A. 5 *B. 3C. 4D. 2

Page 128: 1intranet.tdmu.edu.ua/data/kafedra/internal/xirtop/zbtest... · Web viewModule 1. Operative surgery and topographical anatomy of the head and neck, regions and organs of the thoracic

E. 61043. Nothing opens into the peritoneal cavity except the:A. Epiploic foramenB. UrachusC. Uterine tube *D. Greater omentumE. Mesonephric tubules1044. Which of the following statements concerning the lesser omentum is true?A. It extends from the greater curvature of the stomach to the duodenumB. The cystic duct or common bile duct runs in its free edge *C. It contains the celiac arteryD. It contains the anterior and posterior vagal trunkE. None of the above1045. After an incision through the ventral abdominal wall, the lesser peritoneal sac could

be exposed by incising any of the following except the:A. Phrenicocolic ligament *B. Gastrohepatic ligamentC. Gastrocolic ligamentD. Gastrolienal ligamentE. Transverse mesocolon1046. Which of the following will be posterior to a finger in the epiploic foramen?A. The inferior vena cava *B. The aortaC. The portal veinD. The hepatic veinE. The hepatic artery1047. Surgically, it is possible to gain entry to the lesser sac of peritoneum in all but which

one of the ways listed below?A. Incise the lesser omentumB. Incise the greater omentum superior to the transverse colonC. Incise the greater omentum inferior to the transverse colonD. Incise the transverse mesocolonE. All of the above incisions grant entry into the lesser sac *1048. A surgeon divides the gastrocolic ligament in order to reach the lesser peritoneal sac.

This will give him access to the:A. Right ureterB. Origin of the inferior mesenteric arteryC. Termination of the right colic veinsD. Splenic artery *E. None of the above1049. Which of the following structures is considered to be retroperitoneal?A. The duodenojejunal junctionB. The transverse colonC. The appendixD. The sigmoid colonE. The rectum *1050. In which of the following subphrenic spaces would you first look for an abscess

resulting from a perforation of an ulcer on the posterior wall of the stomach?A. Right anteriorB. Left anteriorC. Right posteriorD. Left posteriorE. Superior recess of lesser sac *

Page 129: 1intranet.tdmu.edu.ua/data/kafedra/internal/xirtop/zbtest... · Web viewModule 1. Operative surgery and topographical anatomy of the head and neck, regions and organs of the thoracic

1051. If you put your finger into the epiploic foramen, the structures crossing it anteriorly should include the:

A. Common bile ductB. Hepatic arteryC. Portal veinD. None of the aboveE. All of the above *1052. The lesser omentum contains the:A. Hepatic arteryB. Common bile ductC. Portal veinD. Right gastric arteryE. All of the above are true *1053. Which of the following is not in contact with the spleen?A. The liver *B. The stomachC. The pancreasD. The left kidneyE. The colon1054. During a splenectomy (removal of the spleen) which of the following structures is

most likely to be encountered?A. The left suprarenalB. The pancreas *C. The left ureterD. The inferior mesenteric veinE. The superior mesenteric vein1055. The transpyloric plane is represented by a horizontal line:A. Midway between the suprasternal notch and the pubic symphysis *B. Midway between the xiphoid and the pubic symphysisC. Through the lower costal marginD. At the level of the umbilicusE. Through the most superior part of the iliac crest1056. Which of the following accompanies the esophagus through the esophageal hiatus of

the diaphragm?A. The phrenic nervesB. The vagus nerves *C. The azygos veinD. The hemiazygos veinE. The cisterna chili1057. Which of the following structures is not related to the posterior surface of the

stomach?A. The pancreasB. The splenic artery *C. The transverse colonD. The duodenojejunal junctionE. The lesser sac of peritoneum1058. Which of the following is not related to some part of the posterior wall of the

stomach?A. The diaphragmB. The splenic arteryC. The left kidneyD. The suprarenal glandE. The left gastric artery *1059. Which of the following structures is not closely related to the gall bladder?

Page 130: 1intranet.tdmu.edu.ua/data/kafedra/internal/xirtop/zbtest... · Web viewModule 1. Operative surgery and topographical anatomy of the head and neck, regions and organs of the thoracic

A. The anterior abdominal wallB. The colonC. The duodenumD. The right suprarenal gland *E. All of the above are closely related1060. In the course of a cholecystectomy, a surgeon encounters brisk hemorrhage. He

panics and uses a large hemostat blindly. Structures endangered by this clamp include all but which one of the following?

A. The common bile ductB. The common hepatic ductC. The pancreatic duct *D. The hepatic arteryE. The portal vein1061. All of the following help to indicate the position of the gall bladder except the;A. Ninth ribB. Infracostal plane *C. Transpyloric planeD. Right linea semilunarisE. None of the above help indicate the position of the gallbladder1062. All but which one of the following is in contact with the normal liver?A. The stomachB. The right colic flexureC. The left colic flexure *D. The right kidneyE. The duodenum1063. A patient with obstructive jaundice is found to have dilated common hepatic duct,

gall bladder and common bile ducts. Which is the least likely cause of the obstruction?A. Carcinoma of the duodenal papillaB. Carcinoma of the head of the pancreasC. Calculus in the common bile ductD. Enlarged nodes in the porta hepaticE. Any of the above is likely to cause this syndrome *1064. Posterior extension of a cancer of the body of the pancreas may involve the:A. AortaB. Left suprarenalC. Splenic veinD. Aorta and splenic vein *E. Aorta and inferior vena cava1065. A carcinoma of the head of the pancreas may extend posteriorly and involve the:A. Common bile ductB. Sympathetic trunkC. Inferior vena cavaD. Common bile duct and inferior vena cava *E. Sympathetic trunk and inferior vena cava1066. The pancreas:A. Lies transversely across the posterior abdominal wall from the duodenum to the spleenB. Lies behind the stomachC. Has a main and an accessory ductD. Receives vessels from both the celiac and superior mesenteric arteriesE. All of the above are correct *1067. If the common bile duct were obstructed by a stone just below the termination of the

cystic duct, the:A. Pancreatic ducts would be blockedB. Gallbladder might be distended

Page 131: 1intranet.tdmu.edu.ua/data/kafedra/internal/xirtop/zbtest... · Web viewModule 1. Operative surgery and topographical anatomy of the head and neck, regions and organs of the thoracic

C. Biliary system would be obstructedD. A and BE. A and C *1068. The viscera in contact with the surface of the right lobe of the liver include the:A. ColonB. StomachC. Right kidneyD. Colon and right kidney *E. Colon and stomach1069. Which of the following are true of the bare area of the liver?A. The inferior vena cava is closely related to the bare area of the liverB. The bare area is bounded above by the anterior (superior) coronary ligamentC. The bare area is bounded below by the posterior (inferior) coronary ligamentD. The bare area is bounded to the left by the groove for the ligamentum venosumE. All are correct *1070. The lymphatic drainage of the stomach tends to accompany its arterial blood supply.

Where would you not expect gastric lymph to pass?A. Nodes at the hilus of the spleenB. Nodes along the hepatic arteryC. The celiac nodesD. Nodes along the upper border of the pancreasE. Nodes behind the stomach at the origin of the jejunum *1071. The lymphatic drainage from the pyloric end of the greater curvature of the stomach

is to the:A. Left gastric lymph nodesB. Right gastric lymph nodesC. Gastroduodenal or gastroepiploic lymph nodes *D. Pancreaticolienal lymph nodesE. None of the above1072. The coeliac artery contributes to the blood supply of all of the following except the:A. First part of the duodenumB. Third part of the duodenum *C. Gall bladderD. SpleenE. Stomach1073. Which of the following arteries is not derived from the hepatic artery (either directly

or from one of its branches)?A. The gastroduodenal arteryB. The right gastric arteryC. The right gastroepiploic arteryD. The superior pancreaticoduodenal arteryE. The short gastric arteries *1074. All of the following veins are tributaries of the portal vein except the:A. Superior mesentericB. SplenicC. Hepatic *D. Inferior mesentericE. Gastric1075. All of the following arteries contribute to the blood supply of the stomach except the:A. Left gastric arteryB. Splenic arteryC. Hepatic arteryD. Inferior pancreaticoduodenal artery *E. All of the above contribute to the blood supply of the stomach

Page 132: 1intranet.tdmu.edu.ua/data/kafedra/internal/xirtop/zbtest... · Web viewModule 1. Operative surgery and topographical anatomy of the head and neck, regions and organs of the thoracic

1076. The cystic artery is usually a branch of the:A. Right hepatic artery *B. Left hepatic arteryC. Common hepatic arteryD. Proper hepatic arteryE. Gastroduodenal artery1077. Which of the following statement(s) are true?A. The splenic vein drains into the portal vein posterior to the pancreasB. The portal vein passes up in the free edge of the lesser omentum posterior to the common

bile ductC. The portal vein has no valvesD. In the embryo, the portal vein communicates with the hepatic veinsE. All of the above are correct *1078. The portal vein is conventionally formed by major contributions of the:A. Superior mesenteric and splenic veins *B. Renal and gastric veinsC. Renal and splenic veinsD. Superior mesenteric and gastric veinsE. Gastric and splenic veins1079. All but which one of the following plexuses in the abdomen is fed by the splanchnic

nerve from the thoracic sympathetic trunk?A. The left gastric plexusB. The hypogastric plexus *C. The superior mesenteric plexusD. The renal plexusE. The celiac plexus1080. Which of the following statements about the portal vein is untrue?A. It lies near the free edge of the hepatoduodenal ligamentB. It is formed by the junction of the superior mesenteric and splenic veinsC. It lies posterior to the common bile ductD. It lies posterior to the hepatic arteryE. All of the above are true *1081. A surgeon wishes to anastomose the splenic and renal veins to treat portal venous

hypertension. After removing the spleen, the free end of the splenic vein is to be found in the:

A. Gastrophrenic ligamentB. Lienorenal ligament *C. Gastrosplenic ligamentD. Left edge of the lesser omentumE. None of the above places

1082. Advanced cirrhosis of the liver often results in portal hypertension. Which of the following signs is not directly related to portal hypertension?

A. Gastroesophageal varicesB. Hemorrhoids (varicose veins in the rectum)C. Caput medusae (dilation of the superficial umbilical veins)D. Ascites (accumulation of fluid in the abdominal cavity)E. Varicocele (varicose veins of the pampiniform plexus) *

1083. If the inferior mesenteric artery were clamped off, which of the following structures would have its blood supply endangered?

A. The descending colon *B. The small intestineC. The ascending colonD. The appendixE. The cecum

Page 133: 1intranet.tdmu.edu.ua/data/kafedra/internal/xirtop/zbtest... · Web viewModule 1. Operative surgery and topographical anatomy of the head and neck, regions and organs of the thoracic

1084. Venous drainage from which of the following enters the liver via the portal veins?A. AppendixB. Sigmoid colonC. PancreasD. SpleenE. All of these *1085. Which of the following statements about the superior mesenteric artery is usually not

true?A. It arises from the anterior aspect of the aortaB. It passes posterior to the neck of the pancreasC. It passes anterior to the inferior part of the duodenumD. It gives rise to the superior pancreaticoduodenal artery *E. It gives rise to the middle colic artery1086. The inferior mesenteric vein:A. Drains into the portal systemB. Receives the superior rectal veinC. Drains the sigmoid colonD. Drains the descending colonE. All of the above *1087. The left colic artery:A. Arises from the superior mesenteric arteryB. Arises from the inferior mesenteric artery *C. Anastomoses with the right colic arteryD. Supplies the hepatic flexureE. None of the above1088. The appendicular artery arises from the:A. Iliocolic artery and passes ventral to the terminal ileumB. Iliocolic artery and passes dorsal to the terminal ileum *C. Right colic artery and descends along the cecumD. Right colic artery and reaches the appendix through a peritoneal foldE. Inferior mesenteric artery1089. In carrying out a posterior gastrojejunostomy through the transverse mesocolon, you

must be careful to avoid injuring the:A. Right gastro-epiploic arteryB. Left gastro-epiploic arteryC. Right colic arteryD. Middle colic artery *E. None of the above1090. Arterial blood to the pancreas may by supplied from which of the following major

vessels?A. The splenic arteryB. The superior mesenteric arteryC. The common hepaticD. The gastroduodenal arteryE. All are correct *1091. The superior mesenteric artery supplies the:A. Distal duodenum *B. StomachC. LiverD. Head of the pancreasE. Abdominal part of the esophagus1092. The superior mesenteric artery supplies the:A. StomachB. Liver

Page 134: 1intranet.tdmu.edu.ua/data/kafedra/internal/xirtop/zbtest... · Web viewModule 1. Operative surgery and topographical anatomy of the head and neck, regions and organs of the thoracic

C. Ascending colon *D. PancreasE. Spleen1093. The superior mesenteric artery supplies the:A. Stomach B. Cecum *C. LiverD. KidneyE. Abdominal part of the esophagus1094. The left kidney is related to all of the following except the:A. Liver *B. Suprarenal glandC. PancreasD. SpleenE. Colon1095. The right kidney is related to all of the following except the:A. LiverB. Suprarenal glandC. Pancreas *D. DuodenumE. Colon1096. An invasive tumor of the right suprarenal gland may invade all but which one of the

following?A. Inferior vena cavaB. Right kidneyC. DiaphragmD. Gall bladder *E. Liver1097. The sigmoid colon gains its parasympathetic nerve supply from the:A. VagusB. Celiac ganglionC. Greater splanchnic nervesD. Pelvic splanchnic nerves *E. It has no parasympathetic nerve supply1098. The ascending colon gains its parasympathetic nerve supply from the:A. Vagus nerve *B. Celiac ganglionC. Greater splanchnic nervesD. Pelvic splanchnic nervesE. It has no parasympathetic nerve supply1099. Carcinoma of the lower end of the esophagus affecting both vagus nerves might

cause serious disorders of gut motility as far as the:A. Meckel’s diverticulum if presentB. Ileocaecal junctionC. Hepatic flexure of the colonD. Splenic flexure of the colon *E. Sigmoid colon1100. The point of greatest tenderness in acute appendicitis is usually:A. One-third the way along a line from the anterior superior ileac spine to the umbilicus *B. Half-way along a line from the anterior superior ileac spine to the umbilicusC. Two-thirds the way along a line from the anterior superior ileac spine to the umbilicusD. At the junction of the midclavicular line and a line from the anterior superior ileac spine to

the umbilicusE. One-third the way along a line from the anterior superior ileac spine to the pubic tubercle

Page 135: 1intranet.tdmu.edu.ua/data/kafedra/internal/xirtop/zbtest... · Web viewModule 1. Operative surgery and topographical anatomy of the head and neck, regions and organs of the thoracic

1101. A student examining a rather thin female patient was excited when he found a somewhat elongated palpable mass in the left lower quadrant of the abdomen. He has probably palpated:

A. An ovarian cystB. A mass in the ileumC. The inferior pole of the left kidneyD. A normal sigmoid colon *E. An enlarged lymph node1102. Which of the following structures a surgeon can expect to find behind the first part of

the duodenum?A. The portal veinB. The gastroduodenal arteryC. The common bile ductD. All of the above *E. Two of the above1103. In a normal individual which portion of the gut is susceptible to volvulus?A. The rectumB. The jejunum *C. The ascending colonD. The descending colonE. The second part of the duodenum1104. If Meckel’s diverticulum becomes inflamed, you might expect the pain to be referred

during early stages of the disease to the:A. Skin in the midline, halfway between the umbilicus and xiphoidB. Skin around the umbilicus *C. Skin in the midline, halfway between the umbilicus and pubisD. Skin of the right loinE. Wherever the diverticulum happened to be lying at that movement1105. The midgut extends between which of the following points?A. The pylorus of stomach to the ileocaecal valveB. The duodenal papilla to the middle of the transverse colon *C. The ileocaecal valve to the rectumD. The duodenal-jejunal flexure to the rectumE. None of the above1106. Peyer’s patches are most commonly found in the:A. Ileum *B. JejunumC. DuodenumD. CaecumE. Rectum1107. Which of the following abdominal viscera can normally be felt in a thin individual?A. The liverB. The spleenC. The pancreasD. The sigmoid colon *E. All of these1108. Which of the following statements about the Meckel’s diverticulum is untrue?A. It is present in a minority of adultsB. It is found at some point along the terminal one-fourth of the ileumC. It is found on the antimesenteric border (i.e., away from the mesenteric side) of the bowelD. Its blood supply is from the inferior mesenteric artery *E. Its venous drainage is through the portal venous circulation1109. Which of the following statements about the Meckel’s diverticulum is true?A. It is found at some point along the terminal one-fourth of the ileum

Page 136: 1intranet.tdmu.edu.ua/data/kafedra/internal/xirtop/zbtest... · Web viewModule 1. Operative surgery and topographical anatomy of the head and neck, regions and organs of the thoracic

B. It is found on the antimesenteric border (i.e., away from the mesenteric side) of the bowelC. Its blood supply is from the superior mesenteric arteryD. Its venous drainage is through the portal venous circulationE. All of the above statements are true *1110. You find a loop of small bowel protruding through a knife wound. You know it is the

jejunum rather than the ileum because:A. The jejunum has a thicker wallB. The jejunum is of larger diameterC. The jejunal mesentery has less fatD. The jejunal arterial arcades are simple or double with long vasa recti while the ileal arcades

are multiple with short vasa rectiE. All of the above are true *1111. A small segment of colon exposed through a limited surgical incision can be

distinguished from a segment of small bowel by:A. The presence of teniaeB. The identification of haustrationsC. The presence of epiploic appendagesD. All of the above *E. None of the above1112. A small segment of colon exposed through a limited surgical incision can be

distinguished from a segment of small bowel by:A. The presence of teniae *B. Its universally larger diameterC. The presence of arterial pulsationD. The presence of arterial arcadesE. None of the above1113. A small segment of colon exposed through a limited surgical incision can be

distinguished from a segment of small bowel by:A. The presence of arterial arcadesB. The presence of epiploic appendages *C. Its universally larger diameterD. The presence of arterial pulsationE. All of the above1114. A small segment of colon exposed through a limited surgical incision can be

distinguished from a segment of small bowel by:A. Its universally larger diameterB. The presence of arterial pulsationC. The presence of arterial arcadesD. The identification of haustrations *E. None of the above1115. Sigmoid colon:A. Extends from pelvic brim to S2B. Relates to both uretersC. Is longer in male than in femaleD. Indirectly drain into splenic vein *E. All of the above are true1116. Sigmoid colon:A. Cover entirely by peritoneum *B. Extends from pelvic brim to S2C. Relates to both uretersD. Is longer in male than in femaleE. All of the above are untrue1117. Sigmoid colon:A. Has wider tenia coli *

Page 137: 1intranet.tdmu.edu.ua/data/kafedra/internal/xirtop/zbtest... · Web viewModule 1. Operative surgery and topographical anatomy of the head and neck, regions and organs of the thoracic

B. Extends from pelvic brim to S2C. Relates to both uretersD. Is longer in male than in femaleE. All of the above are untrue1118. Appendix:A. Is longer in male than in femaleB. Has 2 bands of longitudinal muscleC. Has mesoappendix *D. Cover entirely by peritoneumE. Relates to both ureters1119. Appendix:A. Emerges from posteromedial aspect of cecum *B. Is longer in male than in femaleC. Has 2 bands of longitudinal muscleD. Cover entirely by peritoneumE. Relates to both ureters1120. Appendix:A. Is longer in male than in femaleB. Has 2 bands of longitudinal muscleC. Cover entirely by peritoneumD. Lumen is irregular *E. Relates to both ureters1121. Appendix:A. Has numerous lymphoid tissue in mucosa *B. Is longer in male than in femaleC. Has tenia coliD. Cover entirely by peritoneumE. Relates to both ureters1122. Appendix:A. Is longer in male than in femaleB. Has 2 bands of longitudinal muscleC. Cover entirely by peritoneumD. Has prominent muscularis mucosaE. Opens into posteromedial wall of cecum *1123. Appendix:A. Has tenia coliB. Has 2 bands of longitudinal muscleC. Has a mesentery *D. Cover entirely by peritoneumE. Relates to both ureters1124. Appendix:A. Has a mesenteryB. Opens into posteromedial wall of cecumC. Has numerous lymphoid tissue in mucosaD. Lumen is irregularE. All of the above statements are true *1125. Appendix:A. All of these statements are untrue *B. Has tenia coliC. Has 2 bands of longitudinal muscleD. Is longer in male than in femaleE. Has prominent muscularis mucosa1126. Meckel’s diverticulum is:A. Derived from cephalic end of hindgut

Page 138: 1intranet.tdmu.edu.ua/data/kafedra/internal/xirtop/zbtest... · Web viewModule 1. Operative surgery and topographical anatomy of the head and neck, regions and organs of the thoracic

B. Derived from vitello-intestinal duct *C. Supplied by the inferior mesenteric arteryD. Has venous drainage in inferior vena cava systemE. Opens into posteromedial wall of cecum1127. Meckel’s diverticulum is:A. Derived from cephalic end of hindgutB. Supplied by the inferior mesenteric arteryC. Has venous drainage in inferior vena cava systemD. Usually situated on distance 120 cm (approximately) from the ileocecal valve *E. Opens into posteromedial wall of cecum1128. Meckel’s diverticulum is:A. Sometimes lined by ectopic gastric mucosa *B. Derived from cephalic end of hindgutC. Supplied by the inferior mesenteric arteryD. Has venous drainage in inferior vena cava systemE. Opens into posteromedial wall of cecum1129. Regarding appendix:A. Pain is at T10 segmentB. Lined by columnar epitheliumC. Appendicular artery is an end arteryD. All of the above *E. None of the above1130. Megacolon:

A. Is covered anteriorly, along its whole length, by peritoneumB. Results from absence of all sympathetic ganglion cells in mesenteric plexusesC. Sympathetic ganglion cells are absent proximal to dilatationD. The dilated colon has normal nerve supply *E. Results from failure of development of muscular wall

1131. Jejunum differs from ileum in that it has:A. Wider lumen *B. Thin wallC. More lymphoid tissueD. More arterial arcades in its mesenteryE. Drained into internal ileac nodes1132. Jejunum differs from ileum in that it has:A. Thin wallB. More lymphoid tissueC. Often empty lumen *D. More arterial arcades in its mesenteryE. Drained into internal ileac nodes1133. Meckel’s diverticulum (when present):A. Contains gastric or pancreatic tissue *B. Develop from ventral mesoduodenumC. Always produces symptomsD. Has three constrictionsE. Obstruction can cause hematenesis1134. Meckel’s diverticulum (when present):A. Develop from ventral mesoduodenumB. Always produces symptomsC. Has three constrictionsD. May ulcerate or perforate *E. Obstruction can cause hematenesis1135. Meckel’s diverticulum (when present):A. Develop from ventral mesoduodenum

Page 139: 1intranet.tdmu.edu.ua/data/kafedra/internal/xirtop/zbtest... · Web viewModule 1. Operative surgery and topographical anatomy of the head and neck, regions and organs of the thoracic

B. Attached to terminal ileum *C. Obstruction can cause hematenesis D. Always produces symptomsE. Has three constrictions1136. Regarding appendix:

A. Pain is at T10 segment *B. Common site is pelvisC. 3 tenia coli are presentD. Has 2 bands of longitudinal muscleE. Has prominent muscularis mucosa

1137. Regarding appendix:A. Common site is pelvisB. 3 tenia coli are presentC. Has 2 bands of longitudinal muscle D. Lined by columnar epithelium *E. Is longer in male than in female

1138. Regarding appendix:A. Appendicular artery is an end artery *B. Common site is pelvisC. 3 tenia coli are presentD. Is longer in male than in femaleE. None of the above

1139. Lymphatic spread of carcinoma of the sigmoid colon is likely to metastasize to the;A. Left common ileac nodesB. Inferior mesenteric nodes *C. Superior mesenteric nodesD. Celiac nodesE. Para-aortic nodes

1140. Bacteria from diverticulitis of the proximal two thirds of the transverse colon are likely to spread via lymph vessels to the:

A. Inferior mesenteric nodesB. Celiac nodesC. Para-aortic nodesD. Superior mesenteric nodes *E. Left renal nodes1141. Which of the following is type of a gastrojejunostomy?A. Bilrot’s I operationB. Antecolica anterior *C. Side-to-side anastomosesD. Side-to-end anastomosesE. End-to-end anastomoses1142. Which of the following is type of a gastrojejunostomy?A. Antecolica anteriorB. Retrocolica anteriorC. Antecolica posteriorD. Retrocolica posteriorE. All of the above *1143. Topography of the root of mesentery:A. Left L2 to right sacroileac junction *B. Right L2 to left sacroileac junctionC. Left L1 to left sacroileac junctionD. Right Th12 to left sacroileac junctionE. Left Th12 to right sacroileac junction1144. Inferior abdominal compartment is distance from the:

Page 140: 1intranet.tdmu.edu.ua/data/kafedra/internal/xirtop/zbtest... · Web viewModule 1. Operative surgery and topographical anatomy of the head and neck, regions and organs of the thoracic

A. Mesentery of the transverse colon to the pelvic brim *B. Mesentery of the transverse colon to the superior edge of the rectumC. Diaphragm to the root of mesenteryD. Diaphragm to the mesentery of the transverse colonE. Inferior part of the duodenum to the pelvic brim1145. What is critical Zudeck’s point of blood supplying?A. Point of rectal artery divideB. Point of sigmoid artery dividing *C. Point of inferior mesentery artery divideD. Point of superior mesentery artery divideE. Point of ileocolica artery divide1146. Which of the following is not a branch of the superior mesenteric artery?A. Inferior pancreaticoduodenal arteryB. Ileocolic arteryC. Right colic arteryD. Middle colic arteryE. Left colic artery *1147. Which of the following is a branch of the inferior mesenteric artery?A. Left colic arteryB. Sigmoid arteryC. Superior rectal arteryD. All of the above *E. None of these1148. Topographical anatomy studies:

A. System of organsB. Variants of organ structureC. Organs innervation and blood supplyD. Organs age changesE. Morphological structure of organism regions *

1149. What is skeletotopy:A. Anatomical structures attitude to body partsB. Anatomical structures attitude to organism regionsC. Variants of organs blood supplyD. Relationship between organsE. Anatomical structures attitude to skeleton bones *

1150. What is holotopyA. Anatomical structures attitude to skeleton bonesB. Anatomical structures attitude to organism regionsC. Variants of organs blood supplyD. Relationship between organsE. Anatomical structures attitude to body parts *

1151. What is syntopyA. Anatomical structures attitude to body partsB. Anatomical structures attitude to skeleton bonesC. Anatomical structures attitude to organism regionsD. Variants of organs blood supplyE. Relationship between organs *

1152. Indicate type of local anesthesiaA. Conduction anesthesiaB. Intravenous anesthesiaC. Spinal anesthesiaD. Endotracheal anesthesiaE. Infiltration anesthesia *

1153. Bleeding from dural sinuses cann’t be arrested because:

Page 141: 1intranet.tdmu.edu.ua/data/kafedra/internal/xirtop/zbtest... · Web viewModule 1. Operative surgery and topographical anatomy of the head and neck, regions and organs of the thoracic

A. High pressure in dural sinusesB. Many venous tributariesC. Negative pressure in dural sinusesD. High speed of blood flowE. They are not collasible *

1154. Which of the following arteries is damage in case of temporal region injuryA. Anterior cerebral arteryB. Medial cerebral arteryC. Superficial temporal arteryD. Anterior meningeal arteryE. Medial meningeal artery *

1155. Which of the following methods of arrest of bleeding is temporary:A. TamponadeB. ElectrocoagulationC. Vessels ligation in woundD. Hemostatic spongeE. Hemostatic bandage *

1156. Which of the following methods of arrest of bleeding is permanent:A. TourniquetB. Hemostatic bandageC. Pressing in the woundD. TamponadeE. Hemostatic sponge *

1157. Which of the following methods of arrest of bleeding is temporaryA. TamponadeB. ElectrocoagulationC. Vessels ligation in woundD. Vessels ligation with suturingE. Pressing in the wound *

1158. In which layer of calvarium hematoma has bump formA. Subaponeurotical tissueB. Subperiosteal tissueC. Diploe substanceD. SkinE. Subcutaneous tissue *

1159. In which layer of calvarium hematoma has diffuse characterA. SkinB. Subcutaneous tissueC. Diploe substanceD. Subperiosteal tissueE. Subaponeurotical tissue *

1160. In which layer of calvarium hematoma has bone formA. SkinB. Subcutaneous tissueC. Diploe substanceD. Subaponeurotical tissueE. Subperiosteal tissue *

1161. Which of the following layers form scalping flapA. Periosteum, subperiosteal tissue, boneB. Epicranial aponeurosis, subaponeurotical tissueC. Subcutaneous tissue, epicranial aponeurosis, subaponeurotical tissueD. External lamina of bone, diploe, periosteumE. Skin, subcutaneous tissue, epicranial aponeurosis *

1162. The head region divides on which of the following parts

Page 142: 1intranet.tdmu.edu.ua/data/kafedra/internal/xirtop/zbtest... · Web viewModule 1. Operative surgery and topographical anatomy of the head and neck, regions and organs of the thoracic

A. Cerebral, occipitalB. Occipital, frontalC. Temporal, occipitalD. Frontal, occipitalE. Cerebral, facial *

1163. Indicate the anterior border of frontoparietooccipital regionA. Superior temporal lineB. Zygomatic arch, frontal processC. Superior nuchal lineD. Zygomatic archE. Glabella, supraorbital edge of frontal bone *

1164. Which of the following structures forms the posterior border of frontoparietooccipital region

A. Medial nuchal lineB. Inferior nuchal lineC. Inferior temporal lineD. Superior temporal lineE. Superior nuchal line *

1165. Which of the following structures forms the lateral border of frontoparietooccipital region

A. Medial temporal lineB. Inferior temporal lineC. Mastoid processD. Zygomatic archE. Superior temporal line *

1166. The epicranial aponeurosis locates between which of the following structuresA. Frontal and occipital musclesB. Left and right temporal musclesC. Frontal and occipital bonesD. Left and right temporal bonesE. Frontal and occipital bellies of frontooccipital muscle *

1167. Which of the following vessels locates in subaponeurotical tissue of frontoparietooccipital region

A. Temporal superficial arteryB. Temporal superficial artery and veinC. Posterior auricular arteryD. Diploe locatesE. No vessels *

1168. Diploe locatesA. Under the skinB. Under the epicranial aponeurosisC. Under the temporal muscleD. Between laminas of the temporal aponeurosisE. Between external and internal laminae of skull bone *

1169. Which of the following arteries furnish circulation of occipital regionA. Left and right occipital arteriesB. Temporal and occipital arteriesC. Left and right vertebral arteriesD. Posterior cerebral arteryE. Left and right occipital arteries, posterior auricular arteries *

1170. Which of the following arteries is source of occipital arteryA. Internal carotid arteryB. Auricular arteriesC. Vertebral arteries

Page 143: 1intranet.tdmu.edu.ua/data/kafedra/internal/xirtop/zbtest... · Web viewModule 1. Operative surgery and topographical anatomy of the head and neck, regions and organs of the thoracic

D. Medial temporal arteryE. External carotid artery *

1171. How many venous layers is in frontoparietooccipital regionA. OneB. TwoC. FourD. FiveE. Three *

1172. The first venous layer of frontoparietooccipital region locates in which of the following

A. Subaponeurotical tissueB. Subperiosteal tissueC. DiploeD. Epidural spaceE. Subcutaneous tissue *

1173. The second venous layer of frontoparietooccipital region locates in which of the following

A. Subcutaneous tissueB. Subaponeurotical tissueC. Subperiosteal tissueD. Epidural spaceE. Diploe *

1174. Venous drainage through emissary veins realizes in which of the following directionsA. To the veins of the head soft tissuesB. To the venous sinusesC. To the temporal superficial veinD. To the sigmoid sinusE. To the veins of the head soft tissues and venous sinuses *

1175. Where locates the auriculotemporal nerveA. In the interaponeurotical tissue of temporal regionB. In the subaponeurotical tissue of temporal regionC. In the subperiosteal tissue of temporal regionD. In the epidural space of temporal regionE. In the subcutaneous tissue of temporal region *

1176. Tendinous septa locate in which of the following calvarium fat tissuesA. SubperiostealB. SubaponeuroticalC. InteraponeuroticalD. All cellular spacesE. Subcutaneous *

1177. Which of the following arteries is source of ophthalmic arteryA. Common carotid arteryB. External carotid arteryC. Ethmoidal arteryD. Medial cerebral arteryE. Internal carotid artery *

1178. Which of the following arteries is source of supratrochlear arteryA. Anterior cerebral arteryB. Anterior meningeal arteryC. Medial cerebral arteryD. Ethmoidal arteryE. Ophthalmic artery *

1179. Which of the following arteries is source of temporal superficial arteryA. Facial artery

Page 144: 1intranet.tdmu.edu.ua/data/kafedra/internal/xirtop/zbtest... · Web viewModule 1. Operative surgery and topographical anatomy of the head and neck, regions and organs of the thoracic

B. Maxillary arteryC. Supratrochlear arteryD. Common carotid arteryE. External carotid artery *

1180. Which of the following arteries is source of anterior auricular arteryA. Facial arteryB. Maxillary arteryC. Internal carotid arteryD. External carotid arteryE. Temporal superficial artery *

1181. Which of the following arteries is source of deep temporal arteryA. Temporal superficial arteryB. Facial arteryC. Internal carotid arteryD. External carotid arteryE. Maxillary artery *

1182. Which of the following arteries is source of middle temporal arteryA. Parietal arteryB. Ophthalmic arteryC. Maxillary arteryD. External carotid arteryE. Temporal superficial artery *

1183. Venous return from temporal superficial vein realize inA. Facial veinB. Maxillary veinC. Parietal veinD. Angular veinE. Retromandibular vein *

1184. Temporal superficial artery locates in which of the following calvarium layersA. Subaponeurotical tissueB. Subperiosteal tissueC. Interaponeurotical tissueD. Epidural spaceE. Subcutaneous tissue *

1185. When permanent arrest of bleeding from skull bones is correctA. CoagulationB. Hemostatic spongeC. Hot physiologic salineD. Vessels ligation by catgutE. Rodding *

1186. Which of the following arteries is source of zygomaticoorbital arteryA. Ophthalmic arteryB. Temporal superficial arteryC. Facial arteryD. Internal carotid arteryE. Maxillary artery *

1187. How realize the permanent arrest of bleeding from skull bonesA. By the catgut ligationB. By the using of hot physiologic salineC. By the electrocoagulationD. By the clippingE. By the rubbing of wax *

1188. How realize the permanent arrest of bleeding from meningeal arteriesA. By the catgut ligation

Page 145: 1intranet.tdmu.edu.ua/data/kafedra/internal/xirtop/zbtest... · Web viewModule 1. Operative surgery and topographical anatomy of the head and neck, regions and organs of the thoracic

B. By the rubbing of waxC. By the using of hot physiologic salineD. By the electrocoagulationE. By the clipping. *

1189. Which of the following arteries is source of posterior auricular arteryA. Facial arteryB. Maxillary arteryC. Internal carotid arteryD. Common carotid arteryE. External carotid artery. *

1190. How realize the permanent arrest of bleeding from skull bonesA. By the catgut ligationB. By the using of hot physiologic salineC. By the electrocoagulationD. By the clipping.E. By the rubbing of rasping of a bone *

1191. Penetretive skull wound is:A. Damage of skull boneB. Damage of dura mater and arachnoid of brainC. Damage of all meningesD. Damage of internal lamina of skull bonesE. Damage of dura mater of brain *

1192. Venous layers connects by which of the following veinsA. Diploidic veinsB. Venous sinusesC. Venous plexusesD. Superficial veins of calvarium.E. Emissary veins *

1193. Frontal nerve is branch of which of followingB. Facial nerveC. Abducent nerveD. Optic nerveE. Trochlear nerve.F. Ophthalmic nerve *1194. Frontal belly of frontooccipital muscle is innervated by which of the following nerve:

A. Abducent nerveB. Ophthalmic nerveC. Optic nerveD. Maxillary nerve.E. Facial nerve *

1195. Lesser occipital nerve is branch of which of the following nerve:A. Spinal nerveB. Posterior auricular nerveC. Facial nerveD. Trigeminal nerveE. Cervical plexus *

1196. Greater occipital nerve is branch of which of the following nerve:A. Third spinal nerveB. First spinal nerveC. Cervical plexusD. Vagus nerve.E. Second spinal nerve *

1197. Posterior auricular nerve is branch of which of the following nerveA. First spinal nerve

Page 146: 1intranet.tdmu.edu.ua/data/kafedra/internal/xirtop/zbtest... · Web viewModule 1. Operative surgery and topographical anatomy of the head and neck, regions and organs of the thoracic

B. Second spinal nerveC. Vagus nerveD. Phrenic nerve.E. Facial nerve *

1198. Lymphatic outflow from the frontal region of calvarium realize to which of the following lymph nodes:

A. Superficial parotid lymph nodesB. Deep parotid lymph nodesC. Superficial, middle and deep parotid lymph nodesD. Facial lymph nodes.E. Superficial and deep parotid lymph nodes *

1199. Lymphatic outflow from the parietal region of calvarium realize to which of the following lymph nodes:

A. Parotid lymph nodesB. Deep parotid lymph nodesC. Occipital lymph nodesD. Facial lymph nodes.E. Retroauricular lymph nodes *

1200. Where locate subperiostal hematoma and abscessA. Subcutaneous tissueB. Subaponeurotical tissueC. Interaponeurotical tissueD. PeriosteumE. Subperiostal tissue *

1201. In sponge substance of skull bone locates which of the following structuresA. Venous sinusesB. Cranial nervesC. Deep arteries of calvariumD. Emissary veinsE. Diploidic veins *

1202. Which anatomical structures pass through the optic canalA. Optic nerve, ophthalmic veinB. Ophthalmic artery, ophthalmic veinC. Optic nerve, vein and arteryD. Ophthalmic nerve, ophthalmic vein and arteryE. Optic nerve, ophthalmic artery and vein *

1203. Which anatomical structures pass through the round foramenA. Ophthalmic and maxillary nerveB. Ophthalmic and mandibular nerveC. Maxillary and mandibular nerveD. Optic and maxillary nerveE. Maxillary nerve *

1204. Cecal foramen locates inA. Middle cranial fossaB. Posterior cranial fossaC. Sphenoid boneD. Near optic chiasmE. Anterior cranial fossa *

1205. Pacchionian granulations is formations of which of the following structuresA. Dura mater of brainB. Pia mater of brainC. Venous sinusesD. Pericranium.E. Arachnoid of brain *

Page 147: 1intranet.tdmu.edu.ua/data/kafedra/internal/xirtop/zbtest... · Web viewModule 1. Operative surgery and topographical anatomy of the head and neck, regions and organs of the thoracic

1206. Which nerve pass through the oval foramenA. Optic nerveB. Maxillary nerveC. Facial nerveD. Trochlear nerve.E. Mandibular nerve *

1207. Which anatomical structure pass through the spinous foramenA. Ethmoidal arteryB. Anterior meningeal arteryC. Posterior meningeal arteryD. Middle cerebral arteryE. Middle meningeal artery *

1208. Internal skull base forms by which of the following structuresA. Temporal and infratemporal fossaB. Orbital and nasal cavityC. Trigeminal impression and orbital cavityD. Superior and inferior orbital fissureE. Anterior, middle and posterior cranial fossa *

1209. Indicate brain structures, which form venous sinusesA. PericraniumB. Pia mater encephaliC. Arachnoid encephaliD. Epidural spaceE. Dura mater encephali *

1210. Inferior sagittal sinus locates onA. Upper edge of falx of cerebrumB. Sulcus of transverse sinus of occipital boneC. Edge of Turkish saddleD. Edge of internal occipital tuberosityE. Lower edge of falx of cerebrum *

1211. Which of the following sinuses realizes venous drainage from the skullA. Superior sagittal sinusB. Inferior sagittal sinusC. Transverse sinusD. Rectal sinusE. Cavernous sinus *

1212. Which of the following vessels pass through the cavernous sinusA. External carotid arteryB. Ophthalmic arteryC. Middle meningeal arteryD. Ophthalmic veinE. Internal carotid artery *

1213. Middle meningeal artery locates inA. Subdural spaceB. Subarachnoid spaceC. Subperiostal tissueD. Epidural and subdural spaceE. Epidural space *

1214. Superior sagittal sinus locates onA. Lower edge of falx of cerebrumB. Edge of falx of cerebellumC. Edge of Turkish saddleD. Edge of internal occipital tuberosityE. Upper edge of falx of cerebrum *

Page 148: 1intranet.tdmu.edu.ua/data/kafedra/internal/xirtop/zbtest... · Web viewModule 1. Operative surgery and topographical anatomy of the head and neck, regions and organs of the thoracic

1215. Rectal sinus locates onA. Edge of falx of cerebellumB. Upper edge of falx of cerebrumC. Sulcus of transverse sinus of occipital boneD. Edge of Turkish saddleE. Connective place of falx and tentorium of cerebellum *

1216. Transverse sinus locates onA. Sulcus between cerebellar hemispheresB. Edge of falx of cerebellumC. Edge of Turkish saddleD. Lower edge of falx of cerebrumE. Junction between tentorium of cerebellum and sulcus of transverse sinus of occipital

bone *1217. How realize the arrest of bleeding from cerebral vessels

A. By the catgut ligationB. By the rubbing of waxC. By the using of hot physiologic salineD. By the electrocoagulationE. By the clipping *

1218. How realize the arrest of bleeding from superior sagittal sinusA. By the electrocoagulationB. By the clippingC. By the ligationD. By the rubbing of waxE. By the vessel suture *

1219. Subcutaneous emphysema is possible in case of damage of which of the following structures:

A. Turkish saddleB. Cavernous sinusC. Superior sagittal sinusD. Sigmoid sinusE. Frontal sinus *

1220. Which of the following nerves pass through the cavernous sinusA. Olfactory nervesB. Optic nerveC. Mandibular nerveD. Maxillary nerveE. Abducent nerve *

1221. Which of the following nerves pass through the cavernous sinusA. Optic nerveB. Maxillary nerveC. Facial nerveD. Olfactory nervesE. Oculomotor nerve *

1222. Which of the following nerves pass through the cavernous sinusA. Maxillary nerveB. Accessory nerveC. Facial nerveD. Optic nerveE. Ophthalmic nerve *

1223. Which of the following nerves pass through the cavernous sinusA. Facial nerveB. Accessory nerveC. Vagus nerve

Page 149: 1intranet.tdmu.edu.ua/data/kafedra/internal/xirtop/zbtest... · Web viewModule 1. Operative surgery and topographical anatomy of the head and neck, regions and organs of the thoracic

D. None of the aboveE. Trochlear nerve *

1224. Which of the following anatomical structures produces the liquorA. Pacchionian granulationsB. Lateral ventricles of cerebrumC. Third ventricle of cerebrumD. Subarachnoid spaceE. Vascular plexuces *

1225. Which of the following nerves realizes the innervetion of dura mater of brainA. Facial nerveB. Hypoglossal nerveC. Optic nerveD. Olfactory nerveE. Accessory nerve *

1226. Indicate the reason of convergent strabismusA. Damage of facial nerveB. Damage of optic nerveC. Ophthalmic vein thrombosisD. Damage of ophthalmic arteryE. Cavernous sinus thrombosis *

1227. Which of the following anatomical structures pass through the great occipital foramen:

A. Temporal superficial arteryB. Occipital arteryC. Great occipital nerveD. None of the aboveE. Left vertebral artery *

1228. Which of the following anatomical structures pass through the great occipital foramen:

A. Hypoglossal nerveB. Occipital arteryC. Posterior auricular arteryD. Facial nerveE. Right vertebral artery *

1229. Which of the following anatomical structures pass through the great occipital foramen:

A. Spinal cordB. Occipital arteryC. Occipital veinD. Occipital artery and veinE. Medulla oblongata *

1230. Which of the following anatomical structures pass through the anterior part of jugular foramen:

A. Jugular arteryB. Jugular veinC. Bulb of jugular vein and arteryD. None of the aboveE. IX, X, XI cranial nerves *

1231. Liquor passes from 4-th ventricle of cerebrum to subarachnoid space through which of the following foramen:

A. Interventricular Monro’s foramenB. Cerebral Silvii’s aqueductC. Roland’s foramenD. Lushka’s foramen

Page 150: 1intranet.tdmu.edu.ua/data/kafedra/internal/xirtop/zbtest... · Web viewModule 1. Operative surgery and topographical anatomy of the head and neck, regions and organs of the thoracic

E. Magendie’s foramen *1232. Magendie’s foramen connects which of the following structures

A. Later ventricles of cerebrumB. 2-nd ventricle with 3-rdC. 3-rd venticle with 4-thD. 4-th venticle with subarachnoid space of spinal cordE. 4-th venticle with subarachnoid cavity *

MSQs for figures:

1233. Which of the following anatomical structures in fig. 4 is indicated by 1? A. m. pterygoideus lateralisB. a. supraorbitalisC. m. masseterD. a. et m. pterygoideus lateralis *E. a. facialis

1234. Which of the following anatomical structures in fig. 4 is indicated by 2?A. a. pterygoideus lateralisB. a. supraorbitalis *C. a. supratrochlearisD. a.ophtalmicaE. a. facialis

1235. Which of the following anatomical structures in fig. 4 is indicated by 3?A. a. pterygoideus lateralisB. a. supraorbitalisC. a. supratrochlearis *D. a.ophtalmicaE. a.facialis

1236. Which of the following anatomical structures in fig. 4 is indicated by 4?A. a. pterygoideus lateralisB. a. supraorbitalisC. a. supratrochlearisD. a.ophtalmica *E. a.facialis

1237. Which of the following anatomical structures in fig. 4 is indicated by 5?A. a. pterygoideus lateralisB. a. supraorbitalisC. a. supratrochlearisD. a.ophtalmicaE. a.dorsalis nasi *

1238. Which of the following anatomical structures in fig. 4 is indicated by 6?A. a. angularis *B. a. supraorbitalisC. a. supratrochlearisD. a.ophtalmicaE. a.dorsalis nasi

1239. Which of the following anatomical structures in fig. 4 is indicated by 7?A. a. angularisB. a. supraorbitalis

Page 151: 1intranet.tdmu.edu.ua/data/kafedra/internal/xirtop/zbtest... · Web viewModule 1. Operative surgery and topographical anatomy of the head and neck, regions and organs of the thoracic

C. a. supratrochlearisD. a.ophtalmicaE. a.infraorbitalis *

1240. Which of the following anatomical structures in fig. 4 is indicated by 8?A. a. angularisB. aa. alveolares superiores posteriores *C. aa. alveolares superiores anterioresD. aa. alveolares superiores mediales E. a. maxillaris

1241. Which of the following anatomical structures in fig. 4 is indicated by 10?A. a. angularisB. aa. alveolares superiores anteriores *C. aa. alveolares superiores posterioresD. aa. alveolares superiores mediales E. a. maxillaris

1242. Which of the following anatomical structures in fig. 4 is indicated by 9?A. a. angularisB. aa. alveolares superiores mediales *C. aa. alveolares superiores anterioresD. aa. alveolares superiores posterioresE. a. maxillaris

1243. Which of the following anatomical structures in fig. 4 is indicated by 11?A. a. et n. buccales *B. a.et n. pterigoideus lat.C. a.et n. pterigoideus med.D. aa. alveolares superiores posterioresE. a.et n. maxillaris

1244. Which of the following anatomical structures in fig. 4 is indicated by 12?A. a. et n. buccalesB. a.et n. pterigoideus lat.C. a.et m. pterigoideus med. *D. aa. alveolares superiores posterioresE. a.et m. masseter

1245. Which of the following anatomical structures in fig. 4 is indicated by 13?A. n. buccalisB. raphe pterigomandibularis *C. n. lingualisD. n. mandibularisE. n. facialis

1246. Which of the following anatomical structures in fig. 4 is indicated by 14?A. n. buccalisB. raphe pterigomandibularisC. n. lingualis *D. n. mandibularisE. n. facialis

Page 152: 1intranet.tdmu.edu.ua/data/kafedra/internal/xirtop/zbtest... · Web viewModule 1. Operative surgery and topographical anatomy of the head and neck, regions and organs of the thoracic

1247. Which of the following anatomical structures in fig. 4 is indicated by 15?A. a. buccalisB. a. pterigomandibularisC. a. lingualisD. a. mandibularisE. a. facialis *

1248. Which of the following anatomical structures in fig. 4 is indicated by 16?A. a. buccalisB. a. mentalis *C. a. lingualisD. a. submentalisE. a. facialis

1249. Which of the following anatomical structures in fig. 4 is indicated by 17?A. a. buccalisB. a. submentalis *C. a. mentalisD. a. lingualisE. a. facialis

1250. Which of the following anatomical structures in fig. 4 is indicated by 18?A. a. maxillarisB. a. submentalisC. a. mentalisD. a. lingualis *E. a. facialis

1251. Which of the following anatomical structures in fig. 4 is indicated by 19?A. a. maxillarisB. a. submentalisC. a. mentalisD. a. facialis *E. a. lingualis

1252. Which of the following anatomical structures in fig. 4 is indicated by 20?A. a. maxillarisB. a. carotis externa *C. a. carotis communisD. a. carotis internaE. a. occipitalis

1253. Which of the following anatomical structures in fig. 4 is indicated by 21?A. m. sthylohyoideus *B. m. pterygoideus lateralisC. m. pterygoideus medialisD. m. digastricusE. m. sterocleidomastoideus

1254. Which of the following anatomical structures in fig. 4 is indicated by 22?A. m. sthylohyoideusB. m. pterygoideus lateralisC. m. pterygoideus medialis

Page 153: 1intranet.tdmu.edu.ua/data/kafedra/internal/xirtop/zbtest... · Web viewModule 1. Operative surgery and topographical anatomy of the head and neck, regions and organs of the thoracic

D. m. digastricus *E. m. sterocleidomastoideus

1255. Which of the following anatomical structures in fig. 4 is indicated by 23?A. a. et n. mylohyoideus *B. a. et n. sthylohyoideusC. a. et n. alveolaris inferiorisD. a. et n. pterygoideusE. a. et n. lingualis

1256. Which of the following anatomical structures in fig. 4 is indicated by 24?A. Lig. mylohyoideusB. Lig.. sthylohyoideusC. Lig. Sphenomandibularis *D. lig. pterygoideus lat.E. Lig. pterygoideus med.

1257. Which of the following anatomical structures in fig. 4 is indicated by 25?A. A. et n. mylohyoideusB. A. et n. alveolaris inferior *C. A. et n. pterygoideus lat.D. A. et n. pterygoideus med.E. A. et n. lingualis

1258. Which of the following anatomical structures in fig. 4 is indicated by 26?A. n. maxillarisB. n. mandibularisC. n. pterygoideus lat.D. n. pterygoideus med.E. n. facialis *

1259. Which of the following anatomical structures in fig. 4 is indicated by 27?A. A. occipitalisB. A. auricularis posterior *C. A. temporalis superficialisD. A. pterygoideus med.E. A. temporalis profunda

1260. Which of the following anatomical structures in fig. 4 is indicated by 28?A. A. occipitalisB. A. auricularis posteriorC. A. temporalis superficialis *D. A. maxillarisE. A. carotis externa

1261. Which of the following anatomical structures in fig. 4 is indicated by 29?A. A. massetericaB. A. auricularis anteriorC. A. temporalis superficialisD. A. maxillaris *E. A. carotis externa

1262. Which of the following anatomical structures in fig. 4 is indicated by 30?A. n. auriculotemporalis *

Page 154: 1intranet.tdmu.edu.ua/data/kafedra/internal/xirtop/zbtest... · Web viewModule 1. Operative surgery and topographical anatomy of the head and neck, regions and organs of the thoracic

B. n. temporalis superficialisC. n. temporalis lat.D. n. temporalis med.E. n. auricularis

1263. Which of the following anatomical structures in fig. 4 is indicated by 31?A. A. temporalis mediaB. A. auricularis anteriorC. A. meningea media *D. A. temporalis profundaE. A. meningea posterior

1264. Which of the following anatomical structures in fig. 4 is indicated by 33?A. A. et n. masseterica *B. A. et n. alveolaris superiorisC. A. et n. pterygoideus lat.D. A. et n. pterygoideus med.E. A. et n. buccalis

1265. Which of the following anatomical structures in fig. 1 is indicated by 28?A. m.mylohyoideus *B. m. hyoglossusC. m. cricothyroideusD. m. sternothyroideusE. m. omohyoideus

1266. Which of the following anatomical structures in fig. 1 is indicated by 27?A. m.mylohyoideus B. m. hyoglossus *C. m. cricothyroideusD. m. sternothyroideusE. m. omohyoideus

1267. Which of the following anatomical structures in fig. 1 is indicated by 20?A. m.mylohyoideus B. m. hyoglossus C. m. cricothyroideus *D. m. sternothyroideusE. m. omohyoideus

1268. Which of the following anatomical structures in fig. 1 is indicated by 19?A. m.mylohyoideus B. m. hyoglossus C. m. cricothyroideus D. m. sternothyroideus *E. m. omohyoideus

1269. Which of the following anatomical structures in fig. 1 is indicated by 26?A. m.mylohyoideus B. m. hyoglossus C. m. stylohyoideus *D. m. sternothyroideusE. m. omohyoideus

Page 155: 1intranet.tdmu.edu.ua/data/kafedra/internal/xirtop/zbtest... · Web viewModule 1. Operative surgery and topographical anatomy of the head and neck, regions and organs of the thoracic

1270. Which of the following anatomical structures in fig. 1 is indicated by 16?A. m.mylohyoideus B. m. hyoglossus C. m. cricothyroideus D. m. sternothyroideus *E. m. omohyoideus

1271. Which of the following anatomical structures in fig. 1 is indicated by 18?A. m.mylohyoideus B. m. hyoglossus C. m. cricothyroideus D. m. sternothyroideusE. m. omohyoideus *

1272. Which of the following anatomical structures in fig. 1 is indicated by 13?A. m.mylohyoideus B. m. hyoglossus C. m. cricothyroideus D. m. sternothyroideusE. m. omohyoideus *

1273. Which of the following anatomical structures in fig. 1 is indicated by 12?A. m.trapezius *B. m. scalenus mediusC. m. cricothyroideus D. m. sternothyroideusE. m. omohyoideus

1274. Which of the following anatomical structures in fig. 1 is indicated by 7?A. m.mylohyoideus B. m. hyoglossus C. m. cricothyroideus D. m. sternothyroideusE. m. omohyoideus *

1275. Which of the following anatomical structures in fig. 6 is indicated by 1?A. n. lacrimalis *B. n. frontalisC. n. mandibularisD. n. temporalis profundusE. n. maxillaris

1276. Which of the following anatomical structures in fig. 6 is indicated by 12?A. n. lacrimalis B. n. frontalisC. n. mandibularisD. n. temporalis profundusE. n. maxillaris *

1277. Which of the following anatomical structures in fig. 6 is indicated by 26?A. n. lacrimalis B. n. frontalisC. n. mandibularisD. n. infraorbitalis *

Page 156: 1intranet.tdmu.edu.ua/data/kafedra/internal/xirtop/zbtest... · Web viewModule 1. Operative surgery and topographical anatomy of the head and neck, regions and organs of the thoracic

E. n. maxillaris

1278. Anatomical structure in fig. 6 indicated by 26 is branch of which of the following nerves?

A. n. vagusB. n. frontalisC. n. mandibularisD. n. temporalis profundusE. n. maxillaris *

1279. Which of the following anatomical structures in fig. 6 is indicated by 11?A. n. vagusB. n. frontalisC. n. mandibularis *D. n. temporalis profundusE. n. maxillaris

1280. Anatomical structure in fig. 6 marked by 11 goes from the cranial cavity through which of the following holes?

A. Foramen ovale *B. Foramen lacerumC. Foramen spinosumD. Foramen rotundumE. Foramen jugulare

1281. Anatomical structure in fig. 6 marked by 12 goes from the cranial cavity through which of the following holes?

A. Foramen ovale B. Foramen lacerumC. Foramen spinosumD. Foramen rotundum *E. Foramen jugulare

1282. Which of the following anatomical structures in fig. 5 is indicated by 4?A. Foramen ovale *B. Foramen lacerumC. Foramen spinosumD. Foramen rotundumE. Foramen jugulare

1283. Which of the following anatomical structures in fig. 5 is indicated by 8?A. n. auriculotemporalis *B. n. frontalisC. n. mandibularisD. n. temporalis profundusE. n. maxillaris

1284. Which of the following anatomical structures in fig. 5 is indicated by 10?A. n. auriculotemporalis B. n. facialis *C. n. mandibularisD. n. temporalis profundusE. n. maxillaris

Page 157: 1intranet.tdmu.edu.ua/data/kafedra/internal/xirtop/zbtest... · Web viewModule 1. Operative surgery and topographical anatomy of the head and neck, regions and organs of the thoracic

1285. Which of the following anatomical structures in fig. 5 is indicated by 12?A. n. lingualis *B. n. facialis C. n. mandibularisD. n. temporalis profundusE. n. maxillaris

1286. Which of the following anatomical structures in fig. 5 is indicated by 22?A. n. lingualis B. n. facialis C. n. mandibularisD. n. alveolaris inferioris *E. n. mentalis

1287. Which of the following anatomical structures in fig. 5 is indicated by 21?A. m. digastricus venter anterior *B. m. stylohyoideus C. m. mylohyoideusD. m. sternohyoideus E. m. omohyoideus

1288. Which of the following anatomical structures in fig. 5 is indicated by 16?A. m. digastricus venter posterior *B. m. stylohyoideus C. m. mylohyoideusD. m. sternohyoideus E. m. omohyoideus

1289. Which of the following anatomical structures in fig. 5 is indicated by 17?A. m. digastricus venter posterior B. m. stylohyoideus *C. m. mylohyoideusD. m. sternohyoideus E. m. omohyoideus

1290. Which of the following anatomical structures in fig. 5 is indicated by 24?A. m. digastricus venter posterior B. m. stylohyoideus C. m. mylohyoideus *D. m. sternohyoideus E. m. omohyoideus

1291. Which of the following anatomical structures in fig. 2 is indicated by 2?A. a. obturatoria *B. a. epigastrica inferior C. a. circumflexa ilium profunda D. m. sternohyoideus E. m. omohyoideus

1292. Which of the following anatomical structures in fig. 2 is indicated by 2?A. a. obturatoria *B. a. epigastrica inferior C. a. circumflexa ilium profunda D. ductus deferens

Page 158: 1intranet.tdmu.edu.ua/data/kafedra/internal/xirtop/zbtest... · Web viewModule 1. Operative surgery and topographical anatomy of the head and neck, regions and organs of the thoracic

E. a. pudenda interna

1293. Which of the following anatomical structures in fig. 2 is indicated by 5?A. ductus deferens *B. a. pudenda internaC. a. circumflexa ilium profundaD. a. epigastrica inferiorE. a. obturatoria

1294. Which of the following anatomical structures in fig. 2 is indicated by 7?A. Lig. lacunare *B. Lig. pectineumC. Lig. inguinaleD. Linea arcuataE. Anulus inguinalis superficialis

1295. Which of the following anatomical structures in fig. 2 is indicated by 14?A. Anulus inguinalis profundus *B. Anulus umbilicalisC. Lig. inguinaleD. Linea arcuataE. Anulus inguinalis superficialis

1296. Which of the following anatomical structures in fig. 2 is indicated by 8?A. Anulus femoralis *B. Anulus umbilicalisC. Lig. inguinaleD. Linea arcuataE. Anulus inguinalis superficialis

1297. Which of the following anatomical structures in fig. 2 is indicated by 11?A. M. iliopsoas *B. M. piriformisC. M. obturatoriusD. M. gemellus superiorE. M. gemellus inferior

1298. Which of the following anatomical structures in fig. 3 is indicated by 1?A. Lig. falciforme *B. Lig. teresC. Lig. inguinaleD. Lig. coronariumE. Lig. triangulare

1299. Which of the following anatomical structures in fig. 3 is indicated by 2?A. Lig. falciforme B. Lig. teres *C. Lig. inguinaleD. Lig. coronariumE. Lig. triangulare

1300. Which of the following anatomical structures in fig. 3 is indicated by 3?A. Lobus quadratus *

Page 159: 1intranet.tdmu.edu.ua/data/kafedra/internal/xirtop/zbtest... · Web viewModule 1. Operative surgery and topographical anatomy of the head and neck, regions and organs of the thoracic

B. Lobus dexterC. Lobus sinisterD. Lobus caudatusE. Processus caudatus

1301. Which of the following anatomical structures in fig. 3 is indicated by 16?A. Lobus quadratus B. Lobus dexterC. Lobus sinister *D. Lobus caudatusE. Processus caudatus

1302. Which of the following anatomical structures in fig. 3 is indicated by 4?A. Lobus quadratus B. Lobus dexter *C. Lobus sinisterD. Lobus caudatusE. Processus caudatus

1303. Which of the following anatomical structures in fig. 3 is indicated by 5?A. Vesica fellea *B. Lobus dexterC. Lobus sinisterD. Lobus caudatusE. Processus caudatus

1304. Which of the following anatomical structures in fig. 3 is indicated by 15?A. Lig. hepatoduodenale *B. Lig. hepatogastricaC. Lig. hepatorenaleD. Lig. duodenorenaleE. Lig. gastrocolicum

1305. Which of the following anatomical structures in fig. 3 is indicated by 14?A. Lig. hepatoduodenale B. Lig. Hepatogastrica *C. Lig. hepatorenaleD. Lig. duodenorenaleE. Lig. gastrocolicum

1306. Lig. hepatoduodenale is indicated by which of the following numbers in fig.3?A. 15 *B. 14C. 12D. 7E. 8

1307. Lig. hepatogastrica is indicated by which of the following numbers in fig.3?A. 15 B. 14 *C. 12D. 7E. 8

Page 160: 1intranet.tdmu.edu.ua/data/kafedra/internal/xirtop/zbtest... · Web viewModule 1. Operative surgery and topographical anatomy of the head and neck, regions and organs of the thoracic

1308. Which of the following forms the anatomical structures are indicated in fig. 3 by numbers 15 and 14?

A. Omentum minor *B. Omentum majusC. Mesocolon transversumD. Radix mesenteriiE. Bursae omentalis

1309. Which of the following forms the anatomical structure is indicated in fig. 3 by number 15?

A. Anterior wall of foramen epiploicum *B. Omentum majusC. Posterior wall of foramen epiploicumD. Radix mesenteriiE. Bursae omentalis

1310. Which of the following goes within the anatomical structure that is indicated in fig. 3 by number 15?

A. Ductus choledochus *B. A. hepatica communisC. Truncus coeliacusD. A. hepatica sinistraE. Ductus hepaticus dexter

1311. Which of the following goes within the anatomical structure that is indicated in fig. 3 by number 15?

A. A. hepatica propria *B. A. hepatica communisC. Truncus coeliacusD. A. hepatica sinistraE. Ductus hepaticus dexter

1312. Which of the following doesn’t go within the anatomical structure that is indicated in fig. 3 by number 15?

A. Ductus choledochusB. A. hepatica communis *C. A. hepatica propriaD. V. portaeE. All are not correct

1313. Which of the following limits the foramen epiploicum from behind (see fig. 3)? A. Lig. Hepatorenale *B. Lig. duodenorenale C. Lobus caudatusD. V. portaeE. All are not correct

1314. Which of the following limits the foramen epiploicum from below (see fig. 3)?A. Lig. hepatorenaleB. Lig. duodenorenale *C. Lobus caudatusD. V. portaeE. All are not correct

Page 161: 1intranet.tdmu.edu.ua/data/kafedra/internal/xirtop/zbtest... · Web viewModule 1. Operative surgery and topographical anatomy of the head and neck, regions and organs of the thoracic

1315. Which of the following limits the foramen epiploicum from above (see fig. 3)?A. Lig. hepatorenaleB. Lig. duodenorenale C. Lobus caudatus *D. V. portaeE. All are not correct

1316. Posterior wall of sheath of rectus muscle below the umbilicus is formed by which of the following anatomical structures (see fig. 2)?

A. F. transversa *B. Aponeurosis of m. obliquus abdominis int.C. Aponeurosis of m. obliquus abdominis ext.D. M. transversusE. All are not correct